<<

0523-00 FM 07/15/02 15:30 Page i

Sixth edition

APPLETON & LANGE REVIEW OF ANATOMY

Royce Lee Montgomery, PhD Professor Department of Cell and Developmental Biology School of Medicine University of North Carolina Chapel Hill, North Carolina

Kurt Ogden Gilliland, PhD Department of Cell and Developmental Biology School of Medicine University of North Carolina Chapel Hill, North Carolina

Appleton & Lange Reviews/McGraw-Hill Medical Publishing Division

New York Chicago San Francisco Lisbon London Madrid Mexico City Milan New Delhi San Juan Seoul Singapore Sydney Toronto 0523-00 FM 07/15/02 15:30 Page ii

Appleton & Lange Review of Anatomy, Sixth Edition Copyright © 2003 by The McGraw-Hill Companies, Inc. All rights reserved. Printed in the United States of America. Except as permitted under the United States Copyright Act of 1976, no part of this publication may be reproduced or distributed in any form or by any means, or stored in a data base or retrieval system, without the prior written permission of the publisher. Previous editions copyright © 1995, 1989, by Appleton & Lange; copyright © 1982, 1978, 1974, by Arco Publishing, Inc. 1 2 3 4 5 6 7 8 9 0 VNH VNH 0 9 8 7 6 5 4 3 2 ISBN: 0-07-137727-1

Notice Medicine is an ever-changing science. As new research and clinical experience broaden our knowledge, changes in treatment and drug therapy are required. The authors and the publisher of this work have checked with sources believed to be reliable in their efforts to provide information that is complete and generally in accord with the stan- dards accepted at the time of publication. However, in view of the possibility of human error or changes in med- ical sciences, neither the authors nor the publisher nor any other party who has been involved in the preparation or publication of this work warrants that the information contained herein is in every respect accurate or com- plete, and they disclaim all responsibility for any errors or omissions or for the results obtained from use of the information contained in this work. Readers are encouraged to confirm the information contained herein with other sources. For example and in particular, readers are advised to check the product information sheet included in the package of each drug they plan to administer to be certain that the information contained in this work is accurate and that changes have not been made in the recommended dose or in the contraindications for admin- istration. This recommendation is of particular importance in connection with new or infrequently used drugs.

This book was set in Palatino by Circle Graphics. The editors were Catherine W. Johnson and Lester A. Sheinis. The production supervisor was Lisa Mendez. The cover designer was Aimée Nordin. The indexer was Alexandra Nickerson. Von Hoffmann Graphics, Inc., was printer and binder. This book was printed on acid-free paper. Library of Congress Cataloging-in-Publication Data Montgomery, Royce L. Appleton & Lange review of anatomy / Royce L. Montgomery, Kurt Ogden Gilliland.—6th ed. p. ; cm. Rev. ed. of: Appleton & Lange review of anatomy for the USMLE Step 1 / Royce L. Montgomery, Gerald A. Montgomery. © 1995. Includes bibliographical references. ISBN 0-07-137727-1 (alk. paper) 1. Human anatomy—Examinations, questions, etc. 2. Physicians—Licenses—United States—Examinations—Study guides. I. Title: Appleton & Lange review of anatomy. II. Title: Review of anatomy. III. Gilliland, Kurt Ogden. IV. Montgomery, Royce L. Appleton & Lange review of anatomy for the USMLE Step 1. V. Title. [DNLM: 1. Anatomy—Examination Questions. QS 18.2 M788a 2003] QM32 .M65 2002 611′.0076—dc21 2002016672 International Edition ISBN 0-07-121248-5 Copyright © 2003. Exclusive rights by The McGraw-Hill Companies, Inc., for manufacture and export. This book cannot be reexported from the country to which it is consigned by McGraw-Hill. The International Edition is not available in North America. 0523-00 FM 07/15/02 15:30 Page iii

Contents

Introduction ...... v

1. The Back ...... 1 Answers and Explanations ...... 7 2. The Upper Limb ...... 11 Answers and Explanations ...... 23 3. The ...... 31 Answers and Explanations ...... 41 4. The ...... 47 Answers and Explanations ...... 60 5. The and ...... 67 Answers and Explanations ...... 79 6. The Lower Limb ...... 87 Answers and Explanations ...... 100 7. The Head and ...... 109 Answers and Explanations ...... 132

References ...... 141

Index ...... 143

iii This page intentionally left blank 0523-00 FM 07/15/02 15:30 Page v

Introduction

If you are planning to prepare for the United States Each of the chapters contains single-best answer Medical Licensing Examination (USMLE) Step 1, multiple choice questions. In some cases, a group of then this book is designed for you. Here, in one pack- two or three questions may be related to a situa- age, is a comprehensive review resource with over tional theme. In addition, some questions have illus- 600 examination type anatomy multiple-choice ques- trative material (e.g., line illustrations of anatomy) tions with referenced explanations of each answer. that require understanding and interpretation on This introduction provides specific information your part. Moreover, questions may be of three on the USMLE Step 1, information on question types, levels of difficulty: rote memory, memory question question-answering strategies, and various ways to that requires more understanding of the problem, use this review. and a question that requires both understanding and judgment. In view of the fact that the USMLE Step 1 is moving toward the judgment, critical-thinking THE UNITED STATES MEDICAL LICENSING type question, we have attempted to write this EXAMINATION STEP 1 review with this emphasis.

The United States Medical Licensing Examination One Best-Answer-Single Item Question. The major- Step 1 is a one-day computerized examination con- ity of the questions are posed in the A type, or “one sisting of approximately 400 questions to test your best answer single item” format. This is the most knowledge in the basic sciences. It contains multiple- popular question format in most exams. It generally choice questions organized within three dimensions. contains a brief statement, followed by five options Each dimension is weighted; however, the projected of which only ONE is entirely correct. The options percentages for these dimensions are subject to change on the USMLE are lettered A, B, C, D, and E. Although from exam to exam. The three dimensions are: (1) Sys- the format for this question type is straightforward, tem, (2) Process, and (3) Organizational Level. The the questions can be difficult because some of the application materials illustrate the percentage break- distractors may be partially right. The instructions out and offer you a detailed content outline to aid you will see for this type of question will generally you in your review. appear as below:

Question Formats DIRECTIONS (Question 1): Each of the numbered The style and presentation of the questions have been items or incomplete statements in this section is fully revised to conform with the United States Med- followed by answers or by completions of the state- ical Licensing Examinations. This will enable you to ment. Select the ONE lettered answer or comple- familiarize yourself with the types of questions to be tion that is BEST in each case. expected and provide practice in recalling your knowl- An example of this question type is: edge in each format. Following the answer to each question, a reference to a particular and easily avail- 1. An obese 21-year-old woman complains of in- able text is provided for further reference and reading. creased growth of coarse hair on her , chin,

v 0523-00 FM 07/15/02 15:30 Page vi

vi Introduction

chest, and abdomen. She also notes menstrual Answers, Explanations, and References irregularity with periods of amenorrhea. The In each of the sections of this book, the question sec- most likely cause is tions are followed by a section containing the answers, explanations, and references to the questions. This sec- (A) polycystic disease tion (1) tells you the answer to each question; (2) gives (B) an ovarian tumor you an explanation/review of why the answer is (C) an adrenal tumor correct and background information on the subject (D) Cushing’s disease matter; and (3) tells you where you can find more in- (E) familial hirsutism depth information on the subject matter in other books and/or journals. We encourage you to use this section In the question above, the key word is “most.” as a basis for further study and understanding. Although ovarian tumors, adrenal tumors, and If you choose the correct answer to a question, Cushing’s disease are causes of hirsutism (described you can then read the explanation (1) for reinforce- in the stem of the question), polycystic ovary disease ment and (2) to add to your knowledge about the sub- is a much more common cause. Familial hirsutism ject matter (remember that the explanations usually is not associated with the menstrual irregularities tell not only why the answer is correct, but also why mentioned. Thus, the most likely cause of the mani- the other choices are incorrect). If you choose the festations described can only be “(A) polycystic ovary wrong answer to a question, you can read the expla- disease.” nation for a learning/reviewing discussion of the material in the question. Furthermore, you can note the reference cited (e.g., “Joklik et al, pp 103–114”), look up the full source in the bibliography at the end of the section (e.g., “Joklik WK, Willett HP, Amos DB. Zinsser’s Microbiology. 20th ed. Norwalk, Conn: Apple- STRATEGIES FOR ANSWERING ONE BEST ton & Lange; 1992”), and refer to the pages cited for a ANSWER-SINGLE ITEM QUESTIONS more in-depth discussion. 1. Remember that only one choice can be the correct answer. 2. Read the question carefully to be sure that you understand what is being asked. Pay attention to key words like “most.” SPECIFIC INFORMATION ON 3. Quickly read each choice for familiarity. (This important step THE STEP 1 EXAMINATION is often not done by test takers.) 4. Go back and consider each choice individually. 5. If a choice is partially correct, tentatively consider it to be The official source of all information with respect to incorrect. (This step will help you eliminate choices and the United States Medical Licensing Examination increase your odds of choosing the correct answer.) Step 1 is the National Board of Medical Examiners 6. Consider the remaining choices and select the one you think (NBME), 3930 Chestnut Street, Philadelphia, PA is the answer. At this point, you may want to quickly scan the stem to be sure you understand the question and your 19104. Established in 1915, the NBME is a voluntary, answer. nonprofit, independent organization whose sole 7. If you do not know the answer, make an educated guess. function is the design, implementation, distribution, Your score is based on the number of correct answers, not and processing of a vast bank of question items, cer- the number you get incorrect. Do not leave any questions unanswered. tifying examinations, and evaluative services in the 8. The actual examination is timed for an average of 60 sec- professional medical field. onds per question. It is important to be thorough to under- Please contact the NBME or visit the USMLE stand the question, but it is equally important for you to web site (www.usmle.org) for information on exam keep moving. registration and scoring. 0523-01 C hap 1 07/15/02 15:31 Page 1

CHAPTER 1 The Back Questions

DIRECTIONS (Questions 1 through 40): Each of the (C) The transverse processes are long and numbered items or incomplete statements in this strong and extend posterolaterally. section is followed by answers or by completions of (D) The articular processes contain superior the statements. Select the ONE lettered answer or facets directed posteriorly and slightly completion that is BEST in each case. laterally. (E) The spinous processes are long and slope 1. The does all of the following posteroinferiorly. EXCEPT (A) protect the and spinal 4. Which of the following is a distinctive charac- (B) support the weight of the body teristic of a typical lumbar vertebra? (C) provide a pivot for the head (A) The body is massive and -shaped (D) play an important role in posture and when viewed superiorly. locomotion (B) The vertebral foramen is circular and (E) form the main part of the appendicular smaller than those of cervical and lumbar skeleton vertebrae. (C) The transverse processes are long and 2. Which of the following is NOT a distinctive slender and contain mammillary characteristic of a typical cervical vertebra? processes. (A) The body is small and wider from side to (D) The articular processes contain accessory side than anteroposteriorly. processes. (B) The vertebral foramen is large and (E) The spinous processes are long and slope triangular. posteroinferiorly. (C) The transverse processes contain trans- 5. Which of the following is true regarding the verse foramina. intervertebral disc between the C1 and C2 ver- (D) The articular processes contain superior tebrae? facets directed inferoanteriorly. (E) The spinous processes are short and bifid. (A) Its annulus fibrosus is composed of con- centric lamellae of fibrocartilage. 3. Which of the following is NOT a distinctive (B) It does not contain a nucleus pulposus as characteristic of a typical thoracic vertebra? other intervertebral discs do. (C) It is thicker than other intervertebral discs. (A) The body is -shaped. (D) It acts like a shock absorber in response to (B) The vertebral foramen is triangular and axial forces. larger than in cervical and . (E) There is no intervertebral disc between the C1 and C2 vertebrae.

1 0523-01 C hap 1 07/15/02 15:31 Page 2

2 1: The Back

6. All of the following are true regarding the pos- 11. The terior longitudinal ligament EXCEPT (A) flex the vertebral column (A) It is narrower and weaker than the ante- (B) flex the head rior longitudinal ligament. (C) control flexion of the back (B) It runs within the vertebral canal and con- (D) prevent lateral bending of the vertebral nects the vertebral bodies to each other. column (C) It is provided with pain endings. (E) assist with elevation (D) It helps prevent hyperextension of the vertebral column. 12. Which of the following is not a member of the (E) It is attached to the intervertebral discs and minor deep layer of the back? the posterior edges of the vertebral bodies. (A) levatores costarum 7. Spinal supplying the vertebrae are (B) cervical branches of the (C) (D) interspinales (A) vertebral and ascending cervical arteries in the neck (E) thoracic intertransversarii (B) posterior in the lumbar 13. The transversospinalis muscles do all of the fol- region lowing EXCEPT (C) subcostal and lumbar arteries in the pelvis (D) iliolumbar and lateral and medial sacral (A) extend the head arteries in the thorax (B) extend the thoracic and cervical regions of (E) aorta the vertebral column (C) elevate , assisting inspiration 8. Which of the following back muscles is inner- (D) stabilize vertebrae vated by dorsal rami? (E) assist with rotation of the vertebral column (A) latissimus dorsi 14. Which of the following muscles does NOT (B) levator scapulae attach to transverse processes of vertebrae? (C) rhomboid major (D) rhomboid minor (A) semispinalis (E) (B) multifidus (C) rotatores 9. Which of the following is NOT a deep (or intrin- (D) intertransversarii sic) muscle of the back? (E) spinalis (A) serratus posterior inferior 15. Which of the following muscles does NOT lat- (B) levatores costarum erally bend the cervical intervertebral ? (C) (D) multifidus (A) longus colli (E) splenius capitis (B) iliocostalis cervicis (C) longissimus capitis and cervicis 10. The splenius capitis and cervicis (D) splenius capitis (A) extend the head and neck (E) splenius cervicis (B) flex the head and neck 16. Which of the following muscles are likely to be (C) elevate ribs, assisting inspiration organs of proprioception instead of producers (D) stabilize the and axis of motion? (E) move the upper limb 0523-01 C hap 1 07/15/02 15:31 Page 3

Questions: 6–23 3

(A) iliocostalis (A) muscles of the (B) spinalis (B) skin over the neck and occipital (C) rotatores (innervated by ) (D) splenius capitis (C) intrinsic muscles of the back (E) longissimus (D) skin of the neck and scalp (innervated by lesser occipital nerve) 17. Which of the following statements is true regard- (E) skin of the central part of the back ing the suboccipital and deep neck muscles? 21. Which of the following statements is correct? (A) The rectus capitis posterior major arises from the spinous process of C2 and (A) The spinal cord is enlarged in the thoracic inserts into the lateral part of the inferior region for innervation of the upper limbs. nuchal line. (B) The spinal cord is enlarged in the lumbo- (B) The obliquus capitis inferior arises from sacral region for innervation of the lower the posterior tubercle of the posterior arch limbs. of C1 and inserts into the medial part of (C) In embryos, the spinal cord occupies only the inferior nuchal line. the superior two-thirds of the vertebral (C) The obliquus capitis superior arises from canal. the spinous process of C2 and inserts into (D) In adults, the spinal cord occupies the full the transverse process of C1. length of the vertebral canal. (D) The rectus capitis posterior minor arises (E) The cauda equina is composed of ventral from the transverse process of C1 and but not dorsal roots. inserts into the occipital bone between the nuchal lines. 22. Which of the following is NOT true in adults? (E) The are innervated (A) The tapering end of the spinal cord may by the ventral rami of C1. terminate as high as T12 or as low as L3. 18. ALL of the following extend the atlanto-occipital (B) The first cervical nerves lack dorsal roots EXCEPT in 50% of people. (C) The coccygeal nerve may be absent. (A) rectus capitis posterior major and minor (D) The terminal filum is the vestigial rem- (B) semispinalis capitis nant of the caudal part of the spinal cord (C) longus capitis that was in the tail of the embryo. (D) splenius capitis (E) The spinal cord has a lumbar enlargement (E) longissimus capitis for the lower limb but no equivalent enlargement for the smaller upper limb. 19. The suboccipital triangle is composed of 23. Which of the following is contained in the extra- (A) a superolateral and superomedial border dural (epidural) space? (the superior oblique and rectus capitis posterior major) (A) fat (loose connective tissue) (B) an inferolateral border (semispinalis (B) external vertebral venous plexus capitis) (C) CSF (C) a floor (C2) (D) denticulate ligaments (D) a roof () (E) radicular, medullary, and spinal arteries (E) greater and lesser occipital nerves

20. ALL of the following are innervated by dorsal rami EXCEPT 0523-01 C hap 1 07/15/02 15:31 Page 4

4 1: The Back

24. All of the following are contained in the sub- (D) sensory fibers from blood vessels and arachnoid (leptomeningeal) space EXCEPT glands (A) CSF (E) motor fibers to smooth muscle (B) arachnoid trabeculae 29. Which of the following is not true regarding the (C) segmental medullary arteries parasympathetic ? (D) spinal arteries (A) The cell body of the presynaptic (E) internal vertebral plexus neuron is located in the gray matter of the CNS. 25. Which of the following does NOT give rise to arteries supplying the spinal cord? (B) The cell body of the postsynaptic neuron is located in an autonomic ganglion out- (A) ascending cervical side the CNS. (B) (C) The postsynaptic neuron emits norepi- (C) intercostal arteries nephrine. (D) lumbar arteries (D) Its neurons are craniosacral in origin. (E) thoracoacromial artery (E) It promotes quiet and orderly processes of the body. 26. Which of the following is true? 30. Postsynaptic sympathetic fibers that ultimately (A) There are paired anterior spinal arteries. innervate the body wall and limbs do which of (B) There are paired posterior spinal arteries. the following? (C) The sulcal (central) arteries are formed by the union of branches of the vertebral (A) pass from the sympathetic trunks to adja- arteries. cent ventral rami through gray rami com- municantes (D) Each is a branch of either the posteroinferior (B) pass from the sympathetic trunks to adja- or the . cent ventral rami through white rami communicantes (E) There is usually one anterior and one pos- terior spinal (C) pass from the sympathetic trunks to adjacent dorsal rami through gray rami 27. Which of the following is true? communicantes (D) pass from the sympathetic trunks to (A) There are usually two anterior and two adjacent dorsal rami through white rami posterior spinal . communicantes (B) Veins of the spinal cord are distributed in (E) pass from the sympathetic trunks to a similar fashion to that of spinal arteries. splanchnic nerves (C) Spinal veins are unique in that they do not communicate with each other. 31. Postsynaptic sympathetic fibers do ALL of the (D) Spinal veins are arranged laterally. following EXCEPT (E) Spinal veins are drained by sulcal and (A) constrict the pupil of the eye meningeal vein. (B) stimulate contraction of blood vessels 28. All of the following are contained in typical (C) stimulate contraction of erector pili spinal nerves EXCEPT muscles (D) cause sudomotion (A) sensory fibers from and joints (E) cause goose bumps (B) motor fibers to muscles (C) parasympathetic fibers to glands 0523-01 C hap 1 07/15/02 15:31 Page 5

Questions: 24–38 5

32. Which of the following is NOT true? 36. Which of the following is NOT true in respect to caudal epidural anesthesia? (A) Variations in vertebrae are affected by race, sex, genetic factors, and environ- (A) A local anesthetic is injected into the mental factors. sacral hiatus or the posterior sacral foram- (B) An increased number of vertebrae occurs ina. more often in males, and a reduced num- (B) The anesthetic acts on S2-4 and the coc- ber occurs more often in females. cygeal nerves. (C) The number of can be 6, (C) The height to which the anesthetic travels 7, or 8. is primarily limited by the amount of fat (D) Some people have more than five lumbar in the epidural space. vertebrae and therefore fewer thoracic (D) Sensation is lost inferior to the epidural vertebrae. block. (E) The is typically composed of five (E) The sacral hiatus is located between the fused vertebrae. sacral cornua and inferior to the 4th sacral spinous process or median sacral crest. 33. Which of the following statements about kyphosis is true? 37. Which of the following is NOT a vertebral problem? (A) Kyphosis may result from developmental anomalies as well as from osteoporosis. (A) sacralization of L5 (B) The vertebral column curves anteriorly. (B) lumbarization of S1 (C) Kyphosis results in an increase in the lat- (C) lumbarization of T12 eral diameter of the thorax. (D) lumbar spinal stenosis (D) Women may develop a temporary kypho- (E) hemisacralization of L5 sis during pregnancy. (E) It is also known as “swayback” or 38. Which of the following statements is true? “hollow back.” (A) In spina bifida cystica, the laminae of L5 and possibly S1 do not fuse properly. 34. Lordosis is characterized by which of the fol- lowing? (B) In spina bifida occulta, one or more verte- bral arches do not develop, allowing (A) an abnormal increase in thoracic meninges and even the spinal cord to her- curvature niate. (B) an anterior rotation of the pelvis (C) Paralysis of the limbs and problems with (C) an abnormal lateral curvature bladder/bowel control may be associated (D) rotation of the vertebrae with meningomyelocele. (E) lateral curvature of the spine (D) Some cases of spina bifida result from an improper closure of the neural tube dur- 35. Scoliosis may be caused by which of the fol- ing the 8th week of embryonic develop- lowing? ment. (E) A meningocele is a spina bifida associated (A) asymmetrical weakness of intrinsic back with brain herniation. muscles (B) difference in length of the upper limbs (C) dehydrated intervertebral discs (D) ipsilateral weakness in gluteal muscles (E) sciatic nerve lesion 0523-01 C hap 1 07/15/02 15:31 Page 6

6 1: The Back

39. Which of the following are derivatives of the 40. Shortly after week four of development, dorsal epimere? primary rami begin to innervate which of the following? (A) erector spinae muscles (B) prevertebral muscles (A) ventral axial skeletal musculature (C) quadratus lumborum (B) vertebral joints (D) striated muscles of the anus (C) skin of the upper limb (E) sternalis (D) sweat glands of the lateral back region (E) erector pili muscles

DIRECTIONS (Questions 41 through 46): Identify the anatomical features indicated on the art below. 0523-01 C hap 1 7/17/02 3:25 PM Page 7

Answers and Explanations

1. (E) The vertebral column forms the main part zius, which is innervated by the accessory nerve, of the axial skeleton (Moore, p 432). are innervated by ventral rami (Moore, p 467).

2. (D) The articular processes contain superior 9. (A) The serratus posterior inferior is an inter- facets directed superoposteriorly (Moore, p 439). mediate extrinsic back muscle (Moore, p 467).

3. (B) The vertebral foramen is circular and smaller 10. (A) The splenius capitis and cervicis, when act- than the foramina of cervical and lumbar verte- ing together, extend the head and neck (Moore, brae (Moore, p 441). p 468).

4. (A) The body is massive and kidney-shaped 11. (C) The erector spinae muscles extend the ver- when viewed superiorly. The vertebral foramen tebral column and head when acting bilaterally, is triangular and larger than the foramina in tho- control flexion of the back by gradually length- racic vertebrae and smaller than those in cervi- ening their fibers, and bend the vertebral column cal vertebrae. The transverse processes are long laterally when acting unilaterally (Moore, p 470). and slender and contain accessory processes. The articular processes contain mammillary pro- 12. (C) The spinalis is a member of the erector spinae cesses. The spinous processes are short, thick, muscles, which comprise the intermediate mus- and broad (Moore, p 442). cle layer of the back (Moore, p 470).

5. (E) There is no intervertebral disc between the 13. (C) The transversospinalis muscles do not ele- C1 and C2 vertebrae (Moore, p 451). vate the ribs to assist inspiration (Moore, p 470).

6. (D) The posterior longitudinal ligament helps 14. (E) The spinalis inserts on spinous processes— prevent hyperflexion of the vertebral column not transverse processes. The semispinalis, multi- (Moore, p 451). fidus, rotatores, and intertransversarii all attach to transverse processes of vertebrae (Moore, p 470). 7. (A) Spinal arteries supplying the vertebrae are branches of the vertebral and ascending cervical 15. (A) The longus colli flexes the cervical inter- arteries in the neck, posterior intercostal arteries vertebral joints but does not bend them laterally in the thorax, subcostal and lumbar arteries in (Moore, p 473). the lumbar region, and iliolumbar and lateral and medial sacral arteries in the pelvis (Moore, 16. (C) Because of their small size and lack of p 467). mechanical advantage, it has been proposed that the rotatores are likely to be organs of pro- 8. (E) The longissimus, a deep or intrinsic back prioception instead of producers of motion muscle, is innervated by dorsal rami. All super- (Moore, p 474). ficial or extrinsic back muscles except the trape-

7 0523-01 C hap 1 07/15/02 15:31 Page 8

8 1: The Back

17. (A) The rectus capitis posterior major arises 24. (E) CSF, arachnoid trabeculae, segmental med- from the spinous processes of C2 and inserts into ullary arteries, radicular arteries, and spinal the lateral part of the inferior nuchal line. The arteries are located in the subarachnoid (lepto- rectus capitis posterior minor arises from the meningeal) space. The internal vertebral venous posterior tubercle of the posterior arch of C1 plexus is located in the extradural (epidural) and inserts into the medial part of the inferior space (Moore, p 480). nuchal line. The obliquus capitis inferior arises from the spinous processes of C2 and inserts 25. (E) The vertebral, ascending cervical, deep cer- into the transverse process of C1. The obliquus vical, intercostal, lumbar, and lateral sacral arter- capitis superior arises from the transverse pro- ies give rise to arteries supplying the spinal cord cess of C1 and inserts into the occipital bone (Moore, p 486). between the nuchal lines. The suboccipital mus- cles are innervated by the dorsal rami of C1 26. (B) There are paired posterior spinal arteries (Moore, pp 475–476). (Moore, p 486).

18. (C) The longus capitis flexes but does not extend 27. (B) Veins of the spinal cord are distributed in a the atlanto-occipital joint (Moore, p 476). similar fashion to that of spinal arteries (Moore, p 486). 19. (A) The suboccipital triangle is the deep trian- gular area between the rectus capitis posterior 28. (C) Typical spinal nerves do not contain para- major and the superior and inferior oblique sympathetic fibers (Moore, p 44–45). muscles. The boundaries and course of the suboccipital triangle include the rectus capitis 29. (C) The postsynaptic neurons of the para- posterior major, the superior oblique, and the sympathetic nervous system emit acetylcholine inferior oblique. The floor is formed by the (Moore, p 45). atlanto-occipital membrane and posterior arch of C1. The roof is formed by the semispinalis 30. (A) Postsynaptic sympathetic fibers that ulti- capitis. The suboccipital triangle contains the mately innervate the body wall and limbs pass vertebral artery and (Moore, from the sympathetic trunks to adjacent ventral pp 476–477). rami through gray rami communicantes (Moore, p 47). 20. (D) The lesser occipital nerve, which is com- posed of ventral rami of C2 and C3, innervates 31. (A) Postsynaptic sympathetic fibers dilate but the skin of the neck and scalp (Moore, p 477). do not constrict the pupil of the eye (Moore, p 47).

21. (B) The spinal cord is enlarged in the lumbo- 32. (C) The number of cervical vertebrae is con- sacral region for innervation of the lower limbs stant at seven (Moore, p 434). (Moore, p 477). 33. (A) Kyphosis (humpback or hunchback) may 22. (E) The spinal cord is enlarged in two regions result from developmental anomalies as well as for innervation of the limbs. The tapering end of from osteoporosis. It is characterized by an the spinal cord may terminate as high as T12 or abnormal increase in the thoracic curvature as low as L3. The first cervical nerves lack dorsal with the vertebrae curving posteriorly, result- roots in 50% of people. The coccygeal nerve may ing in an increase in the anteroposterior diame- be absent. The terminal filum is the vestigial ter of the thorax. Women may develop a remnant of the caudal part of the spinal cord that temporary lordosis—not kyphosis—during was in the tail of the embryo (Moore, pp 477–479). pregnancy (Moore, p 434).

23. Fat (loose connective tissue) is contained in the 34. (B) Lordosis is characterized by an abnormal extradural (epidural) space (Moore, p 480). rotation of the pelvis (Moore, p 434). 0523-01 C hap 1 07/15/02 15:31 Page 9

Answers and Explanations: 17–46 9

35. (A) Scoliosis may be caused by asymmetrical one or more vertebral arches do not develop. In weakness of intrinsic back muscles (myopathic spina bifida occulta, the laminae of L5 and pos- scoliosis), difference in length of the lower sibly S1 do not fuse properly. Some cases of spina limbs, failure of one half of a vertebra to bifida result from an improper closure of the develop, or occasionally habitual standing or neural tube during the 4th week of embryonic sitting in an improper position (habit scoliosis) development (Moore, pp 448–449). (Moore, p 435). 39. (A) Myoblasts of the epimeres form the extensor 36. (C) The height to which the anesthetic travels muscles of the vertebral column (Sadler, p 190). is primarily limited by the amount injected and by the position of the patient (Moore, p 435). 40. (A) Dorsal primary rami innervate dorsal axial musculature, vertebral joints, and the skin of 37. (C) Part or all of L5 may fuse with the sacrum the back (Sadler, p 421). (hemisacralization or sacralization). In addi- tion, S1 may separate from the sacrum and fuse 41. intertransverse ligament with L5. Lumbar stenosis occurs when an inter- vertebral disc bulges and narrows the vertebral 42. internal vertebral venous plexus canal in the lumbar region, compressing the roots. T12 is not known to fuse 43. dura mater with L1 (Moore, pp 446–447). 44. arachnoid layer 38. (C) Paralysis of the limbs and problems with bladder/bowel control may be present in severe 45. conus medullaris cases of meningomyelocele, which is associated with spina bifida cystica, a condition in which 46. dorsal root ganglion This page intentionally left blank 0523-02 C hap 2 07/15/02 15:31 Page 11

CHAPTER 2 The Upper Limb Questions

DIRECTIONS (Questions 1 through 83): Each of the (C) The acromion is superior to the glenoid numbered items or incomplete statements in this cavity and projects anterolaterally. section is followed by answers or by completions of (D) The is fastened securely to the the statement. Select the ONE lettered answer or thoracic cage at the scapulothoracic joint. completion that is BEST in each case. (E) The acromioclavicular joint represents the true joint. 1. Which of the following is NOT true regarding the ? 4. Which of the following is NOT included in the (A) Its medial end is enlarged where it condyle of the ? attaches to the . (A) radial, coronoid, and olecranon fossae (B) Its lateral end is flat where it articulates (B) epicondyles with the humerus. (C) trochlea (C) The medial two-thirds of the shaft are (D) capitulum convex anteriorly. (E) greater tubercle (D) The clavicle transmits shock from the upper limb to the axial skeleton. 5. Which of the following is NOT true in respect (E) The clavicle is a “long bone” that has no to the and ? medullary cavity. (A) The brachialis attaches to the tuberosity of 2. The trapezius attaches to which of the follow- the ulna. ing regions of the clavicle? (B) The ulnar styloid process is much larger than the radial styloid process and (A) lateral one-third of the clavicle extends farther distally. (B) conoid tubercle (C) The head of the ulna lies distally, whereas (C) subclavian groove the head of the radius articulates with the (D) trapezoid line humerus. (E) quadrangular tubercle (D) The ulna is medial to the radius in the anatomical position. 3. Which of the following is true in respect to the (E) The bodies of these are firmly bound scapula? together by the interosseous membrane. (A) The spine of the scapula continues later- ally as the coracoid process. (B) The lateral surface of the scapula forms the glenoid cavity.

11 0523-02 C hap 2 07/15/02 15:31 Page 12

12 2: The Upper Limb

6. Which of the following is true regarding the (D) subclavius carpus? (E) serratus anterior (A) The scaphoid articulates proximally with the ulna and has a tubercle. 11. Which of the following best describes the action of the ? (B) The lunate articulates with the ulna and is broader anteriorly than posteriorly. (A) stabilizes scapula by drawing it inferiorly (C) The triquetrum articulates proximally and anteriorly against with the articular disc of the distal (B) anchors and depresses clavicle radioulnar joint. (C) adducts and medially rotates humerus (D) The pisiform lies on the palmar surface of (D) rotates scapula the trapezium. (E) flexes humerus (E) It is composed of seven bones. 12. Which of the following muscles attaches to the 7. Which of the following describes the correct coracoid process of the scapula? order of the distal row of carpals from lateral to medial? (A) pectoralis minor (B) triceps brachii (A) triquetrum, trapezoid, capitate, hamate (C) brachialis (B) trapezoid, trapezium, capitate, hamate (D) (C) trapezium, trapezoid, capitate, hamate (E) subclavius (D) trapezium, triquetrum, capitate, hamate (E) scaphoid, lunate, triquetrum, pisiform 13. All of the following are medial rotators of the EXCEPT 8. Which of the following is actually a lateral cuta- neous branch of an intercostal nerve, innervat- (A) latissimus dorsi ing the skin of the medial surface of the arm? (B) teres major (C) subscapularis (A) (D) infraspinatus (B) superior lateral cutaneous nerve of the arm (E) anterior part of deltoid (C) inferior lateral cutaneous nerve of the arm (D) medial cutaneous nerve of the arm 14. What muscles are necessary to raise the arm (E) lateral pectoral nerve above the shoulder?

9. Which of the following is NOT a branch of the (A) first the supraspinatus, next the deltoid, radial nerve? and then the serratus anterior (B) first the deltoid, next the supraspinatus, (A) posterior cutaneous nerve of the arm and then the serratus anterior (B) posterior cutaneous nerve of the forearm (C) first the supraspinatus, next the serratus (C) inferior lateral cutaneous nerve of the arm anterior, and then the deltoid (D) superior lateral cutaneous nerve of the (D) first the serratus anterior, next the deltoid, arm and then the supraspinatus (E) posterior interosseous nerve (E) first the deltoid, next the serratus anterior, and then supraspinatus 10. Which of the following is NOT an anterior tho- racoappendicular muscle? 15. Which of the following is innervated by the (A) pectoralis major dorsal scapular nerve? (B) pectoralis minor (A) serratus anterior (C) deltoid (B) rhomboid major and minor 0523-02 C hap 2 07/15/02 15:31 Page 13

Questions: 6–23 13

(C) erector spinae (C) teres major (D) subscapularis (D) subscapularis (E) supraspinatus (E) levator scapulae

16. Which of the following is an extrinsic shoulder 21. Which of the following is NOT contained in the muscle? ? (A) deltoid (A) axillary blood vessels (B) teres major (B) lymph nodes (C) levator scapulae (C) trunks and divisions of the brachial (D) teres minor plexus (E) supraspinatus (D) axillary nerve (E) lymph nodes 17. Which of the following is true in respect to the trapezius? 22. Which of the following is most correct? (A) It is innervated by the dorsal scapular (A) The subscapular artery arises from the nerve. third part of the axillary artery and (B) Its superior fibers retract the scapula. contributes to blood supply of muscles (C) Its middle fibers elevate the scapula. near the scapula and humerus. (D) Its inferior fibers retract the scapula. (B) The second part of the axillary artery typi- cally contains two branches—the thora- (E) Its superior and inferior fibers act together coacromial artery and the superior in rotating the scapula on the thoracic thoracic artery. wall. (C) The first part of the axillary artery lies 18. A patient is asked to place the hands posteriorly posterior to the pectoralis minor. on the hips and to push the elbows posteriorly (D) The thoracoacromial artery supplies the against resistance. Which muscle is being tested? , axillary lymph nodes, and most importantly the lateral part of (A) levator scapulae the in women. (B) rhomboid (E) The lateral thoracic artery divides into (C) trapezius four branches, the acromial, deltoid, pec- (D) latissimus dorsi toral, and clavicular. (E) serratus anterior 23. Which of the following is NOT correct? 19. Which rotator cuff muscle does NOT rotate the (A) The is formed by the union humerus? of the ventral rami of C5 through T1. (A) supraspinatus (B) The roots of the brachial plexus and the (B) infraspinatus pass through the gap (C) teres minor between the anterior and middle scalene muscles. (D) subscapularis (C) Gray rami contribute sympathetic fibers (E) teres major to each root. 20. The axillary nerve innervates which of the fol- (D) Each of the three trunks of the brachial lowing muscles? plexus divide into anterior and posterior divisions. (A) coracobrachialis (E) The cords of the brachial plexus surround (B) teres minor the brachial artery. 0523-02 C hap 2 07/15/02 15:31 Page 14

14 2: The Upper Limb

24. Which of the following is NOT a supraclavicu- (D) It is primarily innervated by the musculo- lar branch of the brachial plexus? cutaneous nerve, but some of its lateral part is innervated by a branch of the (A) dorsal scapular nerve radial nerve. (B) lateral pectoral nerve (E) It crosses two joints. (C) long thoracic nerve (D) nerve to the subclavius 29. A patient is asked to abduct the arm 90 degrees (E) suprascapular nerve and then to extend the flexed forearm against resistance. Which muscle is being tested? 25. Which of the following is true regarding the (A) triceps brachii quadrangular space? (B) brachialis (A) It is bounded superiorly by the teres (C) coracobrachialis major. (D) biceps brachii (B) It is bounded inferiorly by the subscapu- (E) supinator laris and teres minor. (C) It is bounded medially by the humerus 30. The deep artery of the arm accompanies which and laterally by the long head of the of the following before passing around the triceps. body of the humerus? (D) It contains the posterior circumflex humeral artery and the axillary nerve. (A) radial nerve (E) Brachial plexus herniations occur here. (B) musculocutaneous nerve (C) median nerve 26. Which of the following is NOT innervated by (D) ulnar nerve the suprascapular nerve? (E) axillary nerve (A) supraspinatus 31. Which muscle assists in extension of the fore- (B) infraspinatus arm, resists abduction of the ulna during prona- (C) glenohumeral joint tion of the forearm, and tenses the capsule of (D) skin over superior part of scapula the elbow joint so that it is not pinched when (E) shoulder joint the joint is extended? (A) anconeus 27. Which of the following is NOT a branch of the posterior cord of the brachial plexus? (B) triceps brachii (C) coracobrachialis (A) upper and lower subscapular nerves (D) brachialis (B) thoracodorsal nerve (E) biceps brachii (C) axillary nerve (D) radial nerve 32. Which of the following is a branch of the brachial (E) long thoracic nerve artery? (A) anterior and posterior circumflex humeral 28. Which of the following is NOT true in respect arteries to the brachialis? (B) deltoid artery (A) Its origin is the distal half of the anterior (C) superior and inferior ulnar collateral surface of the humerus. arteries (B) Its insertion is the coronoid process and (D) thoracoacromial artery tuberosity of the ulna. (E) anterior and posterior ulnar recurrent (C) It flexes the forearm in all positions. arteries 0523-02 C hap 2 07/15/02 15:31 Page 15

Questions: 24–42 15

33. Which of the following nerves supply NO 38. Which muscle does NOT cross the elbow joint? branches to the arm? (A) flexor pollicis longus (A) musculocutaneous and median (B) pronator teres (B) radial and ulnar (C) flexor carpi radialis (C) median and ulnar (D) flexor carpi ulnaris (D) median and radial (E) flexor digitorum superficialis (E) musculocutaneous and radial 39. The ulnar nerve innervates which of the fol- 34. Which of the following nerves is correctly paired lowing muscles in the flexor compartment? with its cutaneous branch? (A) the medial part of the flexor digitorum (A) median nerve and medial antebrachial superficialis cutaneous nerve (B) flexor carpi radialis (B) musculocutaneous nerve and lateral ante- (C) pronator quadratus brachial cutaneous nerve (D) pronator teres (C) ulnar nerve and posterior antebrachial (E) the medial part of flexor digitorum cutaneous nerve profundus (D) median nerve and medial brachial cuta- neous nerve 40. The radial artery lies just lateral to the of (E) radial nerve and superior lateral brachial which muscle? cutaneous nerve (A) pronator teres 35. The does NOT contain which of the (B) flexor carpi radialis following? (C) palmaris longus (D) flexor carpi ulnaris (A) terminal part of the brachial artery (E) flexor digitorum superficialis (B) deep accompanying veins of the arteries (C) median nerve 41. The palmaris longus tendon is a useful guide to (D) biceps brachii tendon which nerve at the wrist? (E) ulnar nerve (A) anterior interosseous nerve 36. A patient is unable to flex the arm and forearm. (B) posterior interosseous nerve Where is the lesion likely to be? (C) median nerve (D) ulnar nerve (A) ventral rami of C3–C4 (E) radial nerve (B) ventral rami of C5–C6–C7 (C) dorsal rami of C6–C7–C8 42. To pronate the forearm, which of the following (D) ventral rami of C8–T1 must occur? (E) dorsal rami of T1 (A) The pronator quadratus initiates prona- tion, assisted later by the pronator teres. 37. The radial nerve innervates muscles in the exten- sor compartment of the forearm, but it also in- (B) The pronator teres initiates pronation, nervates the following flexor: assisted later by the pronator quadratus. (C) The anconeus initiates pronation, assisted (A) brachioradialis later by the pronator teres. (B) pronator teres (D) The pronator quadratus initiates prona- (C) palmaris longus tion, assisted later by the anconeus. (D) pronator quadratus (E) The ulnar nerve must be used. (E) palmaris longus 0523-02 C hap 2 07/15/02 15:31 Page 16

16 2: The Upper Limb

43. The extensor carpi radialis longus tendon is 47. Which of the following is true in respect to the crossed by which two muscles? anatomical snuff box? (A) abductor pollicis longus and extensor pol- (A) It is bounded anteriorly by the tendons of licis longus the extensor pollicis longus. (B) extensor indicis and extensor digitorum (B) It is bounded posteriorly by the tendons (C) extensor digitorum and extensor pollicis of the abductor pollicis longus and exten- brevis sor pollicis brevis. (D) abductor pollicis longus and extensor pol- (C) The radial artery lies in the floor of the licis brevis snuff box. (E) extensor indicis and extensor carpi radi- (D) The scaphoid and triquetrum can be pal- alis brevis pated within the snuff box. (E) The snuff box is visible when the thumb is 44. Which of the following is true in respect to the fully flexed. supinator? 48. Which of the following does NOT abduct the (A) It is innervated by the ulnar nerve. hand at the wrist joint? (B) It supinates the forearm by rotating the ulna. (A) flexor carpi radialis (C) It forms the floor of the cubital fossa along (B) extensor carpi radialis longus with the brachioradialis. (C) extensor carpi radialis brevis (D) It supinates the forearm when the forearm (D) abductor pollicis longus is already flexed. (E) palmaris longus (E) It rotates the radius to turn the palm ante- riorly. 49. Which of the following is derived from the radial artery? 45. Which of the following does NOT take an ori- (A) dorsal and palmar carpal arteries gin from the lateral epicondyle of the humerus? (B) common interosseous artery (A) extensor carpi radialis brevis (C) anterior interosseous artery (B) extensor carpi ulnaris (D) poster interosseous artery (C) abductor pollicis longus (E) ulnar recurrent artery (D) supinator (E) extensor digiti minimi 50. The median nerve does which of the following? (A) innervates the elbow joint with articular 46. Which of the following is correctly paired with branches its nerve? (B) innervates the medial half of the flexor (A) flexor pollicis longus and anterior digitorum profundus interosseous nerve (C) innervates the hypothenar muscles (B) flexor digitorum profundus and anterior (D) innervates lumbricals 3 and 4 interosseous nerve (E) innervates the skin of the dorsum of the (C) extensor carpi radialis longus and poste- hand rior interosseous nerve (D) brachioradialis and posterior interosseous 51. The ulnar nerve does NOT do which of the fol- nerve lowing? (E) abductor pollicis longus and anterior (A) innervate the elbow joint with articular interosseous nerve branches (B) innervate the flexor carpi ulnaris 0523-02 C hap 2 07/15/02 15:31 Page 17

Questions: 43–59 17

(C) innervate the skin on the lateral part of (B) adductor pollicis the palm and dorsum of the hand (C) flexor pollicis brevis (D) innervate the adductor pollicis (D) opponens pollicis (E) innervate the dorsal and palmar interossei (E) The recurrent branch of the median nerve innervates all of the above. 52. The radial nerve does NOT do which of the fol- lowing? 56. Which of the following muscles is correctly (A) give a superficial branch that innervates matched with the accompanying description? the dorsum of the hand (A) lumbricals 1 and 2 . . . bipennate (B) innervate the brachioradialis and extensor (B) lumbricals 3 and 4 . . . unipennate carpi radialis longus (C) dorsal interossei 1–4 . . . bipennate (C) give a deep branch that innervates the (D) palmar interossei 1–3 . . . bipennate extensor carpi radialis brevis and (E) deltoid . . . bipennate supinator (D) give a posterior interosseous branch that 57. The deep branch of the ulnar does NOT inner- innervates all remaining extensor muscles vate which of the following? in the posterior compartment of the fore- arm (A) abductor digiti minimi (E) innervate the glenohumeral joint (B) flexor digiti minimi brevis (C) lumbricals 1 and 2 53. Which of the following is NOT true in respect (D) dorsal interossei 3 and 4 to the flexor pollicis brevis? (E) palmar interossei 1 and 2 (A) It is located medial to the abductor polli- cis brevis. 58. The carpal tunnel does NOT contain which of (B) It flexes the thumb at the carpometacarpal the following? joint. (A) median nerve (C) It flexes the thumb at the metacarpopha- (B) four tendons of the flexor digitorum langeal joint. superficialis (D) Its tendon typically contains a sesamoid (C) four tendons of the flexor digitorum pro- bone. fundus (E) It is innervated by C5–C6. (D) the tendon of the flexor pollicis longus (E) ulnar nerve 54. Which of the following is true in respect to the palmaris brevis? 59. The sternoclavicular joint . . . (A) It aids the palmaris longus in tightening (A) . . . is a saddle-type but the palmar aponeurosis. functions as a ball-and-socket joint. (B) It is innervated by the median nerve. (B) . . . is supplied by lateral thoracic and (C) It is in the hypothenar compartment. thoracoacromial arteries. (D) It covers and protects the radial artery. (C) . . . is innervated by the lateral and medial (E) It wrinkles the skin of the hypothenar pectoral nerves. eminence and deepens the hollow of the (D) . . . is the articulation of the clavicle and palm. gladiolus of the sternum. (E) . . . dislocates easily. 55. The recurrent branch of the median nerve does NOT innervate which of the following? (A) abductor pollicis brevis 0523-02 C hap 2 07/15/02 15:31 Page 18

18 2: The Upper Limb

60. Which of the following is true in respect to the (A) proximal and distal radioulnar joints . . . acromioclavicular joint? condyloid type of synovial joint (A) It is a saddle-type synovial joint. (B) radiocarpal joint . . . pivot type of synovial joint (B) It is strengthened by the coracohumeral and transverse humeral ligaments. (C) intercarpal joints . . . plane type of syn- ovial joints (C) It is supplied by the lateral thoracic arteries. (D) metacarpophalangeal joints . . . hinge type of synovial joints (D) It is innervated by the nerve to the sub- clavius. (E) interphalangeal joints . . . condyloid type of synovial joints (E) When dislocated, it is often referred to as a “separated shoulder.” 65. All carpometacarpal and intermetacarpal joints are plane types of synovial joints EXCEPT for 61. Which of the following flexes the arm at the glenohumeral joint? (A) the carpometacarpal joint of the thumb. (A) deltoid (posterior part) (B) the carpometacarpal joint of the fifth metacarpal. (B) pectoralis major (C) the carpometacarpal joint of the third (C) latissimus dorsi metacarpal. (D) subscapularis (D) the intermetacarpal joint of the 4th and (E) infraspinatus 5th metacarpals. (E) the intermetacarpal joint of the 1st and 62. In respect to movement of the arm at the gleno- 2nd metacarpals. humeral joint, which of the following move- ments is correctly paired with its prime mover? 66. Which of the following is NOT true in respect (A) extension . . . deltoid (posterior part) to the clavicle? (B) abduction . . . pectoralis major and latis- (A) The clavicle varies more in shape than simus dorsi most other long bones. (C) adduction . . . deltoid (B) The clavicle can be pierced by a branch of (D) medial rotation . . . infraspinatus the supraclavicular nerve. (E) lateral rotation . . . subscapularis (C) The clavicle is thicker and more curved in manual workers. 63. Which of the following is true in respect to the (D) The right clavicle is stronger than the left elbow joint? and is usually shorter. (A) It is a plane type of synovial joint. (E) The clavicle is a compact bone. (B) It is strengthened by the radial and ulnar cruciate ligaments. 67. Fractures of the scapula typically involve (C) It is supplied by the cephalic and basilic (A) the acromion. arteries. (B) the coracoid process. (D) It is innervated by the median and axil- (C) the spine. lary nerves. (D) the inferior angle. (E) It is surrounded by the intratendinous ole- (E) the suprascapular notch. cranon bursa, the subtendinous olecranon bursa, and the subcutaneous olecranon 68. Which of the following parts of the humerus is bursa. matched correctly with the nerve with which it is in direct contact? 64. Which of the following joints is paired correctly with its type? 0523-02 C hap 2 07/15/02 15:31 Page 19

Questions: 60–76 19

(A) distal end of humerus . . . radial nerve (A) The teres major atrophies. (B) surgical neck . . . musculocutaneous nerve (B) The rounded contour of the shoulder dis- (C) radial groove . . . musculocutaneous nerve appears. (D) medial epicondyle . . . ulnar nerve (C) A loss of sensation may occur in the (E) scapular notch . . . suprascapular nerve lateral forearm. (D) The patient may lose the ability to adduct 69. “Winging” of the scapula is most likely caused the arm. by which of the following? (E) The patient may exhibit “wrist-drop.” (A) a lesion to the long thoracic nerve 74. Which of the following is true regarding rotator (B) a lesion to the thoracodorsal nerve cuff injuries? (C) injury to the suprascapular nerve (A) Injury or disease may damage the rotator (D) damage to the dorsal scapular nerve cuff, causing instability of the acromio- (E) damage to the upper and lower subscapu- clavicular joint. lar nerves (B) The supraspinatus tendon is the most commonly torn part of the rotator cuff. 70. Which of the following is correct regarding the triangle of ? (C) The teres major takes the longest to reha- bilitate of the rotator cuff muscles. (A) Its borders are the latissimus dorsi, (D) The injuries occur when the muscles pull scapula, and trapezius. away from their origin on the acromion. (B) It is a good location to hear heart murmurs. (E) Acute tears are common in young persons. (C) The 8th and 9th ribs and the 8th inter- costal space are subcutaneous here. 75. A patient has been thrown from a motorcycle, (D) It is a location of back trauma. landing on the shoulder such that the neck and (E) It is a location for dorsal rami to pass to shoulder are widely separated. You suspect an the superficial back. upper brachial plexus injury. What signs do you expect? 71. A patient cannot raise the trunk (as in climb- (A) “clawhand” ing). What is most likely the problem? (B) paralysis of flexor carpi ulnaris, flexor (A) damage to the ventral rami of C5–C6–C7 digitorum superficialis, and flexor digito- (B) paralysis of the latissimus dorsi rum profundus (C) injury to the dorsal scapular nerve (C) adducted shoulder, medially rotated arm, (D) damage to the dorsal rami of C8–T1 and extended elbow (E) injury to the axillary nerve (D) loss of sensation in the medial forearm (E) “wrist-drop” 72. The scapula on one side of a patient is located farther from the midline than that on the nor- 76. A patient exhibits “clawhand.” What might mal side. What might be the problem? have happened? (A) paralysis of the rhomboids on one side (A) upper brachial plexus injury (B) injury to the long thoracic nerve (B) acute brachial plexus neuritis (C) a lesion of C7–C8 (C) compression of the cords of the brachial (D) dislocated shoulder plexus (E) separated shoulder (D) lower brachial plexus injury (E) damage to dorsal rami that send fibers to 73. The axillary nerve is damaged. What is the the brachial plexus likely result? 0523-02 C hap 2 07/15/02 15:31 Page 20

20 2: The Upper Limb

77. A patient receives a knife wound to the axilla. (D) amelia . . . long bones are absent, and What problems do you expect? small hands or feet are attached to the trunk by short, irregular bones (A) damage to the axillary nerve (E) cleft hand (lobster claw deformity) . . . (B) paralysis of the coracobrachialis, biceps, absent third metacarpal, fusion of digits and brachialis 1–2 and 4–5 (C) inability to extend the wrist and digits at the metacarpophalangeal joints 81. Syndactylyl involves (D) loss of sensation on the medial surface of the arm (A) extra fingers or toes. (E) “clawhand” (B) absence of a digit or limb. (C) abnormal fusion of fingers and toes. 78. A patient tries to make a fist, but digits 2 and (D) small hands or feet being attached to trunk 3 remain partially extended. What nerve is by short bones instead of long bones. injured? (E) congenital dislocation of glenohumeral (A) ulnar nerve joint. (B) radial nerve 82. Which of the following is NOT correct? (C) median nerve (D) musculocutaneous nerve (A) During development, dorsal cells orga- nize as the epimere and ventral cells orga- (E) axillary nerve nize as the hypomere. 79. Which of the following is true in respect to ulnar (B) Dorsal rami innervate muscles derived nerve injuries? from the epimere. (C) Ventral rami innervate muscles derived (A) The injury often occurs where the nerve from the hypomere. passes posterior to the medial epicondyle (D) Myoblasts of the hypomere form the of the humerus. extensor muscles of the vertebral column. (B) The patient experiences numbness and (E) Somites and somitomeres form the mus- tingling on the lateral part of the palm culature of the limbs. and the thumb. (C) The patient may exhibit “waiter’s tip 83. A patient in surgery has no pectoralis major. hand.” What do you suspect? (D) Patients have difficulty because they can- not flex their first, second, and third digits (A) trauma at the DIP joints. (B) dominant pectoralis minor (E) Power of abduction is impaired, and (C) drug-induced muscle hypoplasia when the patient attempts to flex the (D) atrophy of the muscle wrist, the flexor carpi ulnaris brings the (E) congenital absence of the muscle hand to the medial side.

80. Which limb defect is correctly matched with its definition? (A) meromelia . . . complete absence of one or more extremities (B) phocomelia . . . all segments of extremities are present but abnormally short (C) micromelia . . . partial absence of one or more extremities 0523-02 C hap 2 07/15/02 15:31 Page 21

Questions: 77–93 21

DIRECTIONS (Questions 84 through 88): Identify the anatomical features indicated on the art below.

DIRECTIONS (Questions 89 through 93): Identify the anatomical features indicated on the art below. 0523-02 C hap 2 07/15/02 15:31 Page 22

22 2: The Upper Limb

DIRECTIONS (Questions 94 through 98): Identify the anatomical features indicated on the art below. 0523-02 C hap 2 07/15/02 15:31 Page 23

Answers and Explanations

1. (B) The lateral end of the clavicle is flat where 7. (C) From lateral to medial, the four bones of the it articulates with the acromion at the acromio- distal row of carpals are the trapezium, trape- clavicular (AC) joint (Moore, pp 665–666). zoid, capitate, and hamate (Moore, p 674).

2. (A) The trapezius attaches to the lateral third of 8. (A) The intercostobrachial nerve is the lateral the clavicle, acromion, and spine of the scapula. cutaneous branch of the second intercostal nerve The deltoid attaches to the deltoid tubercle, the from T2, innervating the skin of the medial sur- conoid ligament attaches to the conoid tubercle, face of the arm (Moore, p 684). the subclavius attaches to the subclavian groove, and the trapezoid ligament attaches to the trape- 9. (D) The posterior cutaneous nerve of the arm, zoid line (Moore, pp 666, 691). posterior cutaneous nerve of the forearm, and inferior lateral cutaneous nerve of the arm are 3. (B) The lateral surface of the scapula forms the branches of the radial nerve. The superior lateral glenoid cavity, superior to which the coracoid cutaneous nerve is a branch of the axillary nerve process projects anterolaterally. The glenohum- (Moore, p 684). eral joint itself represents the true shoulder joint, whereas the scapulothoracic joint, which is a con- 10. (C) The pectoralis major, pectoralis minor, sub- ceptual joint, is a location where the scapula clavius, and serratus anterior are anterior tho- moves easily on the thoracic wall. The spine of racoappendicular muscles. The deltoid is a the scapula continues laterally as the acromion scapulohumeral (shoulder) muscle (Moore, pp (Moore, pp 668–669). 688, 691).

4. (E) The condyle of the humerus (the distal end) 11. (A) The pectoralis minor stabilizes the scapula includes the epicondyles, trochlea, capitulum, by drawing it inferiorly and anteriorly against and the three fossae (radial, coronoid, and radial) the thoracic wall (Moore, p 688). (Moore, p 670). 12. (A) The pectoralis minor, biceps brachii (short 5. (B) The radial styloid process is much larger head), and coracobrachialis attach to the cora- than the ulnar styloid process and extends far- coid process of the scapula (Moore, p 688). ther distally (Moore, p 671–672). 13. (D) The latissimus dorsi, teres major, and sub- 6. (C) The scaphoid articulates proximally with scapularis medially rotate the arm. The infra- the radius and has a large tubercle. The lunate spinatus and teres minor rotate the arm laterally. articulates with the radius and is broader anteri- The deltoid is unique in that its anterior part orly than posteriorly. The triquetrum articulates rotates the arm medially, and its posterior part proximally with the articular disc of the distal rotates the arm laterally (Moore, p 691). radioulnar joint. The pisiform lies on the palmar surface of the triquetrum (Moore, p 674). 14. (A) The supraspinatus initiates abduction of the arm. The deltoid becomes fully effective as

23 0523-02 C hap 2 07/15/02 15:31 Page 24

24 2: The Upper Limb

an abductor following the initial 15 degrees of pass through the gap between the anterior and abduction. The serratus anterior rotates the middle scalene muscles. Gray rami contribute scapula, elevating its glenoid cavity so that the sympathetic fibers to each root. The roots of the arm can be raised above the shoulder (Moore, brachial plexus form three trunks, each of which pp 695, 696, 689). divides into anterior and posterior divisions. The cords of the brachial plexus surround the 15. (B) The dorsal scapular nerve innervates the axillary artery (Moore, p 708). levator scapulae, rhomboid major, and rhom- boid minor (Moore, p 691). 24. (B) The dorsal scapular nerve, long thoracic nerve, nerve to the subclavius, and supra- 16. (C) The trapezius, latissimus dorsi, levator scapular nerve are supraclavicular branches of scapulae, and rhomboids are extrinsic shoulder the brachial plexus, whereas the lateral pectoral muscles. The deltoid, teres major, supraspinatus, nerve is an infraclavicular branch, originating infraspinatus, teres minor, and subscapularis are from the lateral cord (Moore, pp 708–709). intrinsic shoulder muscles (Moore, pp 691–692). 25. (D) The quadrangular space is bounded superi- 17. (E) The trapezius, innervated by the spinal root orly by the subscapularis and teres minor, infe- of the accessory nerve (XI), is composed of three riorly by the teres major, medially by the long types of fibers. Its superior fibers elevate the head of triceps, and laterally by the humerus. It scapula, its middle fibers retract the scapula, and contains the axillary nerve and the posterior cir- its inferior fibers depress the scapula. Its superior cumflex humeral artery (Moore, p 711). and inferior fibers act together in rotating the scapula on the thoracic wall (Moore, p 694). 26. (D) The suprascapular nerve innervates the supraspinatus, infraspinatus, and glenohumeral 18. (B) To test the rhomboids, the patient is asked (shoulder) joint (Moore, p 710). to place the hands posteriorly on the hips and to push the elbows posteriorly against resistance 27. (E) The posterior cord gives rise to the upper and (Moore, p 695). lower subscapular nerves, thoracodorsal nerve, axillary nerve, and radial nerve. The long tho- 19. (A) The supraspinatus is the only rotator cuff racic nerve originates from C5–C6–C7 (Moore, muscle that does not rotate the humerus (Moore, pp 711). pp 697–698). 28. (E) The brachialis originates from the distal half 20. (B) The axillary nerve innervates both the del- of the anterior surface of the humerus and inserts toid and the teres minor (Moore, p 691). on the coronoid process and tuberosity of the ulna. It crosses one joint, flexing the forearm in 21. (C) The axilla contains axillary blood vessels, all positions. While it is primarily innervated by lymph nodes, the cords and branches of the the musculocutaneous nerve, some of its lateral brachial plexus, and the axillary nerve. The part is innervated by a branch of the radial nerve trunks and divisions are found superior to (Moore, p 722). the axilla in the neck (Moore, p 699). 29. (A) To test the triceps brachii, the arm is abducted 22. (A) The subscapular artery arises from the third 90 degrees and then the flexed forearm is extended part of the axillary artery and contributes to against resistance (Moore, p 724). blood supply of muscles near the scapula and humerus (Moore, p 701). 30. (A) The deep artery of the arm accompanies the radial nerve through the radial groove and 23. (E) The brachial plexus is formed by the union passes around the body of the humerus (Moore, of the ventral rami of C5 through T1. The roots p 728). of the brachial plexus and the subclavian artery 0523-02 C hap 2 07/15/02 15:31 Page 25

Answers and Explanations: 15–44 25

31. (A) The anconeus assists in extension of the fore- 36. (B) A patient who is unable to flex the arm and arm, resists abduction of the ulna during prona- forearm is likely to have a lesion in the ventral tion of the forearm, and tenses the capsule of the rami of C5, C6, and C7. The biceps brachii and elbow joint so that it is not pinched when the brachialis receive fibers from C5 and C6, and the joint is extended (Moore, p 724). coracobrachialis receives fibers from C5, C6, and C7. C6 is the main source of fibers for each 32. (C) The axillary artery gives rise to the superior (Moore, p 722). thoracic, thoracoacromial, lateral thoracic, sub- scapular, and anterior and posterior circumflex 37. (A) The brachioradialis is a flexor of the forearm, humeral arteries. The brachial artery gives rise to but it is located in the extensor compartment and the deep artery of the arm, the nutrient humeral is innervated by the radial nerve (Moore, p 734). artery, and the superior and inferior collateral arteries. The ulnar artery gives rise to the anterior 38. (A) The superficial muscles (pronator teres, and posterior ulnar recurrent, common inter- flexor carpi radialis, palmaris longus, flexor osseous, anterior and posterior interosseous, and carpi ulnaris, and flexor digitorum superficialis) dorsal and palmar carpal branch arteries (Moore, cross the elbow joint. The deep muscles (flexor pp 699, 727–728, 750). digitorum profundus, flexor pollicis longus, and pronator quadratus) do not (Moore, p 734). 33. (C) The median and ulnar nerves supply no branches to the arm (Moore, p 730). 39. (E) All muscles in the anterior compartment of the forearm are innervated by the median 34. (B) The median nerve gives rise to a palmar nerve, except for the flexor carpi ulnaris and the cutaneous branch; the ulnar nerve as well has a medial part of the flexor digitorum profundus, palmar cutaneous branch. The radial nerve gives which are innervated by the ulnar nerve (Moore, rise to the posterior brachial cutaneous nerve, pp 736–737). the posterior antebrachial cutaneous nerve, the inferior lateral brachial cutaneous nerve, and a 40. (B) The radial artery lies lateral to the tendon of superficial branch that innervates the dorsum of the flexor carpi radialis (Moore, p 737). the hand and the digits. The musculocutaneous nerve continues as the lateral antebrachial cuta- 41. (C) The palmaris longus tendon is a guide for neous nerve. The axillary nerve gives rise to the locating the median nerve at the wrist (Moore, superior lateral brachial cutaneous nerve. The p 737). medial cord of the brachial plexus gives rise to the medial brachial cutaneous nerve and the 42. (A) To pronate the forearm, the pronator quad- medial antebrachial cutaneous nerve. The supra- ratus initiates pronation, assisted later by the clavicular nerves (from C3–C4) and the inter- pronator teres (Moore, p 741). costobrachial nerve (from T2) also contribute to the cutaneous innervation of the arm (Moore, 43. (D) The extensor carpi radialis longus tendon pp 684, 758–759). is crossed by the abductor pollicis longus and extensor pollicis brevis (Moore, p 745). 35. (E) The cubital fossa contains the terminal part of the brachial artery (and the beginning of the 44. (E) The supinator, which forms the floor of the ulnar and radial arteries), deep accompanying cubital fossa along with the brachioradialis, is veins, the median nerve, and the biceps brachii innervated by the deep branch of the radial tendon. In the tissue superficial to the fossa are nerve. It supinates the forearm by rotating the the median cubital vein and medial and lateral radius. The biceps brachii also supinates the antebrachial cutaneous nerve. The deep and forearm when the forearm is already flexed superficial branches of the radial nerve are (Moore, p 746). within the floor of the fossa (Moore, pp 731–732). 0523-02 C hap 2 07/15/02 15:31 Page 26

26 2: The Upper Limb

45. (C) The abductor pollicis longus originates from ulnaris and medial half of the flexor digitorum the posterior surfaces of the ulna, radius, and profundus. The palmar cutaneous branch inner- interosseous membrane. The following muscles vates the skin of the medial part of the palm, and take at least one of their origins from the lateral the dorsal cutaneous branch innervates the pos- epicondyle of the humerus: extensor carpi radi- terior surface of the medial part of the hand and alis brevis, extensor digitorum, extensor digiti digits. The deep branch innervates the hypo- minimi, extensor carpi ulnaris, and supinator thenar muscles, adductor pollicis, interossei, and (Moore, p 742). the 3rd and 4th lumbricals (Moore, pp 759–760).

46. (A) The flexor pollicis longus is innervated by 52. (E) The radial nerve gives a superficial branch the anterior interosseous nerve from the median that innervates skin on the dorsum of the hand. nerve (Moore, pp 736–737, 742–743). The radial nerve itself innervates the brachiora- dialis and extensor carpi radialis longus. It then 47. (C) The snuff box is bounded anteriorly by the gives a deep branch that innervates the extensor tendons of the abductor pollicis longus and ex- carpi radialis brevis and the supinator before tensor pollicis brevis. Posteriorly it is bounded continuing as the posterior interosseous nerve, by the tendon of the extensor pollicis longus. The which innervates the extensor digitorum, ex- radial artery can be felt in the floor, along with tensor digiti minimi, extensor carpi ulnaris, ab- the radial styloid process, first metacarpal, ductor pollicis longus, extensor pollicis brevis, scaphoid, and trapezium (Moore, p 749). extensor pollicis longus, and extensor indicis (Moore, pp 742, 761). 48. (E) The hand is abducted at the wrist joint by the flexor carpi radialis, abductor pollicis longus, 53. (E) The flexor pollicis brevis is located medial to extensor carpi radialis longus, and extensor carpi the abductor pollicis brevis. It flexes the thumb radialis brevis (Moore, pp 736, 742). at the carpometacarpal and metacarpophalan- geal joints and assists in opposition. Its tendon 49. (A) The radial artery gives rise to the radial typically contains a sesamoid bone. It is inner- recurrent artery as well as dorsal and palmar vated by the recurrent branch of the median carpal branches. The ulnar artery gives rise to the nerve (C8–T1) (Moore, p 767). common interosseous artery, anterior and poste- rior interosseous arteries, anterior and posterior 54. (E) The palmaris brevis, innervated by the ulnar ulnar recurrent arteries, and dorsal and palmar nerve, wrinkles the skin of the hypothenar emi- carpal branches (Moore, p 750). nence and deepens the hollow of the palm, assist- ing the palmar grip. The muscle actually covers 50. (A) The median nerve assists in the innervation and protects the ulnar artery and the ulnar nerve, of the elbow joint and gives muscular branches which innervates it. The muscle is not by defi- to pronator teres, flexor carpi radialis, palmaris nition in the hypothenar compartment. The pal- longus, and flexor digitorum superficialis. The maris longus, on the other hand, flexes the hand median nerve also has an anterior interosseous at the wrist and tightens the palmar aponeuro- branch that innervates the lateral part of the sis (Moore, p 768). flexor digitorum profundus, flexor pollicis longus, and pronator quadratus. The recurrent 55. (B) The recurrent branch of the median nerve branch of the median nerve innervates the innervates the abductor pollicis brevis, flexor thenar muscles, and the palmar cutaneous pollicis brevis, and opponens pollicis, but the branch innervates the skin of the lateral part of deep branch of the ulnar nerve innervates adduc- the palm (Moore, pp 757–759). tor pollicis (Moore, pp 769–770).

51. (C) The ulnar nerve gives rise to articular 56. (C) Lumbricals 1–2 and palmar interossei 1–3 branches that innervate the elbow joint and mus- are unipennate muscles. Lumbricals 3–4 and dor- cular branches that innervate the flexor carpi sal interossei 1–4 are bipennate muscles. The del- toid is multipennate (Moore, p 770). 0523-02 C hap 2 07/15/02 15:31 Page 27

Answers and Explanations: 45–70 27

57. (C) The deep branch of the ulnar nerve inner- anastomosis around the elbow and is inner- vates the adductor pollicis, abductor digiti min- vated by the musculocutaneous, radial, and imi, flexor digiti minimi brevis, opponens digiti ulnar nerves. It is surrounded by the intratendi- minimi, lumbricals 3 and 4, dorsal interossei 1–4, nous olecranon bursa, the subtendinous olec- and palmar interossei 1–3. The median nerve ranon bursa, and the subcutaneous olecranon innervates lumbricals 1 and 2 (Moore, p 770). bursa (Moore, pp 795–798).

58. (E) The carpal tunnel contains the median nerve, 64. (C) The proximal and distal radioulnar joints the four tendons of the flexor digitorum superfi- are pivot-type synovial joints. The radiocarpal cialis, the four tendons of the flexor digitorum (wrist) joint is a condyloid type of synovial joint. profundus, and the tendon of the flexor pollicis Intercarpal joints are plane-type synovial joints. longus (Moore, p 774). Metacarpophalangeal joints are condyloid types of synovial joints. Interphalangeal joints are 59. (A) The sternoclavicular joint, which does not hinge-type synovial joints (Moore, pp 800, 803, dislocate easily, is a saddle-type synovial joint 807, 809). but functions as a ball-and-socket joint. It is the articulation of the sternal end of the clavicle with 65. (A) All carpometacarpal and intermetacarpal the manubrium of the sternum. The joint is sup- joints are the plane-type synovial joints except plied by the internal thoracic and suprascapular for the carpometacarpal joint of the thumb, arteries and is innervated by branches of the which is a saddle joint (Moore, p 809). medial supraclavicular nerve and the nerve to the subclavius (Moore, pp 781–782). 66. (E) The clavicle varies more in shape than most other long bones and is thicker and more curved 60. (E) The acromioclavicular joint is a plane-type in manual workers. The right clavicle is stronger synovial joint and is strengthened by the AC lig- than the left and is usually shorter. The clavicle ament and the coracoclavicular ligament, which can also be pierced by a branch of the supracla- is composed of the conoid and trapezoid liga- vicular nerve. The clavicle is a long bone with no ments. It is supplied by the suprascapular and medullary cavity. It consists of spongy (cancel- thoracoacromial arteries and is innervated by lous) bone with a shell of compact bone (Moore, the supraclavicular, lateral pectoral, and axillary p 667). nerves. When dislocated, it is referred to as a “separated shoulder” (Moore, pp 784, 787). 67. (A) Fractures of the scapula typically involve the protruding subcutaneous acromion. The remain- 61. (B) The pectoralis major (clavicular head) and der of the scapula is well protected by muscles deltoid (anterior part) flex the arm at the gleno- and the thoracic wall itself (Moore, p 669). humeral joint. The coracobrachialis and the bi- ceps brachii assist (Moore, p 792). 68. (D) The surgical neck of the humerus is in direct contact with the axillary nerve, the radial nerve 62. (A) The posterior portion of the deltoid causes runs in the radial groove, the distal end of the extension of the arm at the glenohumeral joint. humerus is in direct contact with the median The deltoid (as a whole, but especially the cen- nerve, and the medial epicondyle is in contact tral part) causes abduction, whereas the pec- with the ulnar nerve (Moore, p 670). toralis major and latissimus dorsi cause adduc- tion. The subscapularis causes medial rotation, 69. (A) Damage to the long thoracic nerve results whereas the infraspinatus causes lateral rota- in “winging” of the scapula (Moore, p 689). tion (Moore, p 792). 70. (A) The , a good place to 63. (E) The elbow is a hinge type of synovial joint, examine sounds, is bounded by the supe- strengthened by radial and ulnar collateral liga- rior horizontal border of the latissimus dorsi, the ments. It is supplied by arteries derived from the medial border of the scapula, and the inferolat- 0523-02 C hap 2 07/15/02 15:31 Page 28

28 2: The Upper Limb

eral border of the trapezius. The 6th and 7th ribs ened. The patient would also lose sensation on and the 6th intercostal space is subcutaneous the lateral surface of the forearm (Moore, p 731). (Moore, p 693). 78. (C) When the median nerve is injured, the pa- 71. (B) With paralysis of the latissimus dorsi, the tient often exhibits the “hand of benediction.” patient is unable to raise the trunk as necessary When the patient tries to make a fist, digits 2 and for climbing. The cause could be injury to the 3 remain partially extended because flexion of thoracodorsal nerve (C6–C7–C8) (Moore, p 693). the PIP joints is lost in digits 1–3 and weakened in digits 4–5. Flexion of the DIP joints is lost in 72. (A) Injury to the dorsal scapular nerve (C4–C5) digits 2–3 but maintained in digits 4–5 (since the can paralyze the rhomboids, causing the scapula ulnar nerve controls the medial part of the flexor on one side to be located farther from the midline digitorum profundus). Flexion of the MCP joints than that on the normal side (Moore, p 695). of digits 2-3 will also be affected due to a loss of the lumbricals 1 and 2 (Moore, pp 757, 774, 776). 73. (B) The deltoid atrophies when the axillary nerve (C5–C6) is damaged. Therefore, the 79. (A) The ulnar nerve is often injured where it rounded contour of the shoulder often dis- passes posterior to the medial epicondyle of the appears. A loss of sensation may occur on the humerus. The patient experiences loss of sensa- lateral side of the proximal part of the arm tion in the medial part of the palm as well as in 1 (Moore, pp 696–697). the medial 1 ⁄2 digits. Most intrinsic hand mus- cles are paralyzed, and the patient loses the 74. (B) Injury or disease may damage the rotator ability to adduct the hand at the wrist. Patients cuff, causing instability of the glenohumeral cannot make a fist since they are unable to flex the joint. The supraspinatus tendon is the most 4th and 5th digits at the DIP joints. The result- commonly torn part of the rotator cuff. Acute ing deformity is known as “clawhand” (Moore, tears are uncommon in young persons (Moore, pp 761, 776–777). pp 698–699). 80. (E) Amelia is the complete absence of one or 75. (C) In an upper brachial plexus injury causing more extremities while meromelia is the partial Erb-Duchenne palsy, one would expect damage absence of one or more extremities. All segments to C5–C6, resulting in “waiter’s tip position” of extremities are present but abnormally short (adducted shoulder, medially rotated arm, and in micromelia. In phocomelia, long bones are extended elbow). This results from paralysis of absent, and small hands or feet are attached to the deltoid, biceps, brachialis, and brachioradi- the trunk by short, irregular bones. In cleft hand alis. The lateral aspect of the upper limb also (lobster claw deformity), the third metacarpal is experiences loss of sensation (Moore, p 716). absent and digits 1–2 and 4–5 are fused (Sadler, p 179). 76. (D) Damage to the inferior trunks of the brachial plexus (C8–T1) affects the short muscles of the 81. (C) Syndactyly involves abnormal fusion of fin- hand, resulting in “clawhand.” The patient might gers and toes. Cleft hand (lobster claw defor- have grabbed a tree limb to catch himself while mity) consists of an abnormal cleft between the falling to cause this injury. “Clawhand” may also 2nd and 4th , with the 3rd be caused by an injury to the ulnar nerve (Moore, metacarpal and phalangeal bones being absent pp 716–717, 761). and with digits 1–2 and 4–5 being fused. Poly- dactyly involves extra fingers or toes, while 77. (B) A knife wound to the axilla would dam- ectrodactyly involves the absence of a digit. age the musculocutaneous nerve and result in Mutations in HOXA13 result in hand-foot-geni- paralysis of the coracobrachialis, biceps, and tal syndrome, where carpals and short digits are brachialis. Therefore, flexion of the elbow joint fused and the genitalia have altered structures and supination of the forearm would be weak- (Sadler, p 181). 0523-02 C hap 2 07/15/02 15:31 Page 29

Answers and Explanations: 71–98 29

82. (D) During development, dorsal cells organize 87. brachial artery as the epimere and ventral cells organize as the hypomere. Dorsal rami innervate muscles de- 88. median nerve rived from the epimere, whereas ventral rami innervate muscles derived from the hypomere. 89. lateral cord Myoblasts of the epimere form the extensor muscles of the vertebral column, and those of the 90. musculocutaneous nerve hypomere give rise to muscles of the limbs and body wall. Somites and somitomeres form the 91. medial antebrachial cutaneous nerve musculature of the limbs (Sadler, pp 189–190). 92. ulnar nerve 83. (E) Partial or complete absence of one or more muscles is rather common. One of the best- 93. lateral pectoral nerve known examples is total or partial absence of the pectoralis major (Poland anomaly). Similarly, 94. coracobrachialis the palmaris longus, serratus anterior, and quad- ratus femoris may be partially or entirely absent 95. brachialis (Sadler, p 192). 96. ulna 84. biceps brachii 97. long head of triceps brachii 85. cephalic vein 98. lateral head of triceps brachii 86. radial nerve This page intentionally left blank 0523-03 C hap 3 07/15/02 15:32 Page 31

CHAPTER 3 The Thorax Questions

DIRECTIONS (Questions 1 through 75): Each of the (C) venous drainage numbered items or incomplete statements in this (D) poor imaging techniques section is followed by answers or by completions of (E) complex lymphatic drainage the statement. Select the ONE lettered answer or completion that is BEST in each case. 5. Which of the following statements correctly apply to the internal thoracic artery? 1. The articular part of a tubercle articulates with which of the following structures? (A) It runs posterior to the transversus tho- racis muscle. (A) body of the vertebra (B) It ends in the 6th intercostal space. (B) costal (C) It divides into the superior and inferior (C) adjacent rib epigastric arteries. (D) sternum (D) It runs posterior to the superior six ribs. (E) transverse process (E) It gives rise to the posterior intercostal arteries. 2. Where is the groove for the subclavian artery located? 6. Which of the following statements concerning (A) posterior to the scalene tubercle the sternal angle is correct? (B) on the clavicle (A) It lies at the level of the intervertebral disk (C) on the manubrium between T4/T5. (D) at the sternal angle (B) It is flanked by the costal cartilage of the (E) at the angle of the 1st rib 3rd pair of costal . (C) It is crossed by the superior epigastric 3. The retromammary space is located between artery. which of the following structures? (D) It lies in the epigastric fossa. (A) skin and the areola (E) It is located 3 mm superior to the jugular (B) pectoralis major and minor notch. (C) and deep pectoral 7. The pleural cavity contains which of the fol- (D) suspensory ligament and the skin lowing? (E) lactiferous sinus and the nipple (A) 4. The high death rate associated with breast can- (B) bronchi cer is related to which of the following? (C) serous pleural fluid (A) nerve supply (D) lymph nodes (B) blood supply (E) pulmonary arteries and veins

31 0523-03 C hap 3 07/15/02 15:32 Page 32

32 3: The Thorax

8. The parietal pleura consists of all of the follow- (D) heart ing parts EXCEPT (E) azygos vein (A) costal 13. All of the following statements correctly apply (B) pericardial to the right atrium EXCEPT (C) mediastinal (D) diaphragmatic (A) It receives blood from the superior and inferior vena cava and coronary sinus. (E) cervical (B) It forms the right side of the heart. 9. All of the following statements correctly apply (C) It contains the crista terminalis. to the right lung EXCEPT (D) It contains the limbus fossae ovalis. (A) The superior and oblique fissures divide it (E) It contains trabeculae carneae. into three lobes. 14. The interventricular septum contains which of (B) It is larger and heavier than the left lung. the following structures? (C) It is shorter and wider than the left lung. (D) It contains a thin, tonguelike process (A) anterior papillary muscle called the lingula. (B) fossa ovalis (E) It has three surfaces. (C) sinus venarum (D) sinoatrial node 10. All of the following statements correctly apply (E) conus arteriosus to the left main EXCEPT (A) It is wider, shorter, and runs more verti- 15. All of the following statements correctly apply cally than the right main bronchus. to the right coronary artery EXCEPT (B) It passes anterior to the . (A) Typically supplies the SA node in approx- (C) It passes anterior to the thoracic aorta. imately 60% of people. (D) It contains c-shaped rings of hyaline (B) Typically supplies the AV node in cartilage. approximately 80% of people. (E) It arises at the level of the sternal angle. (C) Arises from the arch of the aorta. (D) Runs in the coronary sulcus. 11. All of the following statements correctly apply (E) Dominance is typical. to a bronchopulmonary segment EXCEPT (A) It is separated from adjacent segments by 16. Which of the following is the basic structural connective tissue septa. unit for gas exchange in the lung? (B) It is the largest subdivision of a lobe. (A) terminal bronchioles (C) It is not resectable. (B) respiratory bronchioles (D) It is named according to the segmental (C) alveolar ducts bronchus supplying it. (D) alveoli (E) It is a pyramid-shaped segment of the (E) bronchi lung, with its apex facing the lung root and its base at the pleural surface. 17. Which of the following structures carry highly oxygenated blood from the lungs to the heart? 12. Which of the following structures is located in the middle mediastinum? (A) pulmonary arteries (B) pulmonary veins (A) thoracic duct (C) coronary arteries (B) lungs (D) cardiac veins (C) esophagus (E) ascending aorta 0523-03 C hap 3 07/15/02 15:32 Page 33

Questions: 8–26 33

18. Which of the following structures is located in 22. All of the following veins drain into the coro- the posterior mediastinum? nary sinus EXCEPT (A) lungs (A) anterior cardiac (B) heart (B) small cardiac (C) azygos vein (C) middle cardiac (D) superior vena cava (D) great cardiac (E) right coronary artery (E) oblique vein of the left atrium

19. All of the following statements concerning spinal 23. All of the following statements concerning the nerves are correct EXCEPT sinoatrial node are correct EXCEPT (A) The dorsal and ventral rami are both (A) It is located near the superior end of the motor and sensory. sulcus terminalis. (B) The cutaneous branches include anterior, (B) It is located at the junction of the superior lateral, and posterior branches. vena cava and the right atrium. (C) The dorsal root is both sensory and motor. (C) It is known as the pacemaker of the heart. (D) The ventral root is pure motor. (D) It is specialized cardiac muscle fiber. (E) They supply a bandlike skin area known (E) It is avascular. as a dermatome. 24. Which of the following structures is located in 20. Which of the following statements correctly the left ventricle? applies to the tricuspid valve? (A) fossa ovalis (A) It guards the left atrioventricular orifice. (B) crista terminalis (B) It guards the conus arteriosus. (C) opening of the coronary sinus (C) It is also known as the mitral valve. (D) conus arteriosus (D) Chordae tendineae attach to the free (E) posterior papillary muscle edges of the cusps. (E) The apex of each cusp attaches to the 25. Which of the following structures is located in fibrous ring around the orifice. the left atrium? (A) pectinate muscle 21. All of the following statements concerning the pericardium are correct EXCEPT (B) pulmonary semilunar valves (C) septomarginal trabeculae (A) The external layer of the sac is fibrous. (D) septal papillary muscle (B) The internal layer is reflected onto the (E) atrioventricular node heart as the visceral serous layer (epi- cardium). 26. Which of the following is NOT part of the chest? (C) The internal layer of the fibrous sac is the partietal serous layer. (A) 12 pairs of ribs (D) The pericardial cavity is the potential (B) sternum space between the parietal and visceral (C) costal cartilages serous layers. (D) 12 (E) The fibrous pericardium is attached to (E) clavicle the sternum by the pericardiacophrenic ligament. 0523-03 C hap 3 07/15/02 15:32 Page 34

34 3: The Thorax

27. Which of the following is NOT likely to con- (C) internal intercostals tribute to chest pain? (D) subcostals (A) cardiac disease (E) serratus posterior inferior (B) pulmonary disease 33. Which of the following do NOT elevate the (C) disorders ribs? (D) gallbladder disorders (E) intestinal disorders (A) serratus posterior superior (B) serratus posterior inferior 28. Which of the following is a special feature of a (C) external intercostals thoracic vertebra? (D) levatores costarum (A) foramen for vertebral artery (E) subcostals (B) dens for rotation 34. With which of the following does the intercos- (C) short spinous processes tobrachial nerve communicate? (D) costal facets on bodies (E) costal facets on spinous processes (A) medial brachial cutaneous nerve (B) thoracodorsal nerve 29. Which of the following is a saddle-type synovial (C) long thoracic nerve joint? (D) first intercostal nerve (A) sternoclavicular joint (E) (B) manubriosternal joint 35. The musculophrenic arteries give rise to which (C) interchondral joint of the following? (D) intervertebral joint (E) costochondral joint (A) anterior intercostal arteries for intercostal spaces 7–9 30. The mammary glands are modified versions of (B) posterior intercostals arteries for inter- which type of gland? costal spaces 3–11 (C) subcostal artery (A) sebaceous gland (D) inferior phrenic artery (B) lymph gland (E) lumbar arteries (C) sweat gland (D) tonsillar tissue 36. Anterior and posterior intercostals arteries pass (E) endocrine gland between which two layers?

31. Which of the following does NOT supply the (A) skin and external breast with blood? (B) external and internal intercostal muscles (C) internal and innermost intercostals (A) lateral thoracic artery muscles (B) thoracoacromial artery (D) innermost intercostal muscles and trans- (C) posterior intercostals arteries versalis fascia (D) internal thoracic artery (E) transversalis fascia and peritoneum (E) costoclavicular artery 37. Of the bronchopulmonary segments in the supe- 32. Which of the following are innervated by dor- rior lobe of the left lung, which two are com- sal rami? bined to form the lingula? (A) levatores costarum (A) apical and posterior (B) external intercostals (B) posterior and anterior 0523-03 C hap 3 07/15/02 15:32 Page 35

Questions: 27–46 35

(C) anterior and superior 42. Which of the following is NOT a branch of the (D) superior and inferior right coronary artery? (E) apical and inferior (A) SA nodal (B) right marginal 38. Which of the following areas do NOT receive (C) posterior interventricular visceral afferent fibers? (D) AV nodal (A) bronchial mucosa (E) circumflex (B) bronchial muscles (C) interalveolar connective tissue 43. Which of the following is NOT contained in the (D) pulmonary arteries and veins superior mediastinum? (E) diaphragm (A) (B) great vessels and their branches 39. Sympathetic nerves do NOT contribute to which (C) azygos and hemiazygos veins of the following? (D) (A) diaphragm (E) esophagus (B) bronchial muscle (C) pulmonary vessels 44. The thymus is supplied by which of the fol- (D) alveolar glands of the bronchial tree lowing? (E) sweat glands of the chest (A) anterior intercostal branches of the inter- nal thoracic artery 40. Which of the following is correct? (B) brachiocephalic artery (A) While a person is supine, the arch of the (C) posterior intercostal arteries aorta lies superior to the transverse tho- (D) left racic plane. (E) pericardiacophrenic arteries (B) While a person is supine, the bifurcation of the trachea lies at the level of the 45. Which of the following is NOT a branch of the xiphisternal junction and T9. aorta in the thorax? (C) While a person is supine, the central ten- (A) posterior intercostal arteries don of the diaphragm is transected by the transverse thoracic plane. (B) inferior phrenic artery (D) While a person is standing, the arch of the (C) bronchial arteries aorta lies inferior to the transverse tho- (D) esophageal arteries racic plane. (E) superior phrenic arteries (E) While a person is standing, the tracheal bifurcation is transected by the transverse 46. The posterior mediastinum does NOT include thoracic plane. which of the following? (A) prevertebral muscles 41. The pericardium does NOT receive blood sup- (B) thoracic aorta ply from which of the following? (C) thoracic duct (A) pericardiacophrenic artery (D) thoracic sympathetic trunks (B) musculophrenic artery (E) thoracic splanchnic nerves (C) bronchial arteries (D) esophageal arteries (E) pectoral arteries 0523-03 C hap 3 07/15/02 15:32 Page 36

36 3: The Thorax

47. Which of the following may compress the (C) ganglion impar esophagus? (D) superior cervical ganglia (A) aortic arch (E) inferior cervical ganglia (B) right main bronchus 53. Which of the following does NOT contain syn- (C) left vagus apses for the sympathetic nervous system? (D) right brachiocephalic vein (E) brachiocephalic trunk (A) prevertebral ganglia (B) collateral ganglia 48. The azygos vein receives blood from which of (C) aortic plexus the following? (D) celiac ganglia (A) posterior intercostal veins (E) cardiac plexus (B) anterior intercostal veins 54. Which of the following does NOT occur in the (C) left sympathetic trunk? (D) anterior jugular vein (E) lateral thoracic vein (A) Presynaptic neurons synapse with post- synaptic neurons immediately. 49. The hemiazygos vein does NOT receive blood (B) Fibers ascend and synapse. from which of the following? (C) Fibers descend and synapse. (A) left subcostal vein (D) Presynaptic fibers innervate immediately surrounding blood vessels. (B) ascending lumbar veins (E) Fibers pass, without synapsing, into a (C) inferior three posterior intercostal veins splanchnic nerve. (D) superior phrenic veins (E) small mediastinal veins 55. Parasympathetic fibers stimulate secretion by all glands except which of the following? 50. The accessory hemiazygos vein parallels the vertebral column along which vertebral levels? (A) sweat glands (B) lacrimal glands (A) T1–4 (C) salivary glands (B) T2–6 (D) nasal glands (C) T5–8 (E) palatine glands (D) T8–12 (E) L1–4 56. Vasoconstriction is sympathetically stimulated with the exception of which arteries? 51. Which of the following is NOT contained in the anterior mediastinum? (A) bronchial arteries (B) coronary arteries (A) fat (C) esophageal arteries (B) sternopericardial ligaments (D) adrenal arteries (C) branches of internal thoracic vessels (E) renal arteries (D) lymphatic vessels (E) heart 57. Which of the following is a result of sympathetic stimulation in the heart? 52. Which of the following is NOT part of the sym- pathetic trunks? (A) decrease in the rate and strength of con- traction (A) prevertebral ganglia (B) paravertebral ganglia 0523-03 C hap 3 07/15/02 15:32 Page 37

Questions: 47–66 37

(B) inhibition of the effect of parasympathetic (A) cervical splanchnic nerves system on coronary arteries, allowing (B) upper thoracic splanchnic nerves them to dilate (C) lower thoracic splanchnic nerves (C) production of atrial natriuretic factor (D) lumbar splanchnic nerves (D) opening and closing of mitral valve (E) pelvic splanchnic nerves (E) glandular secretion 63. The greater splanchnic nerve originates from 58. Which of the following is NOT a result of sym- which vertebral levels? pathetic stimulation in the lungs? (A) T1–4 (A) inhibition of parasympathetic system (B) T5–9 (B) bronchodilation (C) T10–11 (C) reduced secretion (D) T12 (D) maximum air exchange (E) L1–4 (E) surfactant production 64. The fibers of the greater splanchnic nerve syn- 59. Sympathetic fibers of the heart are accompanied apse in which ganglion? by visceral afferent fibers that convey which type of sensation? (A) celiac ganglion (B) inferior mesenteric ganglion (A) reflex (C) paravertebral ganglia (B) pain (D) pulmonary plexus (C) rate information (E) adrenal cortex (D) contraction feedback (E) pressure sensation 65. The level of the domes of the diaphragm varies according to various situations. Which of the fol- 60. The pulmonary plexus contains which of the lowing does NOT affect the level of the domes of following? the diaphragm? (A) cell bodies of visceral afferent fibers (A) phase of respiration (B) cell bodies of postsynaptic sympathetic (B) posture fibers (C) size of distention of abdominal viscera (C) cell bodies of postsynaptic parasympa- (D) degree of distention of abdominal viscera thetic fibers (E) heart contractions (D) somatic efferent fibers passing on to diaphragm 66. Which of the following passes through the (E) somatic afferent fibers from diaphragm caval opening of the diaphragm?

61. Visceral afferent fibers of the tenth cranial nerve (A) terminal branches of the right phrenic are NOT distributed to which of the following? nerve (B) thoracic duct (A) bronchial mucosa (C) greater thoracic splanchnic nerve (B) bronchial muscles (D) sympathetic trunk (C) interalveolar connective tissue (E) vagus (D) pulmonary arteries and veins (E) diaphragm

62. The greater, lesser, and least splanchnic nerves are examples of what type of splanchnic nerves? 0523-03 C hap 3 07/15/02 15:32 Page 38

38 3: The Thorax

67. Which of the following does NOT pass through (C) mesoderm the esophageal hiatus of the diaphragm? (D) ectoderm (A) esophagus (E) notochord invagination (B) branches of the left gastric vessels 73. Which of the following is NOT derived from (C) lymphatic vessels the original aortic arch system? (D) vagal trunks (E) lesser thoracic splanchnic nerve (A) carotid arteries (B) arch of the aorta 68. Which of the following passes through the aortic (C) pulmonary artery hiatus of the diaphragm? (D) right subclavian artery (A) least thoracic splanchnic nerve (E) coronary arteries (B) thoracic duct 74. The is an outgrowth of what? (C) branches of right gastric vessels (D) sympathetic trunks (A) middle mediastinum (E) terminal branches of the left phrenic nerve (B) ventral wall of foregut (C) anterior abdominal wall 69. Ribs are formed from which of the following? (D) aortic arches (A) sclerotome portion of paraxial mesoderm (E) pharyngeal arches (B) lateral plate mesoderm 75. Which of the following correctly describes the (C) ectodermal invagination development of the lungs, in order? (D) endodermal migration (E) neural crest cell transitory development (A) pseudoglandular period, canalicular period, terminal sac period, alveolar 70. The pleuropericardial membranes develop into period what structures of the adult? (B) canalicular period, pseudoglandular period, terminal sac period, alveolar (A) fibrous pericardium period (B) diaphragm (C) alveolar period, pseudoglandular period, (C) parietal pleura canalicular period, terminal sac period (D) visceral pleura (D) pseudoglandular period, terminal sac (E) the membranes degenerate completely period, alveolar period, canalicular period (E) terminal sac period, alveolar period, 71. The diaphragm is NOT derived from which of pseudoglandular period, canalicular the following? period (A) septum transversum (B) pleuroperitoneal membranes (C) muscular components from lateral and dorsal body walls (D) mesentery of esophagus (E) anterior thoracic fascia

72. The entire cardiovascular system is derived from which of the following? (A) neural crest cells (B) endoderm 0523-03 C hap 3 07/15/02 15:32 Page 39

DIRECTIONS (Questions 76 through 85): Identify the anatomical features indicated on the art below.

DIRECTIONS (Questions 86 through 90): Identify the anatomical features indicated on the art below. 0523-03 C hap 3 07/15/02 15:32 Page 40

40 3: The Thorax

DIRECTIONS (Questions 91 through 95): Identify the anatomical features indicated on the art below. 0523-03 C hap 3 07/15/02 15:32 Page 41

Answers and Explanations

1. (E) The tubercle has a smooth articular part for intervertebral disc between T4 and T5 vertebrae articulating with the corresponding transverse (Moore, p 66). process of the vertebra and a rough nonarticu- lar part for attachment of the costotransverse 7. (C) The pleural cavity, the potential space be- ligament (Moore, p 63). tween the layers of pleura, contains a capillary layer of serous pleural fluid. It provides the lubri- 2. (A) The surface of the 1st rib has two trans- cation and cohesion that keep the lung surface in versely directed shallow grooves, anterior and contact with the thoracic wall (Moore, p 95). posterior to the scalene tubercle, for the subcla- vian vein and the subclavian artery respectively 8. (B) The parietal pleura includes costal, mediasti- (Moore, pp 63–64). nal, diaphragmatic, and cervical parts (Moore, pp 95–96). 3. (C) Between the breast and deep pectoral fas- cia is a loose connective tissue plane or poten- 9. (D) The right lung has three lobes, the left two. tial space known as the retromammary space The right lung is larger and heavier than the left, (Moore, p 73). but it is shorter and wider, because the right dome of the diaphragm is higher and the heart 4. (E) Because the axillary lymph nodes are the and pericardium bulge more to the left. The ante- most common site of metastases from a breast rior margin of the right lung is relatively straight, cancer, enlargement of the palpable nodes in a whereas the margin of the left lung has a deep woman suggests the possibility of breast cancer cardiac notch. The cardiac notch primarily in- and may be key to early detection. However, the dents the anteroinferior aspect of the superior absence of enlarged axillary nodes is no guaran- lobe of the left lung. This often creates a thin, tee that metastasis from a breast cancer has not -like process of the superior lobe, called occurred, because the malignant cells may have the lingula (Moore, p 101). passed to other nodes, such as the infraclavicu- lar and supraclavicular lymph nodes (Moore, 10. (A) The right main bronchus is wider and p 78). shorter, running more vertically than the left main bronchus as it passes directly to the hilum 5. (B) After descending past the 2nd costal carti- of the lung. The left main bronchus passes infer- lage, the internal thoracic artery runs anterior to olaterally, inferior to the arch of the aorta and the transversus thoracis muscle. It ends in the anterior to the esophagus and thoracic aorta, to 6th intercostal space, where it divides into the reach the hilum of the lung (Moore, p 104). superior epigastric and musculophrenic arteries (Moore, p 91). 11. (C) A bronchopulmonary segment is a pyramid- shaped segment of the lung with its apex facing 6. (A) The sternal angle is located opposite the the lung root and its base at the pleural surface. 2nd pair of costal cartilages at the level of the 4th It is the largest subdivision of a lobe and is sepa-

41 0523-03 C hap 3 07/15/02 15:32 Page 42

42 3: The Thorax

rated from adjacent segments by connective tis- 20. (D) The tricuspid valve guards the right AV sue. It is surgically resectable (Moore, p 104). orifice. The bases of the valve cusps are attached to the fibrous ring around the orifice. Chordae 12. (D) The middle mediastinum contains the heart tendineae attach to the free edges and ventricu- (Moore, p 114). lar surfaces of the anterior, posterior, and septal cusps (Moore, p 127). 13. (E) The right border of the heart is formed by the right atrium; it receives venous blood from 21. (E) The fibrous pericardium is attached to the the SVC, IVC, and coronary sinus. It contains posterior surface of the sternum by the sterno- both the crista terminalis and the limbus fossae pericardial ligaments (Moore, p 116). ovalis. The interior of the ventricles contains irregular muscular elevations called trabeculae 22. (A) The coronary sinus receives the great and carneae (Moore, pp 125–127). small cardiac veins, middle cardiac vein, left pos- terior ventricular vein, and left marginal vein. 14. (E) The interventricular septum is composed The anterior cardiac veins begin over the ante- of membranous and muscular parts. The conus rior surface of the right ventricle and cross over arteriosus leads into the pulmonary trunk. A the coronary groove to end directly in the right thick muscular ridge, the supraventricular crest, atrium (Moore, pp 136–137). separates the ridged muscular wall of the inflow part of the right ventricle from the smooth wall 23. (E) The SA node is located anterolaterally just of the conus arteriosus or outflow part (Moore, deep to the epicardium at the junction of the SVC p 127). and right atrium, near the superior end of the sulcus terminalis. The SA node, a small collec- 15. (C) Typically, the right coronary artery supplies tion of nodal tissue and specialized cardiac mus- the right atrium, most of the right ventricle, part cle fibers, is the pacemaker of the heart. The SA of the left ventricle, part of the AV septum, and node is supplied by a branch of the right coro- the SA node in approximately 60% of people and nary artery in about 60% of individuals (Moore, the AV node in approximately 80% of people p 137). (Moore, p 135). 24. (E) The interior of the left ventricle includes 16. (D) The alveolus is the basic unit of gas exchange anterior and posterior papillary muscles (Moore, in the lung (Moore, p 104). p 131).

17. (B) The pulmonary veins, two on each side, 25. (A) The interior of the left atrium has a large- carry well-oxygenated (“arterial”) blood from walled part and a smaller muscular auricle con- the lungs to the left atrium of the heart (Moore, taining pectinate muscles (Moore, p 129). p 107). 26. (E) The chest consists of 12 pairs of ribs, the ster- 18. (C) The posterior mediastinum contains the tho- num, costal cartilages, and 12 thoracic vertebrae racic aorta, thoracic duct, azygos and hemiazy- (Moore, p 60). gos veins, esophagus, and thoracic sympathetic trunks (Moore, pp 150–151). 27. (C) Chest pain may result from cardiac disease, pulmonary disease, intestinal problems, gall- 19. (C) The dorsal and ventral rami are both motor bladder disorders, and musculoskeletal dis- and sensory. The cutaneous branches include orders (Moore, p 61). anterior, lateral, and posterior branches. The dor- sal root is sensory and the ventral root is motor. 28. (D) Thoracic vertebrae have costal facets or The spinal nerves supply a band-like skin area demifacets on their bodies, costal facets on the known as a dermatome (Moore, p 85). transverse processes (for the first 9 or 10 thoracic vertebrae), and long spinous processes (Moore, p 65). 0523-03 C hap 3 07/15/02 15:32 Page 43

Answers and Explanations: 12–51 43

29. (A) The sternoclavicular joint is a saddle-type 42. (E) The branches of the right coronary artery synovial joint (Moore, p 69). include the SA nodal, right marginal, posterior interventricular, and AV nodal arteries (Moore, 30. (C) The mammary glands are modified sweat p 134). glands (Moore, p 74). 43. (C) The superior mediastinum contains the thy- 31. (E) The breast is supplied by the internal tho- mus, great vessels and their branches, vagus racic, lateral thoracic, thoracoacromial, and pos- nerves, phrenic nerves, cardiac plexus, left recur- terior intercostal arteries (Moore, p 75). rent laryngeal nerve, trachea, esophagus, thora- cic duct, and prevertebral muscles (Moore, p 142). 32. (A) The levatores costarum are innervated by the dorsal primary rami of C8–T11. (Moore, p 84). 44. (A) The thymus is supplied by the anterior intercostal and anterior mediastinal branches of 33. (B) The serratus posterior inferior depresses the the internal thoracic artery (Moore, p 142). ribs (Moore, p 84). 45. (B) Branches of the aorta in the thorax include 34. (A) The intercostobrachial nerve communicates the posterior intercostal, bronchial, esophageal, with the medial brachial cutaneous nerve (Moore, and superior phrenic arteries (Moore, p 145). p 87). 46. (A) The posterior mediastinum includes the 35. (A) The musculophrenic artery gives rise to thoracic aorta, thoracic duct, posterior mediasti- anterior intercostal arteries for intercostal spaces nal lymph nodes, azygos and hemiazygos veins, 7–9 (Moore, p 88). esophagus, esophageal plexus, thoracic sympa- thetic trunks, and thoracic splanchnic nerves 36. (C) Anterior and posterior intercostal arteries (Moore, pp 150–151). pass between the internal and innermost inter- costal muscles (Moore, p 88). 47. (A) The esophagus is compressed by the aortic arch, left main bronchus, and diaphragm (Moore, 37. (D) The superior and inferior bronchopulmo- p 152). nary segments of the superior lobe of the left lung combine to form the lingula (Moore, p 106). 48. (A) The azygos vein receives blood from the posterior intercostal veins, vertebral venous 38. (E) Visceral afferent fibers are distributed to plexuses, mediastinal veins, esophageal veins, bronchial mucosa, bronchial muscles, interalve- and bronchial veins (Moore, p 155). olar connective tissue, and pulmonary arteries and veins (Moore, p 111). 49. (D) The hemiazygos vein receives blood from the left subcostal vein, ascending lumbar veins, 39. (A) The diaphragm receives somatic efferent the inferior three posterior intercostal veins, infe- and afferent innervation—not sympathetic in- rior esophageal veins, and small mediastinal nervation (Moore, p 113). veins (Moore, p 155).

40. (A) While a person is supine, the arch of the 50. (C) The accessory hemiazygos descends on the aorta lies superior to the transverse thoracic left side of the vertebral column from T5 through plane (Moore, p 115). T8 (Moore, p 155).

41. (E) The pericardium receives blood supply from 51. (E) The anterior mediastinum contains fat, ster- the pericardiacophrenic artery, musculophrenic nopericardial ligaments, branches of internal artery, bronchial arteries, esophageal arteries, thoracic vessels, and lymphatic vessels (Moore, superior phrenic arteries, and coronary arteries p 156). (Moore, p 120). 0523-03 C hap 3 07/15/02 15:32 Page 44

44 3: The Thorax

52. (A) The sympathetic trunk contains the supe- receptors) and veins (as chemoreceptors) (Moore, rior, middle, and inferior cervical ganglia; par- p 111). avertebral ganglia; and the ganglion impar (Moore, p 46). 62. (C) The greater, lesser, and least splanchnic nerves are lower thoracic splanchnic nerves 53. (E) The cardiac plexus does not contain syn- (Moore, p 301). apses for sympathetic fibers, as these fibers have already synapsed in the sympathetic 63. (B) The greater thoracic splanchnic nerve orig- trunk (Moore, p 150). inates from vertebral levels T5–9 (Moore, p 301).

54. (D) In the synaptic trunk, presynaptic neurons 64. (A) The fibers of the greater splanchnic nerve synapse with postsynaptic neurons immedi- synapse in the celiac ganglion (Moore, p 302). ately. Some fibers ascend and synapse, while other fibers descend and synapse. Some fibers 65. (E) The level of the domes of the diaphragm pass into a splanchnic nerve without synapsing varies according to phase of respiration, posture, (Moore, p 46). and size and degree of distention of abdominal viscera (Moore, p 289). 55. (A) With the exception of sweat glands, glan- dular secretion is parasympathetically stimu- 66. (A) The inferior vena cava, terminal branches lated (Moore, p 51). of the right phrenic nerve, and lymphatics pass through the caval opening of the diaphragm 56. (B) With the exception of coronary arteries, (Moore, p 294). vasoconstriction is sympathetically stimulated (Moore, p 51). 67. (E) The esophagus, anterior and posterior vagal trunks, esophageal branches of the left gastric 57. (B) Sympathetic stimulation of the heart in- vessels, and lymphatic vessels pass through the creases the rate and strength of contraction and esophageal hiatus (Moore, p 294). inhibits the effect of the parasympathetic sys- tem on coronary arteries, allowing them to 68. (B) The aorta, thoracic duct, and azygos vein dilate (Moore, p 51). (sometimes) pass through the aortic hiatus (Moore, p 295). 58. (E) Sympathetic stimulation of the lungs results in inhibition of the parasympathetic system, 69. (A) Ribs form from costal processes of thoracic bronchodilation, reduced secretion, and maxi- vertebrae and thus are derived from the sclero- mum air exchange (Moore, p 51). tome of paraxial medoderm (Sadler, p 184).

59. (B) Sympathetic fibers of the heart are accom- 70. (A) The pleuropericardial membranes develop panied by visceral afferent fibers, which convey into the fibrous pericardium (Sadler, p 201). pain information (Moore, p 52). 71. (E) The diaphragm is derived from the septum 60. (C) The pulmonary plexus contains cell bodies transversum and pleuroperitoneal membranes. of postsynaptic parasympathetic fibers (Moore, Muscular components stem from lateral and p 110). dorsal body walls and mesentery of the esoph- agus (Sadler, p 206). 61. (E) Visceral afferent fibers of the tenth cranial nerve are distributed to bronchial mucosa (for 72. (C) The entire cardiovascular system is derived cough reflex), bronchial muscles (for stretch from mesoderm. (Sadler, p 256) reception), interalveolar connective tissue (for Hering-Breuer reflexes, limiting respiratory ex- 73. (E) The carotid arteries, arch of the aorta, pul- cursions), and pulmonary arteries (as pressure monary artery, and right subclavian artery are 0523-03 C hap 3 07/15/02 15:32 Page 45

Answers and Explanations: 52–95 45

derived from the original aortic arch system 83. ligamentum arteriosum (Sadler, p 257). 84. left 74. (B) The respiratory system is an outgrowth of the ventral wall of the foregut (Sadler, p 268) 85. left internal jugular vein

75. (A) The lungs develop in the following periods: 86. pulmonary arteries pseudoglandular period, canalicular period, ter- minal sac period, and alveolar period (Sadler, 87. aorta p 266). 88. superior vena cava 76. inferior thyroid vein 89. left inferior pulmonary vein 77. right brachiocephalic vein 90. inferior vena cava 78. phrenic nerve and pericardiacophrenic vessels 91. manubrium of sternum 79. right pulmonary artery 92. oblique fissure 80. left auricle 93. costal cartilage 81. lingula 94. xiphoid process 82. left pulmonary vein 95. copula This page intentionally left blank 0523-04 C hap 4 07/15/02 15:32 Page 47

CHAPTER 4 The Abdomen Questions

DIRECTIONS (Questions 1 through 100): Each of the 4. Where is extraperitoneal fat located? numbered items or incomplete statements in this (A) between the abdominal oblique muscles section is followed by answers or by completions of the statement. Select the ONE lettered answer or (B) deep to the parietal peritoneum completion that is BEST in each case. (C) superficial to Camper’s fascia (D) superficial to the deep fascia 1. For general clinical descriptions, which of the (E) deep to the transversalis fascia following planes is used as one of the planes to define four quadrants of the abdominal cavity? 5. The superficial muscle fibers of the external (A) subcostal abdominal oblique arising from the middle to lower ribs interdigitate with which of the fol- (B) transtubercular lowing muscles? (C) midclavicular (D) transumbilical (A) internal abdominal oblique (E) midaxillary (B) serratus anterior (C) rectus abdominis 2. The midclavicular planes pass through the mid- (D) pyramidalis point of the to the midpoint of which (E) transversus abdominis of the following structures? (A) anterior superior iliac spine 6. The anterolateral abdominal wall is bounded by all of the following structures EXCEPT (B) symphysis pubis (C) umbilicus (A) cartilages of the 7th through 10th ribs (D) inguinal ligament (B) linea alba (E) xiphoid process (C) xiphoid process (D) inguinal ligament 3. The fascial layer that covers the deep surface (E) pelvic bone of the transverse abdominal muscle is known as which of the following? 7. The aponeuroses of all three flat muscles of the (A) parietal peritoneum anterolateral abdominal wall interlace in which of the following structures? (B) deep fascia (C) transversalis fascia (A) inguinal ligament (D) Scarpa’s fascia (B) transversalis fascia (E) Camper’s fascia (C) linea alba (D) anterior superior iliac spine (E) rectus abdominis

47 0523-04 C hap 4 07/15/02 15:32 Page 48

48 4: The Abdomen

8. All of the following structures are located 13. Which of the following structures defines the within the rectus sheath EXCEPT point at which the posterior lamina of the inter- nal oblique and the aponeurosis of the trans- (A) pyramidalis verse abdominal become part of the anterior (B) rectus abdominis rectus sheath? (C) inferior epigastric arteries and veins (A) arcuate line (D) deep inguinal ring (B) inguinal ligament (E) ventral primary rami of T7–T12 nerves (C) tendinous intersections 9. Inferiorly, the inferior margin of the external (D) deep inguinal ring oblique aponeurosis thickens and folds back on (E) medial crus itself to form which of the following structures? 14. The two medial umbilical folds represent rem- (A) rectus sheath nants of which of the following structures? (B) inguinal ligament (C) arcuate line (A) urachus (D) deep inguinal ring (B) umbilical arteries (E) fundiform ligament (C) umbilical veins (D) ductus venosus 10. The reflected inguinal ligament receives fibers (E) ductus arteriosus from the contralateral aponeurosis of which of the following structures? 15. Which of the following fossae are potential sites for direct inguinal hernias? (A) external abdominal oblique (B) internal abdominal oblique (A) supravesical (C) transverse abdominal oblique (B) medial inguinal (D) rectus abdominis (C) lateral inguinal (E) pyramidalis (D) ischiorectal (E) iliac 11. Between the internal oblique and transverse abdominal muscles is a neurovascular plane 16. The inguinal canal contains which of the fol- that contains all of the following EXCEPT lowing nerves? (A) iliohypogastric nerve (A) iliohypogastric (B) deep circumflex iliac artery (B) ilioinguinal (C) inferior epigastric artery (C) genital branch of the genitofemoral (D) (D) obturator (E) ilioinguinal nerve (E) lateral femoral cutaneous

12. The is anchored trans- 17. Which of the following structures give rise to versely by attachment to the anterior layer of the deep inguinal ring? the rectus sheath by which of the following (A) gubernaculum structures? (B) conjoined tendon (A) pubic tubercle (C) lacunar ligament (B) xiphoid process (D) external abdominal oblique aponeurosis (C) linea alba (E) transversalis fascia (D) tendinous intersections (E) umbilicus 0523-04 C hap 4 07/15/02 15:32 Page 49

Questions: 8–28 49

18. The lacunar ligament is a reflection or exten- (C) T12 sion from the deep aspect of which of the fol- (D) femoral lowing structures? (E) obturator (A) falciform ligament (B) round ligament 24. The artery of the ductus deferens arises from which of the following structures? (C) rectus sheath (D) inguinal ligament (A) aorta (E) transversalis fascia (B) inferior epigastric (C) inferior vesical 19. The iliopubic tract is the thickened inferior (D) pudendal margin of which of the following structures? (E) common iliac (A) inguinal ligament (B) transversalis fascia 25. All of the following nerves contribute branches to the EXCEPT (C) conjoined tendon (D) falciform ligament (A) lateral femoral cutaneous (E) round ligament (B) pudendal (C) ilioinguinal 20. The testes develop in which of the following (D) genitofemoral areas? (E) posterior femoral cutaneous (A) scrotum (B) abdominal cavity 26. The epididymis is located on the posterior as- pect of which of the following structures? (C) extraperitoneal (D) rectus sheath (A) (E) superficial fascia (B) (C) testis 21. The gubernaculum is represented postnatally (D) ovary by which of the following structures? (E) uterine tube (A) tunica vaginalis testes (B) processus vaginalis 27. The testes are covered by a tough fibrous coat known as the (C) ductus deferens (D) scrotal ligament (A) cremaster fascia (E) internal spermatic fascia (B) tunica albuginea (C) gubernaculum 22. The cremaster muscle and fascia are derived (D) tunica from which of the following structures? (E) Scarpa’s fascia (A) external abdominal oblique muscle (B) transverse abdominal muscle and fascia 28. Which of the following structures represents the closed-off distal part of the embryonic pro- (C) internal abdominal muscle cessus vaginalis? (D) transversalis fascia (E) external abdominal aponeurosis (A) gubernaculum (B) tunica albuginea 23. The cremaster muscle is innervated by which (C) epididymis of the following nerves? (D) tunica vaginalis (A) genital branch of the genitofemoral (E) (B) ilioinguinal 0523-04 C hap 4 07/15/02 15:32 Page 50

50 4: The Abdomen

29. The pampiniform plexus provides which of (A) greater omentum the following testicular functions? (B) falciform ligament (A) lymphatic drainage (C) gastrohepatic ligament (B) blood supply (D) hepatoduodenal ligament (C) nerve supply (E) gastrocolic ligament (D) thermoregulatory system 35. Which of the following structures forms the (E) hormonal production superior boundary of the omental foramen? 30. The parasympathetic innervation of the testis (A) inferior vena cava includes which of the following nerves? (B) duodenum (A) pelvic splanchnic (C) caudate lobe of the (B) vagus (D) head of the (C) iliohypogastric (E) hepatoduodenal ligament (D) ilioinguinal 36. Which of the following structures contains both (E) pudendal smooth and skeletal muscles? 31. The peritoneal cavity contains which of the (A) following? (B) jejunum (A) liver (C) cecum (B) pancreas (D) esophagus (C) (E) (D) kidney 37. Which of the following arteries provides the (E) peritoneal fluid abdominal parts of the esophagus with its arte- rial supply? 32. Which of the following structures connects the lesser curvature of the stomach and the proxi- (A) cystic mal part of the duodenum to the liver? (B) gastroduodenal (A) lesser omentum (C) left gastric (B) peritoneal ligament (D) hepatic (C) mesentery (E) left gastroepiploic (D) lesser omentum 38. Rugae are located in which of the following (E) peritoneal fold structures? 33. Which of the following structures is often (A) duodenum referred to as the “abdominal policeman”? (B) stomach (A) hepatoduodenal ligament (C) cecum (B) gastrohepatic ligament (D) ileum (C) greater omentum (E) transverse colon (D) gastrocolic ligament 39. The left gastro-omental artery arises from which (E) falciform ligament of the following arteries? 34. Which of the following ligaments conducts (A) splenic the portal triad (portal vein, hepatic artery, (B) hepatic and bile duct)? (C) gastroduodenal 0523-04 C hap 4 07/15/02 15:32 Page 51

Questions: 29–49 51

(D) left gastric 45. The superior anterior and posterior pancreati- (E) right gastric coduodenal arteries arise from which of the fol- lowing arteries? 40. The sympathetic nerve supply of the stomach (A) right colic arises from which of the following cord seg- (B) ileocolic ments? (C) gastroduodenal (A) T1–T5 (D) hepatic (B) T6–T9 (E) splenic (C) T6–T12 (D) L1–L3 46. The root of the mesentery crosses all of the fol- (E) T10–L2 lowing structures EXCEPT (A) ascending and horizontal parts of the 41. The first part of the duodenum is located at duodenum which of the following vertebral levels? (B) abdominal aorta (A) T10 (C) inferior vena cava (B) L2 (D) right (C) L1 (E) splenic artery (D) L5 (E) L3 47. The superior mesenteric and splenic veins unite to form the portal vein posterior to which 42. The bile and pancreatic ducts enter which of of the following structures? the following structures? (A) right kidney (A) stomach (B) neck of the pancreas (B) 2nd portion of the duodenum (C) pylorus of stomach (C) cecum (D) 2nd portion of the duodenum (D) ileum (E) (E) liver 48. The sympathetic fibers in the nerves to the 43. Which of the following structures crosses over jejunum and ileum originate in which of the the inferior or horizontal (third) portion of the following spinal cord segments? duodenum? (A) C5–T1 (A) pancreas (B) T1–T5 (B) hepatic artery (C) T5–T9 (C) portal vein (D) T9–T12 (D) superior mesenteric artery (E) L1–L2

44. The duodenojejunal junction is supported by 49. Circular folds (plicae circulares) are character- the attachment of which of the following istic of which of the following structures? structures? (A) transverse colon (A) suspensory muscle of the duodenum (B) stomach (ligament of Treitz) (C) jejunum (B) falciform ligament (D) cecum (C) hepatoduodenal ligament (E) sigmoid colon (D) greater omentum (E) transverse mesocolon 0523-04 C hap 4 07/15/02 15:32 Page 52

52 4: The Abdomen

50. Omental appendices are located on which of (A) L3 the following structures? (B) L5 (A) stomach (C) S3 (B) duodenum (D) S5 (C) ileum (E) coccygeal 1 (D) ascending colon 56. All of the following statements concerning the (E) liver spleen are correct EXCEPT 51. The are no teniae coli in which of the follow- (A) largest of the lymphatic organs ing structures? (B) associated posteriorly with the left 9th (A) ascending colon through 11th ribs (B) transverse colon (C) located retroperitoneally (C) descending colon (D) normally, does not descend inferior to the costal region (D) sigmoid colon (E) (E) varies considerably in size, weight, and shape 52. The appendicular artery is a branch of which of the following arteries? 57. The splenic artery usually follows a tortuous course along which of the following structures? (A) inferior mesenteric (A) left kidney (B) inferior epigastric (B) greater curvature of the stomach (C) ileocolic (C) pancreas (D) testicular (D) transverse colon (E) renal (E) cecum 53. Which of the following structures can be located deep to a point that is one-third of the 58. The head of the pancreas is embraced by which way along the oblique line, joining the right of the following structures? anterior superior iliac spine to the umbilicus? (A) stomach (A) gallbladder (B) spleen (B) spleen (C) cecum (C) right kidney (D) C-shaped curve of the duodenum (D) appendix (E) transverse mesocolon (E) urinary bladder 59. The head of the pancreas rests posteriorly on 54. Which of the following structures receives which of the following structures? parasympathetic innervation from the pelvic (A) left renal vein splanchnic nerves? (B) superior vena cava (A) appendix (C) splenic artery (B) sigmoid colon (D) duodenum (C) ileum (E) superior mesenteric artery (D) ascending colon (E) duodenum 60. The main pancreatic duct and the bile duct unite to form which of the following structures? 55. The rectum is continuous with the sigmoid (A) common bile duct colon at the level of which of the following ver- (B) hepatic duct tebrae? 0523-04 C hap 4 07/15/02 15:32 Page 53

Questions: 50–70 53

(C) accessory pancreatic duct 66. The spiral valve is located in which of the fol- (D) cystic duct lowing structures? (E) hepatopancreatic ampulla (A) head of pancreas (B) pylorus 61. The round ligament of the liver is the fibrous (C) neck of gallbladder remnant of which of the following structures? (D) cecum (A) umbilical vein (E) rectum (B) ductus venosus (C) ductus arteriosus 67. The cystic artery commonly arises from the (D) umbilical artery right hepatic artery in the angle between the (E) urachus common hepatic duct and which of the follow- ing structures? 62. The porta hepatis gives passage to all of the (A) cystic duct following structures EXCEPT (B) celiac trunk (A) portal vein (C) portal vein (B) hepatic artery (D) proper hepatic artery (C) hepatic ducts (E) gastroduodenal artery (D) lymphatic vessels (E) cystic artery 68. Which of the following veins, when dilated, produces caput medusae? 63. Which of the following ligaments encloses the (A) proper hepatic portal triad? (B) splenic (A) hepatoduodenal (C) cystic (B) hepatogastric (D) paraumbilical (C) gastrocolic (E) rectal (D) hepatorenal (E) gastrosplenic 69. Inferiorly, the posterior surfaces of the kidney are related to all of the following structures 64. The portion of the hepatic artery extending EXCEPT between the celiac trunk and the gastroduo- (A) subcostal nerve denal artery is known as the (B) iliohypogastric nerve (A) proper hepatic (C) ilioinguinal nerve (B) common hepatic (D) 2nd portion of the duodenum (C) right hepatic (E) quadratus lumborum (D) left hepatic (E) middle hepatic 70. The renal papillae empty into which of the fol- lowing structures? 65. The hepatic veins drain into which of the fol- (A) renal vein lowing structures? (B) ureter (A) liver (C) minor calyces (B) inferior vena cava (D) renal pyramid (C) spleen (E) renal column (D) portal vein (E) superior vena cava 0523-04 C hap 4 07/15/02 15:32 Page 54

54 4: The Abdomen

71. All of the following statements concerning the 76. Which of the following muscles is considered renal hilum are correct EXCEPT to be the chief muscle of inspiration? (A) the renal vein is anterior to the renal (A) internal intercostal artery (B) external intercostal (B) the renal artery is anterior to the renal (C) diaphragm pelvis (D) scalene (C) it is the entrance to the renal sinus (E) sternocleidomastoid (D) it is located on the concave medial mar- gin of the kidney 77. All of the following statements concerning the (E) it contains the renal pyramids central tendon of the diaphragm are correct EXCEPT 72. The suprarenal glands are located between the (A) It has no bony attachments. superomedial aspects of the kidneys and which of the following structures? (B) It is incompletely divided into three leaves. (A) neck of the pancreas (C) It is perforated by the aorta. (B) diaphragm (D) It lies near the center of the diaphragm. (C) quadrate lobe of the liver (E) It is perforated by the inferior vena cava. (D) mesentery (E) 1st part of the duodenum 78. The crura of the diaphragm are musculotendi- nous bundles that arise from which of the fol- 73. Which of the following structures is related to lowing structures? the spleen, stomach, pancreas, and the left crus (A) posterior longitudinal ligament of the diaphragm? (B) sternum (A) left suprarenal gland (C) bodies of lumbar vertebrae L1, L2, and L3 (B) left kidney (D) renal fascia (C) left gonadal vein (E) psoas major muscle (D) transverse mesocolon (E) abdominal aorta 79. The nerves of the kidneys and suprarenal glands are derived from which of the following 74. All of the following statements concerning the plexuses? suprarenal cortex are correct EXCEPT (A) celiac (A) derives from mesoderm (B) lumbar (B) secretes corticosteroids (C) inferior mesenteric (C) secretes androgens (D) sacral (D) associated with the sympathetic nervous (E) inferior hypogastric system (E) causes the kidney to retain sodium 80. The lateral arcuate ligaments are formed from thickenings from which of the following mus- 75. The superior suprarenal arteries are branches cular fasciae? of which of the following arteries? (A) psoas major (A) abdominal aorta (B) quadratus lumborum (B) renal (C) transversus abdominis (C) inferior phrenic (D) rectus abdominis (D) celiac trunk (E) sternalis (E) superior mesenteric 0523-04 C hap 4 07/15/02 15:32 Page 55

Questions: 71–90 55

81. All of the following structures may herniate (A) right crus of the diaphragm into the when there is a trauma- (B) right kidney tic diaphragmatic hernia EXCEPT (C) 5th lumbar vertebra (A) stomach (D) crest of the ilium (B) kidney (E) bifurcation of the aorta (C) intestine 87. The cisterna chyli is the inferior end of which (D) mesentery of the following structures? (E) spleen (A) inferior vena cava 82. All of the following structures pass through (B) abdominal aorta the esophageal hiatus EXCEPT (C) renal vein (A) posterior vagal trunk (D) testicular artery (B) esophageal branches of the left gastric (E) thoracic duct vessels 88. How are the pelvic splanchnic nerves distinct (C) anterior vagal trunk from other splanchnic nerves? (D) thoracic duct (E) esophagus (A) derived from ventral primary rami of L2, L3, and L4 83. The greater and lesser splanchnic nerves pass (B) derived from the sympathetic trunks through the diaphragm via which of the fol- (C) convey preganglionic parasympathetic lowing structures? fibers to the inferior hypogastric plexus (A) sternocostal foramen (D) provide postganglionic sympathetic innervation to the cecum (B) aortic hiatus (E) convey postganglionic parasympathetic (C) diaphragmatic crus innervation to the ascending colon (D) vena caval foramen (E) medial arcuate ligament 89. All of the following statements concerning the psoas major muscle are correct EXCEPT 84. The parasympathetic root of the celiac plexus (A) It passes inferolaterally, deep to the is a branch of which of the following? inguinal ligament. (A) greater splanchnic (B) It inserts on the lesser trochanter of the (B) pelvic splanchnic . (C) lumbar splanchnic (C) The pelvic splanchnic nerves are embed- (D) posterior vagal trunk ded in the posterior part of the psoas. (E) lesser splanchnic (D) It is a long, thick and fusiform muscle. (E) Its name stems from a Greek word 85. The bifurcation of the abdominal aorta occurs at meaning “muscle of the loin.” the level of which of the following structures? 90. Which of the following statements correctly (A) crest of the ilium applies to the suprarenal medulla? (B) inguinal ligament (A) derived from mesoderm (C) pubic tubercle (B) secretes corticosteroids and androgens (D) symphysis pubis (C) contains chromaffin cells (E) obturator foramen (D) secretes acetylcholine 86. The inferior vena cava begins anterior to which (E) produces hormones that cause the kidney of the following structures? to retain sodium and water in response to stress 0523-04 C hap 4 07/15/02 15:32 Page 56

56 4: The Abdomen

91. The anatomical left and right lobes of the liver (D) It attaches to the hepatoduodenal are separated on the diaphragmatic surface of ligament. the liver by which of the following structures? (E) It lies to the right and runs parallel to the (A) fissure for the round ligament of the inferior vena cava. liver 96. The left gastro-omental artery arises from (B) fissure for the ligamentum venosum which of the following arteries? (C) falciform ligament (D) porta hepatis (A) celiac trunk (E) lesser omentum (B) right gastric (C) gastroduodenal 92. All of the following statements concerning the (D) splenic spleen are correct EXCEPT (E) hepatic (A) It is the largest branch of the celiac trunk. 97. Rugae are located in which of the following (B) It follows a tortuous course along the structures? inferior border of the pancreas. (C) It divides into five or more branches that (A) duodenum enter the hilum of the spleen. (B) jejunum (D) It runs anterior to the left kidney. (C) ileum (E) It follows posterior to the omental bursa. (D) cecum (E) stomach 93. In the developing embryo, the midgut rotates 270 degrees around which of the following 98. All of the following statements concerning the structures? esophagus are correct EXCEPT (A) superior mesenteric artery (A) It extends from the to the (B) celiac trunk stomach. (C) splenic artery (B) It is crossed by the arch of the aorta. (D) left renal artery (C) It is crossed by the right main bronchus. (E) inferior vena cava (D) It passes through the esophageal hiatus. (E) It normally has four constrictions. 94. Which of the following structures is avascular? (A) superior ileocecal fold 99. Digestion occurs mainly in which of the fol- lowing structures? (B) inferior ileocecal fold (C) mesoappendix (A) cecum and ascending colon (D) appendix (B) transverse colon (E) cecum (C) stomach and duodenum (D) jejunum and ileum 95. Which of the following statements correctly (E) transverse and sigmoid colon relates to the 3rd part of the duodenum? (A) It is supported by the suspensory liga- 100. Most reabsorption of water occurs in which of ment of the duodenum. the following structures? (B) The bile and pancreatic ducts enter its (A) stomach posteromedial wall. (B) jejunum (C) It is crossed by the superior mesenteric (C) sigmoid colon artery and vein. (D) ascending colon (E) rectum 0523-04 C hap 4 07/15/02 15:32 Page 57

Questions: 91–110 57

DIRECTIONS (Questions 101 through 110): Identify the anatomical features indicated on the art below. 0523-04 C hap 4 07/15/02 15:32 Page 58

58 4: The Abdomen

DIRECTIONS (Questions 111 through 115): Identify the anatomical features indicated on the art below.

DIRECTIONS (Questions 116 through 120): Identify the anatomical features indicated on the art below. 0523-04 C hap 4 07/15/02 15:32 Page 59

Questions: 111–130 59

DIRECTIONS (Questions 121 through 125): Identify the anatomical features indicated on the art below.

DIRECTIONS (Questions 126 through 130): Identify the anatomical features indicated on the art below. 0523-04 C hap 4 07/15/02 15:32 Page 60

Answers and Explanations

1. (D) The transumbilical and median planes 8. (D) The contents of the rectus sheath include divide the abdomen into four quadrants the pyramidalis and rectus abdominis mus- (Moore, p 176). cles, the superior and inferior epigastric arter- ies and veins, the lymphatics, and ventral 2. (D) The midclavicular planes pass through primary rami of T7–T12 nerves (Moore, p 180). the midpoint of the clavicles to the midin- guinal points (Moore, p 176). 9. (B) Inferiorly, the inferior margin of the exter- nal oblique aponeurosis thickens and folds back 3. (C) Transversalis fascia lines most of the ab- on itself to form the inguinal ligament, a fibrous dominal wall and covers the deep surface of the band extending between the anterior superior transverse abdominal muscle. The deep fascia iliac spine and the pubic tubercle (Moore, p 180). invests the external abdominal oblique muscle. Both Camper’s and Scarpa’s fascia are located 10. (A) Some fibers of the inguinal ligament pass in the inferior part of the abdominal subcuta- upward to cross the linea alba and blend with neous tissue (Moore, pp 178–179). the lower fibers of the contralateral aponeuro- sis. These fibers form the reflected inguinal lig- 4. (E) The parietal peritoneum is internal to the ament (Moore, p 180). transversalis fascia and is separated from it by a variable amount of endoabdominal (extra- 11. (C) The neurovascular plane of the antero- peritoneal) fat. Camper’s fascia and the deep lateral abdominal wall contains the thoraco- fasciae are all superficial to the extraperitoneal abdominal nerves, cutaneous branches T7–T11, fat (Moore, p 179). ventral ramus of T12, iliohypogastric and ilioin- guinal nerves, lumbar arteries, and the deep cir- 5. (B) The superficial fibers of the external ab- cumflex iliac artery (Moore, p 182). dominal oblique arising from the middle and lower ribs interdigitate with only those of the 12. (D) The rectus muscle is anchored trans- serratus anterior (Moore, p 180). versely by attachments to the anterior layer of the rectus sheath and three or more tendinous 6. (B) The anterolateral abdominal wall includes intersections (Moore, p 183). the cartilages of the 7th–10th ribs and xiphoid process superiorly. Inferiorly it is bounded by 13. (A) The inferior limit of the posterior lamina of the inguinal ligament and the pelvic bone the rectus sheath is marked by the arcuate line, (Moore, p 178). which defines the point at which the posterior lamina of the internal oblique and the aponeu- 7. (C) The aponeuroses of these muscles inter- rosis of the transverse abdominal become part lace at the linea alba with their fellows of the of the anterior rectus sheath (Moore, p 184). opposite side to form the tough, aponeurotic tendinous sheath of the rectus muscle, and 14. (B) The median umbilical fold represents the the rectus sheath (Moore, p 180). remnant of the urachus; the medial umbilical

60 0523-04 C hap 4 07/15/02 15:32 Page 61

Answers and Explanations: 1–31 61

folds are remnants of the occluded fetal umbil- 22. (C) The cremasteric fascia contains loops of cre- ical arteries; and the lateral umbilical folds master muscle, which is formed by the lower- cover the inferior epigastric vessels (Moore, most fascicles of the internal oblique muscle p 191). arising from the inguinal ligament. The cremas- teric fascia is derived from the deep and super- 15. (B) The medial inguinal fossae between the ficial fascia of the internal oblique (Moore, p 198). medial and lateral umbilical folds, the inguinal triangles, are potential sites for the less common 23. (A) The cremaster muscle is innervated by the direct inguinal hernias. The lateral inguinal fos- genital branch of the genitofemoral nerve sae, lateral to the lateral umbilical folds, are po- (Moore, p 198). tential sites for the most common type of hernia in the lower abdominal wall—indirect inguinal 24. (C) The artery of the ductus deferens arises hernia (Moore, p 191). from the inferior vesicle (Moore, p 198).

16. (B) The inguinal canal contains the spermatic 25. (A) Genital branch of the genitofemoral, cord in the male and the round ligament in the anterior scrotal branches of the ilioinguinal, female. The inguinal canal also contains blood posterior scrotal branches of the pudendal, and and lymphatic vessels and the ilioinguinal perineal branches of the posterior femoral cuta- nerve in both sexes (Moore, p 193). neous provide the innervation to the scrotum (Moore, p 201). 17. (E) The deep inguinal ring (entrance to the inguinal canal) is the site of an outpouching of 26. (C) The epididymis lies on the posterior sur- the transversalis fascia approximately 1.2 cm face of the testis, which is covered by the tunica superior to the middle of the inguinal liga- vaginalis except at its posterior margin (Moore, ment (Moore, p 193). p 201). 27. (B) 18. (D) The inguinal ligament is reinforced in its The testes are covered with a tough fi- brous coat know as the tunica albuginea most medial part by the lacunar ligament, a re- (Moore, p 201). flected part or extension from the deep aspect of the inguinal ligament to the pectineal line of 28. (D) The tunica vaginalis is a closed peritoneal the pecten pubis (Moore, p 193). sac partially surrounding the testis, which rep- resents the closed-off distal part of the embry- 19. (B) The iliopubic tract is the thickened inferior onic processus vaginalis (Moore, p 202). margin of the transversalis fascia that appears as a fibrous band running parallel and deep 29. (D) The pampiniform plexus is part of the to the inguinal ligament. The iliopubic tract thermoregulatory system of the testis, helping demarcates the inferior edge of the deep ingui- to keep this gland at a constant temperature (Moore, p 193). nal ring (Moore, p 202).

20. (C) The testes develop in the extraperitoneal 30. (B) The autonomic nerves of the testis arise as connective tissue in the superior lumbar region the testicular plexus of nerves on the testicular of the posterior abdominal wall (Moore, p 193). artery, which contains vagal parasympathetic fibers and sympathetic fibers from the T7 seg- 21. (D) The gubernaculum is a fibrous cord con- ment of the spinal cord (Moore, p 202). necting the primordial testis to the anterolateral wall at the site of the future deep inguinal ring. 31. (E) The peritoneal cavity is a potential space of The gubernaculum is represented postnatally capillary thinness between the parietal and vis- by the scrotal ligament, which extends from the ceral layers of peritoneum. It contains no organs testis to the skin of the scrotum (Moore, p 194). but rather a thin film of peritoneal fluid that lubricates the peritoneal surfaces (Moore, p 210). 0523-04 C hap 4 07/15/02 15:32 Page 62

62 4: The Abdomen

32. (D) The lesser omentum connects the lesser cord passes to the celiac plexus through the curvature of the stomach and the proximal part greater splanchnic nerve (Moore, p 231). of the duodenum to the liver (Moore, p 213). 41. (C) The superior (1st) part of the duodenum 33. (C) The greater omentum has considerable lies anterolateral to the body of L1 vertebra mobility and moves around the peritoneal (Moore, p 237). cavity with peristaltic movements of the vis- cera. It wraps itself around an inflamed 42. (B) The bile and pancreatic ducts enter the pos- such as the appendix, walling it off and teromedial wall of the 2nd portion of the duo- thereby protecting other viscera from it. For denum (Moore, p 237). this reason, the great omentum is referred to as the “policeman of the peritoneal cavity.” 43. (D) The inferior or horizontal (third) portion of the duodenum is crossed by the superior 34. (D) The liver is connected to the duodenum mesenteric artery and vein and the root of the by the hepatoduodenal ligament (the thick- mesentery of the jejunum and ileum (Moore, ened free edge of the lesser omentum that con- p 237). ducts the portal triad: portal vein, hepatic artery, and bile duct) (Moore, p 213). 44. (A) The duodenojejunal junction is supported by the suspensory muscle of the duodenum (lig- 35. (C) The caudate lobe of the liver, covered with ament of Treitz) (Moore, p 237). visceral peritoneum, forms the superior bound- ary of the omental foramen (epiploic foramen, 45. (C) The gastroduodenal artery and its or foramen of Winslow), an opening situated branches, the superior anterior and posterior posterior to the free edge of the lesser omentum pancreaticoduodenal arteries, supply the duo- (hepatoduodenal ligament). denum proximal to the entry of the bile duct into the descending (second) portion of the 36. (D) The esophagus has internal circular and duodenum (Moore, p 241). external longitudinal layers of muscle. In its superior third, the external layer consists of 46. (E) The root of the mesentery crosses (succes- , the inferior third is composed sively) the ascending and horizontal parts of of smooth muscle, and the middle third is made the duodenum, abdominal aorta, inferior vena up of both types of muscle (Moore, p 224). cava, right ureter, right psoas major, and right testicular or ovarian vessels (Moore, p 244). 37. (C) The arterial supply of the abdominal part of the esophagus is from the left gastric artery, 47. (B) The superior mesenteric and splenic veins a branch of the celiac trunk, and the left infe- unite to form the portal vein posterior to the rior phrenic artery (Moore, p 225). neck of the pancreas (Moore, p 244).

38. (B) When the gastric mucosa is contracted, it 48. (C) The sympathetic fibers in the nerves to the is thrown into longitudinal ridges known as jejunum and ileum originate in the T5 through gastric folds or rugae (Moore, p 227). T9 segments of the spinal cord (Moore, p 244).

39. (A) The left gastro-omental artery arises from 49. (C) The jejunum contains circular folds along the splenic artery and courses along the greater with the duodenum. The circular folds are curvature to anastomose with the right gastro- absent in the ileum and large intestine (Moore, omental artery (Moore, p 229). p 244).

40. (B) The sympathetic nerve supply of the stom- 50. (D) The large intestine can be distinguished ach from T6 through T9 segments of the spinal from the by teniae coli, haustra, and omental appendices (Moore, p 249). 0523-04 C hap 4 07/15/02 15:32 Page 63

Answers and Explanations: 32–69 63

51. (E) There are no teniae in the appendix or rec- 61. (A) The round ligament of the liver is the tum (Moore, p 249). fibrous remnant of the umbilical vein that car- ried well-oxygenated and nutrient-rich blood 52. (C) The appendix is supplied by the appendic- from the to the fetus (Moore, p 265). ular artery, a branch of the ileocolic artery (Moore, p 251). 62. (E) The porta hepatis gives passage to the por- tal vein, hepatic artery, hepatic nerve plexus, 53. (D) The base of the appendix lies deep to a hepatic ducts, and lymphatic vessels (Moore, point that is one-third of the way along the p 265). oblique line, joining the right anterior superior iliac spine to the umbilicus (spinoumbilical or 63. (A) The portion of the lesser omentum extend- McBurney’s point) (Moore, p 251). ing between the porta hepatis of the liver and the duodenum (the hepatoduodenal ligament) 54. (B) The pelvic splanchnic nerves provide the encloses the portal triad (Moore, p 265). parasympathetic innervation for the distal one-third of the transverse colon, descending 64. (B) The common hepatic artery extends from colon, and rectum (Moore, p 255). the celiac trunk to the origin of the gastroduo- denal artery. The proper hepatic artery extends 55. (C) The rectum, the fixed terminal part of the from the origin of the gastroduodenal artery large intestine, is continuous with the sigmoid to its bifurcation into right and left hepatic colon at the level of S3 vertebra (Moore, p 255). branches (Moore, p 265).

56. (C) The spleen is the largest of the lymphatic 65. (B) The hepatic veins, formed by the union of organs and is a mobile structure entirely sur- the central veins of the liver, open into the infe- rounded by peritoneum except at the hilum. rior vena cava just inferior to the diaphragm It lies beneath the 9th, 10th, and 11th ribs and (Moore, p 266). does not normally descend inferior to the costal margin. The spleen varies considerably 66. (C) The mucosa of the neck of the gallbladder in size, weight, and shape (Moore, p 256). spirals into a fold known as the spiral valve, which keeps the cystic duct open so that bile 57. (C) The splenic artery, the largest branch of can easily divert into the gallbladder when the celiac trunk, follows a tortuous course pos- the distal end of the bile duct is closed, or so bile terior to the omental bursa, anterior to the left can pass to the duodenum as the gallbladder kidney, and along the superior border of the contracts (Moore, p 274). pancreas (Moore, p 257). 67. (A) The cystic artery commonly arises from 58. (D) The head of the pancreas is embraced by the right hepatic artery in the angle between the C-shaped curve of the duodenum to the the common hepatic duct and the cystic duct right of the superior mesenteric vessels (Moore, (Moore, p 275). p 257). 68. (D) The paraumbilical veins of the anterior 59. (A) The head of the pancreas rests posteriorly abdominal wall (portal system) anastomosing on the inferior vena cava, right renal artery and with superficial epigastric veins (systemic sys- vein and left renal vein (Moore, p 259). tem), when dilated, produce caput medusae (Moore, p 277). 60. (D) The main pancreatic duct and the bile duct unite to form a short, dilated hepatopancreatic 69. (D) Inferiorly, the posterior surfaces of the ampulla, which opens into the descending part kidney are related to the quadratus lumborum of the duodenum at the summit of the major muscle and the subcostal, iliohypogastric, and duodenal papilla (Moore, p 261). ilioinguinal nerves and vessels (Moore, p 280). 0523-04 C hap 4 07/15/02 15:32 Page 64

64 4: The Abdomen

70. (C) The renal papillae empty into the minor cal- 78. (C) The crura of the diaphragm are musculo- ices, which empty into major calices which in tendinous bundles that arise from the ante- turn empty into the pelvis of the ureter (Moore, rior surfaces of the bodies of the superior three p 280). lumbar vertebrae, the anterior longitudinal ligament, and the intervertebral discs (Moore, 71. (E) At the concave medial margin of each kid- p 291). ney is a vertical cleft, the renal hilum, where the renal artery enters and the renal vein and renal 79. (A) The nerves of the kidneys and suprarenal pelvis leave the renal sinus. At the hilum, the glands are derived from the celiac plexus, the renal vein is anterior to the renal artery, which lesser and least thoracic splanchnic nerves, and is anterior to the renal pelvis (Moore, p 280). the aorticorenal ganglion (Moore, p 290).

72. (B) The suprarenal glands are located between 80. (B) The diaphragm is attached on each side to the superomedial aspects of the kidneys and the medial and lateral arcuate ligaments, which the diaphragm (Moore, p 285). are thickenings of the fascia covering the psoas major and quadratus lumborum muscles, 73. (A) The semilunar left suprarenal gland is rela- respectively (Moore, p 292). ted to the spleen, stomach, pancreas, and left crus of the diaphragm (Moore, p 285). 81. (B) The stomach, spleen, mesentery, and intes- tine may herniate into the thorax when there is 74. (D) The suprarenal cortex derives from meso- a traumatic diaphragmatic hernia (Moore, p 295). derm and secretes corticosteroids and andro- gens. These hormones cause the kidneys to 82. (D) The esophageal hiatus is an oval aperture retain sodium and water in response to stress for the esophagus in the muscle of the right crus (Moore, p 285). of the diaphragm at the level of the T10 verte- bra. The esophageal hiatus also transmits the 75. (C) The superior suprarenal arteries are pro- anterior and posterior vagal trunks, esophageal vided by the inferior phrenic. The middle supra- branches of the left gastric vessels, and a few renal arteries are provided by the abdominal lymphatics (Moore, p 294). aorta and the inferior suprarenal arteries are provided by the renal artery (Moore, p 287). 83. (C) The sympathetic trunks pass deep to the medial arcuate ligament. There are two small 76. (C) The diaphragm, the chief muscle of inspira- apertures in each crus of the diaphragm; one tion, descends during inspiration. Only its cen- transmits the greater and the other the lesser tral part moves because its periphery, as the splanchnic nerve (Moore, p 295). fixed origin of the muscle, attaches to the inferior margin of the thoracic cage and the superior 84. (D) The parasympathetic root of the celiac lumbar vertebrae (Moore, p 289). plexus is a branch of the posterior vagal trunk that contains fibers from the right and left vagal 77. (C) The central tendon of the diaphragm has nerves. The sympathetic roots of the plexus are no bony attachments and is incompletely di- the greater and lesser splanchnic nerves (Moore, vided into three leaves, resembling a wide p 3020). cloverleaf. Although it lies near the center of the diaphragm, the central tendon is closer to the 85. (A) The bifurcation of the aorta can be located anterior part of the thorax. The inferior vena on the approximately 2.5 cm cava perforates the central tendon. The aortic superior to the transpyloric plane to a point hiatus is formed by the right and left muscular slightly inferior to and to the left of the umbili- crura (Moore, p 292). cus. Bifurcation is also indicated just to the left of the midpoint of a line joining the highest points of the iliac crests (Moore, p 305). 0523-04 C hap 4 07/15/02 15:32 Page 65

Answers and Explanations: 70–98 65

86. (C) The inferior vena cava begins anterior to left lobe is separated by the attachment of the the L5 vertebra by the union of the common falciform ligament (Moore, p 265). iliac veins. The union occurs approximately 2.5 cm to the right of the median plane, inferior 92. (B) The splenic artery is the largest branch of to the bifurcation of the aorta and posterior to the celiac trunk. It follows a tortuous course the proximal part of the right common iliac posterior to the omental bursa, anterior to the artery (Moore, p 307). left kidney, and along the superior border of the pancreas. It divides into five or more 87. (E) In a small proportion of individuals, the branches that enter the hilum of the spleen inferior end of the thoracic duct—which begins (Moore, p 257). with the convergence of the main lymphatic ducts of the abdomen—takes the form of the 93. (A) For several weeks the rapidly growing commonly depicted, thin-walled sac or dilation midgut, supplied by the superior mesenteric known as the cisterna chyli (Moore, p 308). artery, is physiologically herniated into the proximal part of the umbilical cord. It is at- 88. (C) The pelvic splanchnic nerves are distinct tached to the yolk sac by the yolk stalk. As it from other splanchnic nerves in that they have returns to the abdominal cavity, the midgut nothing to do with the sympathetic trunks and rotates 270 degrees around the axis of the are derived directly from ventral rami of spinal superior mesenteric artery (Moore, p 246). nerves S2 through S4. They also convey pre- synaptic parasympathetic fibers to the inferior 94. (B) The appendicular artery is located in the hypogastric plexus (Moore, p 302). mesoappendix and the anterior cecal artery is located in the superior ileocecal fold. Both the 89. (C) The psoas muscle is a long, thick, fusi- appendix and cecum are vascular. The inferior form muscle that lies lateral to the lumbar ver- ileocecal fold is avascular (Moore, p 238). tebrae. Psoas is a Greek word meaning “muscle of the loin.” The muscle passes inferolaterally, 95. (C) The inferior or horizontal (3rd) part of the deep to the inguinal ligament to reach the lesser duodenum runs transversally to the left, pass- trochanter of the femur. The of ing over the inferior vena cava, aorta, and L3 nerves is embedded in the posterior part of the vertebra. It is crossed by the superior mesenteric psoas (Moore, p 299). artery and vein and the root of the mesentery (Moore, p 237). 90. (C) The suprarenal medulla is derived from neural crest cells associated with the sympa- 96. (D) The left gastro-omental artery arises from thetic nervous system. The chromaffin cells of the splenic artery and courses along the the medulla are related to sympathetic (post- greater curvature to anastomose with the right ganglionic) neurons in both derivation (neural gastro-omental artery (Moore, p 229). crest cells) and function. These cells secrete catecholamines (mostly epinephrine) into 97. (E) The smooth surface of the gastric mucosa, the bloodstream in response to signals from mucous layer of the stomach, is thrown into presynaptic neurons (Moore, p 286). longitudinal ridges when contracted. These are the gastric folds or rugae and most obvious 91. (C) In the current terminology, the left liver along the greater curvature of the stomach includes the caudate lobe and most of the (Moore, p 227). quadrate lobe. The anatomical left lobe is sepa- rated from these lobes on the visceral surface by 98. (C) The esophagus is a muscular tube (approx- the fissure for the round ligament of the liver imately 25 cm long) that extends from the phar- and the fissure for the ligamentum venosum; ynx to the stomach. It normally has four con- on the diaphragmatic surface the anatomical strictions and is crossed by the arch of the aorta and the left main bronchus (Moore, p 221). 0523-04 C hap 4 07/15/02 15:32 Page 66

66 4: The Abdomen

99. (C) Digestion occurs mainly in the stomach 113. right crus of diaphragm and duodenum. Peristalsis—ring-like, contrac- tion waves that begin around the middle of 114. aortic hiatus the stomach and move slowly toward the pylorus—is responsible for mixing the masti- 115. quadratus lumborum cated food mass with gastric juices and for emptying the contents of the stomach into the 116. internal abdominal oblique duodenum (Moore, p 218). 117. superior epigastric artery 100. (D) Most reabsorption occurs in the ascend- ing colon (Moore, p 218). 118. rectus abdominis

101. falciform ligament 119. transversus abdominis

102. lesser omentum 120. inferior epigastric artery

103. left gastric artery 121. transversus abdominis aponeurosis

104. celiac trunk 122. external abdominal oblique aponeurosis

105. gastrolienal ligament 123. transversalis fascia

106. splenic artery 124. rectus abdominis

107. gastrocolic ligament 125. internal abdominal oblique aponeurosis

108. common hepatic artery 126. subcostal nerve

109. pylorus 127. iliohypogastric nerve

110. proper hepatic artery 128. lateral femoral cutaneous nerve

111. caval opening 129. femoral nerve

112. esophageal hiatus 130. genitofemoral nerve 0523-05 C hap 5 07/15/02 15:33 Page 67

CHAPTER 5 The Pelvis and Perineum Questions

DIRECTIONS (Questions 1 through 100): Each of 4. The lateral part of the superior ramus of the the numbered items or incomplete statements in this pubis forms which of the following structures? section is followed by answers or by completions of (A) iliopubic eminence the statement. Select the ONE lettered answer or completion that is BEST in each case. (B) pubic tubercle (C) pecten pubis 1. All of the following statements concerning the (D) anterior inferior iliac spine pelvic cavity are correct EXCEPT (E) acetabulum (A) It is bounded posteriorly by the . 5. All of the following structures provide bound- (B) It is bounded anteriorly by the pubic aries for the pelvic inlet EXCEPT symphysis. (C) The pelvic inlet forms the superior (A) inferior ramus of the pubis boundary. (B) sacral promontory (D) The pelvic outlet forms the inferior (C) anterior border of the ala of the sacrum boundary. (D) arcuate line of the ilium (E) The musculofascial pelvic diaphragm (E) pecten pubis closes the pelvic inlet. 6. When a person is in the anatomical position, 2. All of the following bones contribute to the which of the following structures lie in the formation of the pelvic cavity EXCEPT same vertical plane? (A) ischium (A) sacral promontory and pubic tubercles (B) pelvis (B) anterior superior iliac spines and the (C) pubis anterior aspect of the (D) coccyx (C) posterior superior iliac spines and the (E) sacrum posterior aspect of the (D) ischial spines and the posterior border of 3. Which of the following structures is located the obturator foramen between the ischial spine and the ischial (E) superior pubic rami and the greater sci- tuberosity? atic notch (A) obturator foramen (B) lesser sciatic notch (C) acetabular notch (D) pubic arch (E) arcuate line

67 0523-05 C hap 5 07/15/02 15:33 Page 68

68 5: The Pelvis and Perineum

7. Weak areas of the pelvis include all of the fol- (D) It leaves the pelvis through the greater lowing EXCEPT sciatic foramen inferior to the piriformis and coccygeus muscles. (A) ischial tuberosities (E) It is the chief sensory nerve of the exter- (B) alae of the ilium nal genitalia. (C) pubic rami (D) sacroiliac joint 12. Which of the following nerves exits the pelvis (E) acetabula through the greater sciatic foramen, superior to the piriformis? 8. The pelvic floor is formed by all of the follow- (A) sciatic ing muscles EXCEPT (B) pudendal (A) pubococcygeus (C) superior gluteal (B) coccygeus (D) (C) piriformis (E) obturator (D) puborectalis (E) iliococcygeus 13. Which of the following statements concerning the sacral sympathetic trunks is correct? 9. Which of the following muscles covers and (A) Usually has four sympathetic ganglia. pads the lateral pelvic wall? (B) Descends on the pelvic surface of the (A) obturator internus ischium. (B) piriformis (C) Ends as the dorsal nerve of the penis or (C) pubococcygeus clitoris. (D) iliococcygeus (D) Passes through the obturator canal. (E) ischiococcygeus (E) Pierces the .

10. All of the following statements concerning the 14. The inferior hypogastric plexus receives fibers sciatic nerve are correct EXCEPT from which of the following nerves? (A) It is the largest and broadest nerve in the (A) pudendal and obturator body. (B) sciatic and superior gluteal (B) It is formed by the dorsal rami of L4 to S3. (C) inferior gluteal and lumbosacral (C) It passes through the greater sciatic (D) pelvic splanchnic and hypogastric foramen. (E) sacral sympathetic and obturator (D) It exits the pelvis inferior to the piriformis muscle. 15. All of the following arteries enter the true pelvis (E) It is one of the two main nerves of the EXCEPT . (A) internal iliac 11. All of the following statements concerning the (B) median sacral are correct EXCEPT (C) superior rectal (D) ovarian (A) It is derived from the anterior divisions of the ventral rami of S2 through S4. (E) testicular (B) It accompanies the internal pudendal 16. Which of the following arteries is considered artery. to be the artery of the pelvis? (C) It leaves the pelvis through the greater sciatic foramen. (A) obturator (B) pudendal 0523-05 C hap 5 07/15/02 15:33 Page 69

Questions: 7–25 69

(C) uterine (D) spongy (D) internal iliac (E) external urethral meatus (E) sacral 22. Which of the following structures opens into 17. Which of the following arteries arises from the the prostatic sinus? anterior division of the internal iliac? (A) prostatic utricle (A) superior rectal (B) ejaculatory ducts (B) iliolumbar (C) prostatic ductules (C) superior gluteal (D) (D) gonadal (E) bulbourethral glands (E) obturator 23. The paraurethral glands open into which of 18. All of the following statements concerning the the following structures? are correct EXCEPT (A) near the external urethral orifice (A) They are retroperitoneal. (B) in the neck of the bladder (B) They cross the pelvic brim anterior to the (C) in the prostatic utricle bifurcation of the common iliac arteries. (D) in the seminal colliculus (C) They are fibrous tubes connecting the (E) in the ejaculatory ducts kidneys to the urinary bladder. (D) Their superior halves lie in the abdomen 24. All of the following statements concerning the and the inferior halves lie in the pelvis. ductus deferens are correct EXCEPT (E) Their oblique passage through the blad- (A) It begins in the head of the epididymis. der wall forms a one-way “flap valve.” (B) It ascends in the spermatic cord. 19. The uvula of the bladder is located in which of (C) It passes through the inguinal canal. the following locations? (D) It joins the duct of the seminal vesicle to form the ejaculatory duct. (A) retropubic space (E) It descends medial to the ureter and (B) trigone of the bladder seminal vesicle. (C) apex of the bladder (D) fundus of the bladder 25. All of the following statements concerning the (E) pubovesical ligament seminal vesicles are correct EXCEPT (A) The peritoneum of the rectovesical 20. Parasympathetic fibers to the bladder are de- pouch separates the superior ends of the rived from which of the following nerves? seminal vesicles from the rectum. (A) pelvic splanchnic (B) They do not store sperm. (B) greater splanchnic (C) They are elongated structures that lie (C) T11-L2 between the fundus of the bladder and (D) superior hypogastric plexus the rectum. (E) sacral plexus (D) The inferior ends of the seminal vesicles are separated from the rectum by the 21. Which of the following parts of the male ure- rectovesical septum. thra is the widest and most dilatable? (E) Bulbourethral glands empty into the seminal vesicles. (A) preprostatic (B) prostatic (C) membranous 0523-05 C hap 5 07/15/02 15:33 Page 70

70 5: The Pelvis and Perineum

26. The posterior fornix is the deepest part of which 31. The oocytes expelled from the usually of the following structures? are fertilized in which of the following areas of the uterine tubes? (A) urinary bladder (B) (A) infundibulum (C) deep perineal space (B) ampulla (D) (C) isthmus (E) (D) uterine (E) fimbria 27. All of the following statements concerning the uterus are correct EXCEPT 32. The distal end of the ovary connects to the lat- eral wall of the pelvis by which of the follow- (A) It is anteverted. ing structures? (B) It is anteflexed. (C) The position of the uterus is fixed. (A) round ligament (D) It is divisible into two main parts. (B) ligament of the ovary (E) The body of the uterus lies between the (C) suspensory ligament of the ovary layers of the broad ligament. (D) transverse cervical ligament (E) lateral cervical ligament 28. Which of the following parts of the uterus pro- trudes into the uppermost vagina? 33. Which of the following nerves provides para- sympathetic innervation to the ovaries? (A) round ligament (B) body (A) pudendal (C) fundus (B) pelvic splanchnic (D) isthmus (C) vagus (E) cervix (D) obturator (E) superior hypogastric 29. The rounded vaginal part of the cervix ex- tends into the vagina and communicates with 34. The rectosigmoid junction lies anterior to it through which of the following structures? which of the following structures? (A) uterine tubes (A) S3 vertebra (B) external os (B) prostate (C) ureter (C) obturator foramen (D) urethra (D) bladder (E) internal os (E) pararectal fossae

30. Laterally, the peritoneum of the broad liga- 35. All of the following osseofibrous structures ment is prolonged superiorly over the ovarian mark the boundaries of the perineum EXCEPT vessels as which of the following structures? (A) pubic symphysis (A) mesovarium (B) inferior pubic rami (B) mesosalpinx (C) (C) mesometrium (D) ischial tuberosities (D) suspensory ligament of the ovary (E) ischial rami (E) transverse cervical (cardinal) ligaments 36. The perineum is divided into two triangles by drawing a transverse line between which of the following structures? 0523-05 C hap 5 07/15/02 15:33 Page 71

Questions: 26–45 71

(A) anterior ends of the ischial tuberosities (D) (B) coccyx to pubic tubercles (E) muscles of the deep and superficial peri- (C) inferior iliac spines to pubic symphysis neal pouch (D) medial ends of inguinal ligament to tip of coccyx 42. The anal columns contain which of the follow- ing structures? (E) sacrum to pubic symphysis (A) internal pudendal vessels 37. The perineal body is the site of convergence of (B) pudendal nerve all of the following muscles EXCEPT (C) superior rectal vessels (A) ischiocavernous (D) obturator nerve (B) bulbospongiosus (E) inferior rectal vessels (C) superficial transverse perineal (D) deep transverse perineal 43. All of the following statements concerning the pectinate line are correct EXCEPT (E) (A) It is the junction between the superior 38. In males, the superficial perineal pouch con- and inferior parts of the . tains which of the following structures? (B) The superior rectal artery supplies the (A) prostate superior part of the anal canal. (B) seminal vesicles (C) Lymphatic vessels from the superior part of the anal canal drain into the inter- (C) membranous urethra nal lymph nodes. (D) neck of the bladder (D) The superior part of the anal canal is (E) ischiocavernous muscle drained by the internal rectal venous plexus which drains into tributaries of 39. In females, the deep perineal pouch contains the caval venous system. which of the following structures? (E) The nerve supply to the anal canal supe- (A) clitoris rior to the pectinate line is somatic inner- (B) greater vestibular glands vation. (C) bulbourethral glands 44. The navicular fossa is located in which of the (D) bulbs of the vestibule following structures? (E) external urethral sphincter (A) glans penis 40. The pudendal canal is a space within which of (B) prostate the following structures? (C) bladder (A) the deep perineal space (D) seminal vesicle (B) the superficial perineal space (E) expanded in the clitoris (C) the obturator fascia 45. Which of the following nerves do NOT inner- (D) the broad ligament vate the scrotum? (E) the (A) ilioinguinal 41. The pudendal nerve innervates which of the (B) genitofemoral following structures? (C) pudendal (A) ovaries (D) posterior femoral cutaneous (B) testes (E) obturator (C) uterus 0523-05 C hap 5 07/15/02 15:33 Page 72

72 5: The Pelvis and Perineum

46. The deep arteries of the penis are located in 51. The paramesonephric ducts in the male degen- which of the following areas? erate except for which of the following struc- tures? (A) superficial to the tunica albuginea (B) superficial to the deep fascia (A) efferent ductules of the testis (C) within the corpus spongiosum (B) appendix testis (D) within the corpora cavernosa (C) ductus epididymis (E) prepuce (D) seminal vesicles (E) ductus deferens 47. Which of the following muscles surrounds the crura in the root of the penis? 52. The only parts remaining from the mesoneph- ric system in the female include which of the (A) superficial transverse perineal following structures? (B) deep transverse perineal (C) bulbospongiosus (A) fimbriae (D) ischiocavernosus (B) uterine tubes (E) cremaster (C) epoöphoron (D) uterus 48. The space between the labia minora, the ves- (E) cervix tibule, contains all of the following structures EXCEPT 53. The vaginal fornices are derived from which of the following structures? (A) urethral opening (B) opening of the vagina (A) paramesonephric duct (C) ducts of the greater vestibular glands (B) mesonephric ducts (D) ducts of the lesser vestibular glands (C) ectodermal duct (E) bulbs of the vestibule (D) sinovaginal bulbs (E) urogenital sinus 49. Each of the following statements relating to the labia minora is correct EXCEPT 54. Rapid elongation of the genital tubercle in the male gives rise to which of the following (A) They are folds of fat. structures? (B) They are hairless. (C) They are enclosed in the pudendal cleft (A) testes within the . (B) scrotum (D) They have a core of spongy connective (C) ductus deferens tissue containing erectile tissue. (D) phallus (E) They contain many sebaceous glands (E) epididymis and sensory nerve endings. 55. The scrotum arises from which of the following 50. Parasympathetic stimulation in the female pro- structures? duces which of the following? (A) mesonephric ducts (A) decreased vaginal secretions (B) paramesonephric ducts (B) engorgement of erectile tissue in the (C) urethral folds bulbs of the vestibule (D) cloacal fold (C) engorgement of the greater vestibular (E) genital swellings gland (D) decrease in size of the clitoris 56. The clitoris is derived from which of the fol- (E) the clitoris becomes flaccid lowing structures? 0523-05 C hap 5 07/15/02 15:33 Page 73

Questions: 46–66 73

(A) urethral folds 62. The median umbilical ligament contains which (B) genital swellings of the following structures? (C) urogenital groove (A) inferior epigastric vessels (D) mesonephric ducts (B) obturator vessels (E) genital tubercle (C) remnant of the urachus (D) uterine tubes 57. Which of the following structures give rise to (E) ovarian vessels the labia minora? (A) mesonephric ducts 63. The inferior ends of the seminal vesicles are (B) urogenital groove closely related to the rectum and are sepa- (C) genital swellings rated from it only by which of the following structures? (D) urethral folds (E) genital tubercle (A) pubovesical ligament (B) puboprostatic ligament 58. Which of the following structures extends into (C) rectovesical septum the labia majora? (D) puborectalis muscle (A) suspensory ligament of the ovary (E) (B) ligament of the ovary proper (C) processus vaginalis 64. Which of the following structures is the larg- est accessory gland of the male reproductive (D) round ligament of the uterus system? (E) uterine tube (A) testes 59. Which of the following terms correctly applies (B) seminal vesicles to the pelvis of the normal female? (C) prostate (A) spondyloid (D) bulbourethral glands (B) android (E) epididymis (C) gynecoid 65. All of the following muscles compress the va- (D) anthropoid gina and act like sphincters EXCEPT (E) platypelloid (A) pubovaginalis 60. Persons with spondylolysis have a defect in (B) external urethral sphincter which of the following structures? (C) urethrovaginal sphincter (A) vertebral arch (D) bulbospongiosus (B) body of L5 vertebra (E) pubococcygeus (C) sacrum 66. The mesosalpinx forms the mesentery for which (D) vertebral canal of the following structures? (E) vertebral body (A) uterine tube 61. The neck of the bladder in females is held (B) ovary firmly by which of the following structures? (C) small intestine (A) puboprostatic ligaments (D) bladder (B) puborectalis muscle (E) uterus (C) levator ani muscle (D) coccygeus muscle (E) pubovesical ligaments 0523-05 C hap 5 07/15/02 15:33 Page 74

74 5: The Pelvis and Perineum

67. The ovarian arteries arise from which of the (C) corpora cavernosa following arteries? (D) spermatic cord (A) superior gluteal (E) bulbospongiosum (B) inferior gluteal 73. The term is synonymous with which of (C) abdominal aorta the following structures? (D) superior rectal (E) internal pudendal (A) mons pubis (B) labia majora 68. Which of the following structures provides (C) vestibule support for the ampulla of the rectum? (D) clitoris (A) urogenital diaphragm (E) pudendum (B) puboprostatic ligament 74. Which of the following structures may be re- (C) sacrogenital ligament ferred to as the fourchette? (D) tendinous arch of pelvic fascia (E) levator ani and anococcygeal ligament (A) prepuce of the clitoris (B) frenulum of the labia minora 69. In males the rectum is related anteriorly to all (C) vestibule of the vagina of the following structures EXCEPT (D) glans clitoris (A) fundus of the urinary bladder (E) mons pubis (B) proximal parts of the ureters 75. The lesser vestibular glands open into which (C) ductus deferens of the following structures? (D) seminal vesicles (E) prostate (A) vestibule between the urethral and vagi- nal orifices 70. Some obstetricians apply the term perineum to (B) vestibule on each side of the vaginal a more restricted region that extends between orifice which of the following structures? (C) bulbs of the vestibule (A) perineal body and the mons pubis (D) glans clitoris (B) vagina and anus (E) bulbourethral ducts (C) pubic arch and the rectum 76. Which of the following structures is incised (D) ischial spines and pubic tubercles during median episiotomy for childbirth? (E) vestibule and sacrum (A) perineal body 71. Distally the corpus spongiosum expands to (B) greater vestibular glands form which of the following structures? (C) lesser vestibular glands (A) bulb of the penis (D) clitoris (B) clitoris (E) urethra (C) vestibule 77. Which of the following arteries is a direct con- (D) glans penis tinuation of the inferior mesenteric artery? (E) crura of the penis (A) superior rectal 72. Helicine arteries are located in which of the (B) middle rectal following structures? (C) inferior rectal (A) superficial perineal space (D) iliolumbar (B) deep perineal space (E) lateral sacral 0523-05 C hap 5 07/15/02 15:33 Page 75

Questions: 67–88 75

78. The superior gluteal artery leaves the pelvis (C) posterior lobe through which of the following openings? (D) lateral lobes (A) greater sciatic foramen (E) middle lobe (B) lesser sciatic foramen 84. The relationship (“water passing under the (C) obturator canal bridge”) is an especially important one for sur- (D) pudendal canal geons ligating which of the following arteries? (E) sacral foramina (A) ovarian 79. Which of the following structures separates (B) testicular the bladder from the pubic bones in females? (C) uterine (A) rectouterine fold (D) vaginal (B) vesicouterine pouch (E) obturator (C) trigone of the bladder 85. Immediately superior to the perineal membrane (D) median umbilical fold is located which of the following muscles? (E) retropubic space (A) obturator internus 80. The membranous part of the male urethra is (B) levator ani located in which of the following structures? (C) bulbocavernosus (A) bladder (D) ischiocavernous (B) prostate (E) deep transverse perineal (C) external urethral sphincter 86. The navicular fossa is located in which of the (D) bulb of penis following structures? (E) glans penis (A) bulb of the penis 81. The paraurethral glands are homologues to (B) prostate which of the following structures? (C) membranous urethra (A) testes (D) glans penis (B) prostate (E) neck of bladder (C) seminal vesicles 87. All of the following nerves provide innerva- (D) bulbourethral glands tion to the scrotum EXCEPT (E) epididymis (A) obturator 82. Lithotripsy uses shock waves to break up which (B) ilioinguinal of the following structures? (C) genitofemoral (A) blood clots (D) pudendal (B) foreign bodies (E) posterior femoral cutaneous (C) small tumors 88. All of the following structures surround the (D) urinary calculi corpora cavernosa and corpus spongiosum (E) abscesses EXCEPT

83. Which of the following areas contributes to the (A) loose areolar tissue major part of the prostate? (B) deep fascia (C) tunica albuginea (A) anterior lobe (D) skin (B) isthmus (E) tunica dartos 0523-05 C hap 5 07/15/02 15:33 Page 76

76 5: The Pelvis and Perineum

89. Which of the following arteries gives rise to (C) puborectalis the deferential artery? (D) piriformis (A) internal pudendal (E) coccygeus (B) testicular 95. The pelvic splanchnic nerves contain which of (C) inferior vesicle the following? (D) obturator (E) umbilical (A) somatic afferents (B) sympathetic fibers 90. Which of the following structures is located at (C) parasympathetic fibers the free anterior borders of the levator ani? (D) special visceral efferents (A) coccygeus muscle (E) special somatic afferents (B) piriformis muscle 96. The internal iliac artery is crossed by which of (C) urogenital hiatus the following structures at the level of the 4th (D) obturator internus muscle intervertebral disc between L5 and S1? (E) rectum (A) puborectalis muscle 91. Which of the following structures form a (B) ureter U-shaped sling around the anorectal junction? (C) obturator nerve (A) sacrospinous ligament (D) umbilical artery (B) anococcygeal ligament (E) tendinous arch of the levator ani (C) superficial transverse perineal muscle 97. The superior end of the vagina surrounds (D) puborectalis muscle which of the following structures? (E) sacrotuberous ligament (A) round ligament 92. Which of the following muscles is the larger (B) urogenital hiatus part and most important muscle in the pelvic (C) urethra floor? (D) cervix (A) coccygeus (E) neck of bladder (B) obturator internus 98. Which of the following nerves innervates the (C) piriformis lower one fourth of the vagina? (D) deep transverse perineal muscle (E) levator ani (A) pelvic splanchnics (B) lumbar splanchnics 93. Which of the following muscles passes through (C) pudendal the lesser sciatic foramen? (D) superior hypogastric plexus (A) obturator internus (E) inferior hypogastric plexus (B) piriformis 99. Which of the following structures forms the (C) puborectalis mesentery of the uterus? (D) pubococcygeus (E) iliococcygeus (A) round ligament of the uterus (B) mesosalpinx 94. Which of the following muscles leaves the lesser (C) pelvic diaphragm pelvis through the greater sciatic foramen? (D) endopelvic fascia (A) pubococcygeus (E) mesometrium (B) iliococcygeus 0523-05 C hap 5 07/15/02 15:33 Page 77

Questions: 89–115 77

100. The posterior part of the tendinous arch of (B) pubovesicle pelvic fascia forms which of the following (C) transverse cervical ligaments? (D) sacrogenital (A) puboprostatic (E) sacrospinous

DIRECTIONS (Questions 101 through 105): Identify DIRECTIONS (Questions 106 through 110): Identify the anatomical features indicated on the art below. the anatomical features indicated on the art below.

DIRECTIONS (Questions 111 through 115): Identify the anatomical features indicated on the art below. 0523-05 C hap 5 07/15/02 15:33 Page 78

78 5: The Pelvis and Perineum

DIRECTIONS (Questions 116 through 120): Identify the anatomical features indicated on the art below.

DIRECTIONS (Questions 121 through 125): Identify the anatomical features indicated on the art below. 0523-05 C hap 5 07/15/02 15:33 Page 79

Answers and Explanations

1. (E) The superior boundary of the pelvic cavity 7. (A) Weak areas of the pelvis include the pubic is the pelvic inlet. The pelvis is limited inferi- rami, acetabula, sacroiliac joint, and alae of the orly by the pelvic outlet, which is closed by the ilium (Moore, p 338). musculofascial pelvic diaphragm and bounded posteriorly by the coccyx and anteriorly by the 8. (C) The piriformis muscles cover the postero- pubic symphysis (Moore, p 333). lateral wall of the pelvis. The is formed by the funnel-shaped pelvic diaphragm, 2. (D) The pelvic girdle is formed by the hip (is- which consists of the levator ani and coccygeus. chium, pubis, and ilium) bones and the sac- The levator ani includes the pubococcygeus, rum (Moore, p 333). puborectalis, and iliococcygeus (Moore, p 342).

3. (B) The concavity between the ischial spine 9. (A) The bony framework of the lateral pelvic and the ischial tuberosity is the lesser sciatic walls is formed by the hip bones and the obtu- notch. The larger concavity, the greater sciatic rator foramen, which is closed by the obturator notch, is superior to the ischial spine and is membrane. The obturator internus muscles formed in part by the ilium (Moore, p 334). cover and thus pad most of the lateral pelvic walls (Moore, p 341). 4. (C) The pubis is an angulated bone with a superior ramus that helps to form the acetabu- 10. (B) The two main nerves of the sacral plexus lum and an inferior ramus that helps to form include the sciatic and pudendal. The sciatic the obturator foramen. A thickening on the an- nerve is the largest and broadest nerve in the terior part of the body of the pubis is the pubic body. It is formed by the ventral rami of L4-S3, crest, which ends laterally as a prominent which converge on the anterior surface of the bump, the pubic tubercle. The lateral part of the piriformis. The sciatic nerve usually passes superior ramus has an oblique ridge known as through the greater sciatic foramen, inferior the pecten pubis or pectineal line of the pubis to the piriformis, to enter the gluteal region (Moore, p 335). (Moore, p 347).

5. (A) The pelvic inlet is bounded by the superior 11. (D) The pudendal nerve is derived from the margin of the pubic symphysis anteriorly, pos- anterior divisions of the ventral rami of S2 terior border of the pubic crest, pecten pubis, through S4. It accompanies the internal puden- arcuate line of the ilium, anterior border of the dal artery and leaves the pelvis through the ala of the sacrum, and the sacral promontory greater sciatic foramen between the piriformis (Moore, p 336). and coccygeus muscles. It is the main nerve of the perineum and the chief sensory nerve of the 6. (B) When a person is in the anatomical posi- external genitalia (Moore, p 347). tion, the anterior superior iliac spines and the anterior aspect of the pubic symphysis lie in the 12. (C) The superior gluteal nerve arises from the same vertical plane (Moore, p 337). posterior divisions of the ventral rami of L4

79 0523-05 C hap 5 07/15/02 15:33 Page 80

80 5: The Pelvis and Perineum

through S1 and leaves the pelvis through the 20. (A) Parasympathetic fibers to the bladder are greater sciatic foramen, superior to the piri- derived from the pelvic splanchnic nerves. They formis (Moore, p 347). are motor to the detrusor muscle and inhibitory to the internal sphincter. Sympathetic fibers to 13. (A) The sacral sympathetic trunks descend the bladder are derived from T11 through L2 posterior to the rectum in the extraperitoneal (Moore, p 362). connective tissue and send gray rami commu- nicantes to each of the ventral rami of the sacral 21. (B) The prostatic urethra is the widest and most and coccygeal nerves. Each of the sacral trunks dilatable part of the male urethra (Moore, p 363). is smaller than the lumbar trunks and usually has four sympathetic ganglia (Moore, p 350). 22. (C) The internal surface of the posterior wall of the prostatic urethra has a median ridge known 14. (D) The pelvic splanchnic nerves merge with as the urethral crest. A groove on each side, the hypogastric nerves to form the inferior known as the prostatic sinus, receives the pros- hypogastric (and pelvic) plexuses. (Moore, tatic ductules. A rounded eminence located in p 350). the middle of the median ridge is known as the seminal colliculus. The prostatic utricle is an 15. (E) Four main arteries enter the lesser or true embryonic remnant of the uterus and part of the pelvis. The internal iliac and ovarian arteries vagina. The ejaculatory ducts open beside the are paired, and the median sacral and superior prostatic utricle (Moore, p 364). rectal arteries are unpaired. The testicular artery does not enter the true pelvis, as it fol- 23. (A) The paraurethral glands are homologues lows the testes through the inguinal canal into to the prostate. They have common paraure- the scrotum (Moore, p 350). thral ducts, which open, one on each side, near the external urethral orifice (Moore, p 364). 16. (D) The internal iliac artery is the artery of the pelvis; however, it does supply branches to 24. (A) The ductus deferens begins in the tail of the the , thighs, and the perineum (Moore, epididymis and ascends in the spermatic cord. p 350). It passes through the inguinal canal and enters the pelvis. It ends by joining the duct of the 17. (E) The superior gluteal and iliolumbar arter- seminal vesicle to form the ejaculatory duct. It ies arise from the posterior division of the inter- descends medial to the ureter and seminal vesi- nal iliac arteries. The gonadal arteries arise cle (Moore, p 367). from the abdominal aorta and the superior rec- tal arteries from the inferior mesenteric artery. 25. (E) Ducts of the bulbourethral glands empty The umbilical, obturator, uterine, vaginal, inter- into the urethra. The seminal vesicles are elon- nal pudendal, and inferior gluteal arteries arise gated structures that lie between the fundus of from the anterior division of the internal iliac the bladder and the rectum, and they do not arteries (Moore, pp 354–355). store sperm. They secrete a thick, alkaline fluid that mixes with the sperm as they pass into the 18. (C) The ureters are muscular tubes, 25 to 30 cm ejaculatory ducts to the urethra. The superior long, that connect the kidneys to the urinary ends of the seminal vesicles are covered with bladder. They are retroperitoneal, with their peritoneum and lie posterior to the ureters, superior halves in the abdomen and their infe- where the rectovesical pouch separates them rior halves in the pelvis. The pelvic part of the from the rectum. The inferior ends of the semi- ureters begins where it crosses the bifurcation nal vesicle are separated from the rectum by the of the common iliac artery (Moore, p 357). rectovesical septum (Moore, p 368).

19. (B) The uvula of the bladder is a slight projec- 26. (B) The posterior fornix is the deepest part of tion of the trigone of the bladder. It is usually the vagina and is closely related to the recto- more prominent in older men (Moore, p 362). uterine pouch. The vaginal fornix is the recess 0523-05 C hap 5 07/15/02 15:33 Page 81

Answers and Explanations: 13–41 81

around the cervix and is described as having 33. (C) The parasympathetic fibers in the ovarian anterior, posterior, and lateral parts (Moore, plexus are derived from the vagus nerve (Moore, pp 372–373). p 384).

27. (C) The uterus is a thick-walled, pear-shaped 34. (A) The rectosigmoid junction lies anterior to muscular organ whose body lies between the the S3 vertebra (Moore, p 384). layers of the broad ligament. In the adult, the uterus is usually anteverted and anteflexed; its 35. (C) The osseofibrous structures marking the position changes with the degree of fullness of boundaries of the perineum include the pubic the bladder and rectum (Moore, pp 373–374). symphysis, inferior pubic rami, ischial rami, ischial tuberosities, sacrotuberous ligaments, 28. (E) Only the cylindrical, narrow inferior part of inferiormost sacrum, and coccyx (Moore, p 389). the uterus known as the cervix protrudes into the uppermost vagina (Moore, p 376). 36. (A) A transverse line joining the anterior ends of the ischial tuberosities divides the perineum 29. (B) The rounded vaginal part of the cervix ex- into two triangles. The , containing tends into the vagina and communicates with it the anus, is posterior to this line. The urogen- through the external os. The cervical canal is ital triangle, containing the root of the scro- broadest at its middle part and communicates tum and penis in males and the external geni- with the uterine cavity through the internal talia in females is anterior to this line (Moore, os and with the vagina through the external pp 389–390). os (Moore, pp 376–377). 37. (A) The bulbospongiosus, external anal sphinc- 30. (D) Laterally, the peritoneum of the broad lig- ter, and superficial and deep transverse per- ament is prolonged superiorly over the vessels ineal muscles converge at the site of the perineal as the suspensory ligament of the ovary. The body (Moore, p 390). part of the broad ligament by which the ovary is suspended is the mesovarium. The part of the 38. (E) The following structures are found in the broad ligament forming the mesentery of the superficial perineal space: the root of the penis uterine tube is the mesosalpinx. The major part and its associated muscles, the ischiocavernous of the broad ligament, the mesentery of the ute- and bulbospongiosus. The proximal part of rus, or mesometrium, is below the mesosalpinx the spongy urethra, superficial transverse per- and mesovarium (Moore, p 381). ineal muscles, internal pudendal vessels, and branches of the pudendal nerve are also located 31. (B) The ampulla is the widest and longest part in the superficial perineal space (Moore, p 394). of the uterine tubes. It begins at the medial end of the infundibulum. Oocytes expelled from 39. (E) In females, the deep perineal pouch con- the ovaries are usually fertilized in the ampulla tains the proximal part of the urethra, the (Moore, p 383). external urethral sphincter muscle, the deep transverse perineal muscles, and related ves- 32. (C) The distal end of the ovary connects to the sels and nerves (Moore, p 394). lateral wall of the pelvis by the suspensory lig- ament of the ovary. The ligament conveys the 40. (C) The pudendal canal is a space within the ovarian vessels, lymphatics, and nerves to and obturator fascia, which covers the medial aspect from the ovary and constitutes the lateral part of the obturator internus and lines the lateral of the mesovarium of the broad ligament. The wall of the ischioanal fossa (Moore, p 395). ovary also attaches to the uterus by the liga- ment of the ovary (Moore, p 384). 41. (E) The pudendal nerve supplies most of the innervation to the perineum. Toward the distal end of the pudendal canal, the pudendal nerve 0523-05 C hap 5 07/15/02 15:33 Page 82

82 5: The Pelvis and Perineum

splits, giving rise to the perineal nerves and 46. (D) The arterial supply of the penis is mainly continuing as the dorsal nerve of the penis or from the branches of the internal pudendal clitoris (Moore, p 395). arteries. The deep arteries of the penis are the main vessels supplying the cavernous spaces 42. (C) The superior half of the anal canal con- in the erectile tissue of the corpora cavernosa tains mucous membrane that is characterized and are therefore involved in the erection of by a series of longitudinal ridges called anal the penis. The deep arteries pierce the crura columns. These columns contain the terminal and run within the corpora cavernosa (Moore, branches of the superior rectal artery and vein p 410). (Moore, p 395). 47. (D) The ischiocavernous muscles surround the 43. (D) The pectinate line indicates the junction of crura in the root of the penis. Each muscle arises the superior part of the anal canal and the infe- from the internal surface of the ischial tuberos- rior part. The anal canal superior to the pecti- ity and ischial ramus and passes anteriorly on nate line differs from the part inferior to the the crus of the penis, where it is inserted into pectinate line in its arterial supply, innervation, the sides and ventral surface of the crus and and venous and lymphatic drainage. The supe- the perineal membrane. The ischiocavernous rior rectal artery supplies the superior part of muscles force blood from the cavernous spaces the anal canal and the inferior rectal arteries in the crura into the distal parts of the corpora supply the inferior part. Superior to the pecti- cavernosa, thus increasing the turgidity of the nate line, the internal rectal venous plexus penis (Moore, p 409). drains chiefly into the superior rectal vein—a tributary of the inferior mesenteric vein and the 48. (E) The vestibule is the space between the labia portal system. Inferior to the pectinate line the minora containing the openings of the urethra, internal rectal plexus drains into the inferior vagina, and ducts of the greater and lesser rectal veins—tributaries to the caval venous sys- vestibular glands. The greater vestibular glands tem. Superior to the pectinate line, the lymphat- are round or oval and are partly overlapped ics drain into the internal iliac lymph nodes into posteriorly by bulbs of the vestibule and, like the common iliac and lumbar nodes. Inferior to the bulbs, are partially surrounded by the bul- the pectinate line, the lymphatics drain into the bospongiosus muscles (Moore, p 413). superficial inguinal lymph nodes. Superior to the pectinate line, the innervation is visceral; 49. (A) The labia minora are folds of fat-free, hair- inferior to the pectinate line, the innervation is less skin. They are enclosed in the pudendal somatic (Moore, pp 398–400). cleft within the labia majora, immediately sur- rounding the vestibule of the vagina. They have 44. (A) The lumen of the spongy urethra is approx- a core of spongy connective tissue containing imately 5 mm in diameter and expanded in the erectile tissue and many small blood vessels bulb of the penis to form the intrabulbar fossa (Moore, p 413). and in the glans penis to form the fossa navicu- laris (Moore, p 403). 50. (B) Parasympathetic stimulation in the female produces an increase in vaginal secretions, erec- 45. (E) The anterior aspect of the scrotum is sup- tion of the clitoris, and engorgement of erectile plied by anterior scrotal nerves derived from tissue in the bulbs of the vestibule (Moore, p 415). the ilioinguinal nerve and by the genital branch of the genitofemoral nerve. The posterior as- 51. (B) Except for the most cranial portion, the pect of the scrotum is supplied by posterior appendix epididymis, the mesonephric ducts scrotal nerves, branches of the superficial per- persist and form the main genital ducts. Imme- ineal nerves, and the perineal branch of the pos- diately below the entrance of the efferent duc- terior femoral cutaneous nerve (Moore, p 405). tules, the mesonephric ducts elongate and become highly convoluted, forming the ductus 0523-05 C hap 5 07/15/02 15:33 Page 83

Answers and Explanations: 42–69 83

epididymis. From the tail of the epididymis coid pelvis is the normal female type of pelvis to the outbudding of the seminal vesicle, the (Moore, p 337). mesonephric ducts obtain a thick muscular coat and form the ductus deferens. The region of 60. (A) Persons with spondylolysis have a defect the ducts beyond the seminal vesicles is the in the vertebral arch. When this is bilateral, it ejaculatory duct. Except for a small portion at results in the L5 vertebra being divided into their cranial ends, the appendix testis, the para- two pieces. If the parts separate, the abnor- mesonephric ducts in the male degenerate mality is spondylolisthesis, which is anterior (Sadler, p 326). displacement of the body of the L5 vertebra on the sacrum (Moore, p 339). 52. (C) The only parts remaining from the meso- nephric system are the epoöphoron, paro- 61. (E) The bladder is relatively free within the öphoron, and Gartner’s cyst (Sadler, p 325). extraperitoneal subcutaneous fatty tissue ex- cept for its neck, which is held firmly by the 53. (A) The winglike expansions of the vagina puboprostatic ligaments in males and the pubo- around the end of the uterus, the vaginal for- vesical ligaments in females (Moore, p 359). nices, are of paramesonephric origin (Sadler, p 329). 62. (C) The median umbilical ligament is formed by the remnant of the urachus (Moore, p 361). 54. (D) The rapid elongation of the genital tuber- cle in the male gives rise to the phallus (Sadler, 63. (C) The inferior ends of the seminal vesicles p 331). are closely related to the rectum and are sepa- rated from it only by the rectovesical septum, 55. (E) The genital swellings, known in the male a membranous partition (Moore, p 368). as the scrotal swellings, arise in the inguinal region. With further development, they move 64. (C) The prostate is the largest accessory gland caudally, and each swelling then makes up half of the male (Moore, p 369). of the scrotum (Sadler, p 332). 65. (E) The pubovaginalis, external urethral 56. (E) In females, the genital tubercle elongates sphincter, urethrovaginal sphincter, and bul- only slightly and forms the clitoris (Sadler, bospongiosus compress the vagina and act like p 335). sphincters (Moore, p 372).

57. (D) The urethral folds do not fuse, as in the 66. (A) The part of the broad ligament forming male, but develop into the labia minora (Sadler, the mesentery of the uterine tube is the meso- p 336). salpinx (Moore, p 377).

58. (D) Descent of the gonads is considerably less 67. (C) The ovarian arteries arise from the abdom- in the female than in the male, and the ovaries inal aorta (Moore, p 384). finally settle just below the rim of the true pel- vis. The cranial genital ligament forms the sus- 68. (E) The dilated terminal part of the rectum, pensory ligament of the ovary, whereas the lying directly above and supported by the pel- caudal genital ligament forms the ligament of vic diaphragm (levator ani) and anococcygeal the ovary proper and the round ligament of the ligament is the ampulla of the rectum (Moore, uterus. The latter extends into the labia majora p 385). (Sadler, p 342). 69. (B) In males the rectum is related anteriorly 59. (C) Android and anthropoid pelves are com- to the fundus of the urinary bladder, terminal mon in males. The platypelloid pelvis is un- parts of the ureters, ductus deferens, seminal common in both males and females. The gyne- vesicles, and prostate (Moore, p 385). 0523-05 C hap 5 07/15/02 15:33 Page 84

84 5: The Pelvis and Perineum

70. (B) Some obstetricians apply the term per- sciatic foramen, superior to the piriformis mus- ineum to a more restricted region that includes cle, to supply the gluteal muscles in the but- the area between the vagina and anus (Moore, tocks (Moore, p 355). p 389). 79. (E) When empty, the adult male or female uri- 71. (D) Distally the corpus spongiosum expands nary bladder is in the lesser pelvis, lying pos- to form the conical glans penis (Moore, p 407). terior and slightly superior to the pubic bones. It is separated from these bones by the poten- 72. (C) The deep arteries of the penis are the main tial retropubic space and lies inferior to the vessels supplying the cavernous spaces in the peritoneum, where it rests on the pelvic floor erectile tissue of the corpora cavernosa and are (Moore, p 359). therefore involved in the erection of the penis. They give off numerous branches that open 80. (C) The intermediate part of the urethra (mem- directly into the cavernous spaces. When the branous part) is the section passing through the penis is flaccid, these arteries are coiled and external urethral sphincter and the perineal therefore are called helicine arteries (Moore, membrane. The short intermediate part, extend- p 410). ing from the prostatic urethra to the spongy ure- thra, is the narrowest and least distensible part 73. (E) The synonymous terms vulva and puden- of the urethra (Moore, p 364). dum include the clitoris, vestibule of the vagina, bulbs of vestibule, and greater vestibular gland 81. (B) Urethral glands are present particularly (Moore, p 413). in the superior part of the female urethra. The paraurethral glands are homologues to the 74. (B) In young women, especially virgins, the prostate (Moore, p 364). labia minora are connected by a small fold known as the frenulum of the labia minora or 82. (D) Lithotripsy uses shock waves to break up the fourchette (Moore, p 413). a stone into small fragments that can be passed in the urine (Moore, p 358). 75. (A) The slender ducts of the greater vestibular glands pass deep to the bulbs of the vestibule 83. (D) The lateral lobes on either side of the ure- and open into the vestibule on each side of the thra form the major part of the prostate (Moore, vaginal orifice. The lesser vestibular glands are p 369). small glands on each side of the vestibule that open into it between the urethral and vaginal 84. (C) In its uppermost portion, at the base of the orifices (Moore, p 414). peritoneal broad ligament, the uterine artery runs transversely toward the cervix while the 76. (A) The perineal body is the major structure ureters pass immediately beneath them as they incised during median episiotomy for child- pass on each side of the cervix toward the blad- birth (Moore, p 391). der. This relationship (“water passing under the bridge”) is an especially important one for 77. (A) The superior rectal artery is the direct surgeons ligating the uterine artery, as in a hys- continuation of the inferior mesenteric artery. terectomy (Moore, p 380). The superior rectal artery anastomoses with branches of the middle rectal artery (a branch of 85. (E) Immediately superior to the perineal mem- the internal iliac artery) and with the inferior brane is the deep transverse perineal muscle rectal artery (a branch of the internal pudendal (Moore, p 390). artery) (Moore, p 355). 86. (D) The lumen of the spongy urethra is approx- 78. (B) The superior gluteal artery leaves the imately 5 mm in diameter: however, it is ex- pelvis through the superior part of the greater panded in the bulb of the penis to form the 0523-05 C hap 5 07/15/02 15:33 Page 85

Answers and Explanations: 70–105 85

intrabulbar fossa and in the glans penis to form attach to the upper border of the greater tro- the navicular fossa (Moore, p 403). chanter of the femur (Moore, p 341).

87. (A) The anterior aspect of the scrotum is sup- 95. (C) The pelvic splanchnic nerves contain para- plied by anterior scrotal nerves derived from sympathetic fibers derived from S2, S3, and S4 the ilioinguinal and the genital branch of the spinal cord segments and visceral afferent genitofemoral nerve. The posterior aspect of fibers from cell bodies in the spinal ganglia of the scrotum is supplied by posterior scrotal the corresponding spinal nerves (Moore, p 350). nerves, provided by perineal branches of the pudendal and perineal branches of the poste- 96. (B) The internal iliac artery begins at the level rior femoral cutaneous nerve (Moore, p 405). of the fourth disc between L5 and S1 vertebrae, where it is crossed by the ureter. It is separated 88. (E) The penis is composed of three cylindrical from the sacroiliac joint by the internal iliac bodies of erectile cavernous tissue enclosed by vein and the lumbosacral trunk (Moore, p 350). a fibrous capsule, the tunica albuginea. Super- ficial to the capsule is the deep fascia of the 97. (D) The superior end of the vagina surrounds penis (Buck’s fascia). Superficial to the deep the cervix; the lower end passes anteroinfe- fascia is the loose areolar tissue, which lies just riorly through the pelvic floor to open in the beneath the skin of the penis. The tunica dar- vestibule (Moore, p 371). tos is located in the scrotum (Moore, p 406). 98. (C) Only the lower one-fifth to one-fourth of 89. (C) The tiny deferential artery usually arises the vagina is somatic in terms of innervation. from the inferior vesical artery and terminates The innervation of this part of the vagina is by anastomosing with the testicular artery, from the deep perineal branch of the pudendal posterior to the testis (Moore, p 367). nerve (Moore, p 373).

90. (C) The free anterior borders of the levator ani 99. (E) The major part of the broad ligament, the are separated by a gap, the urogenital hiatus, mesentery of the uterus or mesometrium, through which the urethra (and, in the female, is below the mesosalpinx and mesovarium the vagina) passes (Moore, p 357). (Moore, p 377).

91. (D) The puborectalis muscle unites with its 100. (D) The anterior part of the tendinous arch of partner to from a U-shaped sling around the pelvic fascia forms the puboprostatic ligament anorectal junction. The puborectalis is respon- in males or the pubovesical ligament in females. sible for the anorectal angle (perineal flexure), The posterior part of the tendinous arch of pel- which is important in maintaining fecal conti- vic fascia forms the sacrogenital ligaments from nence (Moore, p 345). the sacrum around the side of the rectum to attach to the prostate in the male or the vagina 92. (E) The levator ani, a broad muscular sheet, is in the female (Moore, p 380). the largest and most important muscle in the pelvic floor (Moore, p 341). 101. pubic symphysis

93. (A) Each obturator internus passes posteriorly 102. inferior pubic ramus from the lesser pelvis through the lesser sciatic foramen and turns sharply laterally to attach 103. ischial tuberosity to the greater trochanter of the femur (Moore, p 341). 104. sacrotuberous ligament

94. (D) The piriformis muscle leaves the lesser 105. coccyx pelvis through the greater sciatic foramen to 0523-05 C hap 5 07/15/02 15:33 Page 86

86 5: The Pelvis and Perineum

106. bulbocavernosus 116.

107. ischiocavernous 117. prepuce of clitoris

108. superficial transverse perineal muscle 118. perineal branches of pudendal nerve

109. anal sphincter 119. inferior rectal nerve

110. levator ani 120. inferior cluneal nerve

111. rectouterine pouch 121. fundus of bladder

112. uterine tube 122. prostate gland

113. fundus of uterus 123. seminal vesicle

114. apex of bladder 124. rectovesical pouch

115. pubic symphysis 125. corpora cavernosa 0523-06 C hap 6 07/15/02 15:35 Page 87

CHAPTER 6 The Lower Limb Questions

DIRECTIONS (Questions 1 through 100): Each of the (A) pecten pubis numbered items or incomplete statements in this (B) pubic crest section is followed by answers or by completions of (C) lunate surface the statement. Select the ONE lettered answer or (D) ala completion that is BEST in each case. (E) ramus 1. All of the following statements concerning the femur are correct EXCEPT 5. A fracture of which of the following bones is among the most troublesome and problematic (A) It is the longest bone in the body. of all fractures? (B) It is the heaviest bone in the body. (A) (C) Its length is approximately a quarter of (B) fibula the person’s height. (C) (D) The distal end of the femur undergoes ossification just before birth. (D) femoral neck (E) Its average adult length is 36 inches. (E) medial malleolus

2. The adductor tubercle is located on which of 6. When it is said that an elderly person has a the following bones? “broken hip,” the usual injury is a fracture of which of the following structures? (A) femur (A) acetabulum (B) tibia (B) neck of the femur (C) fibula (C) pelvic rami (D) ischium (D) ischial tuberosity (E) ilium (E) ilium 3. The margin of the acetabulum is deficient infe- riorly at which of the following structures? 7. The medial and lateral malleoli articulate with which of the following bones? (A) ischial tuberosity (A) femur (B) obturator foramen (B) calcaneus (C) linea aspera (C) talus (D) acetabular notch (D) cuboid (E) ischial spine (E) fibula 4. A deficiency in the smooth articular surface of the acetabulum is known as which of the fol- lowing?

87 0523-06 C hap 6 07/15/02 15:35 Page 88

88 6: The Lower Limb

8. Which of the following bones is the most com- (C) crural fascia mon site for a compound fracture? (D) fascia lata (A) femur (E) plantar fascia (B) tibia 14. Which of the following structures covers the (C) fibula saphenous opening in the fascia lata? (D) ilium (E) ischium (A) lateral intermuscular septum (B) medial intermuscular septum 9. Which of the following bones is the largest and (C) crural fascia strongest bone of the foot? (D) cribriform fascia (A) talus (E) iliotibial tract (B) calcaneus 15. Which of the following structures passes (C) cuboid through the saphenous opening? (D) navicular (E) lateral cuneiform (A) femoral artery (B) femoral vein 10. The sustentaculum tali projects from the supe- (C) femoral nerve rior surface of which of the following bones? (D) obturator nerve (A) intermediate cuneiform (E) great saphenous vein (B) talus 16. The small saphenous vein empties into which (C) cuboid of the following veins? (D) navicular (E) calcaneus (A) femoral (B) tibial 11. Which of the following bones articulates with (C) peroneal the fibula, tibia, calcaneus, and navicular? (D) popliteal (A) talus (E) great saphenous (B) lateral cuneiform 17. Which of the following statements correctly (C) intermediate cuneiform applies to the iliopsoas muscle? (D) medial cuneiform (E) cuboid (A) It is a flat quadrangular muscle. (B) It is the chief flexor of the thigh. 12. Which of the following statements applies to (C) It is enclosed between two layers of the 2nd metatarsal bone? fascia lata. (A) It is the shortest. (D) It inserts into the iliotibial tract. (B) It is the strongest. (E) It is located in the posterior compart- ment of the thigh. (C) It is the longest. (D) Its base has a large tuberosity. 18. All of the following statements concerning the (E) Its base articulates with the navicular sartorius muscle are correct EXCEPT bone. (A) It is known as the “tailor’s muscle.” 13. The deep fascia of the thigh is known as which (B) It is the longest muscle in the body. of the following? (C) It acts across two joints. (A) Scarpa’s fascia (D) It extends the hip. (B) Colles’ fascia 0523-06 C hap 6 07/15/02 15:35 Page 89

Questions: 8–25 89

(E) It is located in the anterior compartment (C) It has adductor and hamstring parts. of the thigh. (D) It is a composite, triangular muscle with two parts that differ in nerve supply. 19. All of the following statements concerning the (E) Its main action is to adduct the thigh. quadriceps femoris are correct EXCEPT (A) Collectively constitutes the largest and 23. Which of the following statements correctly most powerful muscle group in the applies to the adductor hiatus? body. (A) It is an opening in the aponeurotic distal (B) It is the great extensor of the thigh. attachment of the adductor longus. (C) It inserts onto the tibia. (B) It transmits the femoral nerve, artery and (D) It forms the main bulk of the anterior vein. thigh muscles. (C) The opening is located just inferior to the (E) The patella provides additional leverage adductor tubercle of the femur. for the quadriceps. (D) It extends from the adductor canal in the thigh to the . 20. Which of the following statements concerning (E) The great saphenous vein passes the components of the quadriceps femoris is through the adductor hiatus. correct? (A) The rectus femoris is considered the 24. All of the following statements concerning the “kicking muscle.” are correct EXCEPT (B) The vastus lateralis is the smallest com- (A) Its superior border is the inguinal ponent of the quadriceps. ligament. (C) The articularis genus is a derivative of (B) Its lateral border is the sartorius. the vastus lateralis. (C) It is bisected by the femoral artery and (D) The rectus femoris lies deep to the vastus vein. intermedius. (D) The saphenous nerve passes through the (E) The vastus intermedius is the chief flexor femoral triangle. of the thigh. (E) Its medial border is the adductor magnus. 21. Which of the following statements concerning the gracilis muscle is correct? 25. All of the following statements concerning the (A) It passes through the lesser sciatic femoral sheath are correct EXCEPT foramen. (A) It extends 3 to 4 cm inferior the inguinal (B) It crosses the knee joint. ligament. (C) It lies deep to the pectineus and adductor (B) It is formed by an inferior prolongation longus muscles. of transversalis and iliopsoas fascia. (D) It is located in the anterior compartment (C) It encloses the femoral nerve. of the thigh. (D) Its medial wall is pierced by the great (E) It is a short, fan-shaped muscle. saphenous vein and lymphatic vessels. (E) It ends by becoming continuous with the 22. All of the following statements concerning the adventitia of the femoral vessels. adductor magnus are correct EXCEPT (A) It is the largest muscle in the adductor group. (B) It is located in the posterior compart- ment of the thigh. 0523-06 C hap 6 07/15/02 15:35 Page 90

90 6: The Lower Limb

26. All of the following statements concerning the (D) It exits the adductor canal through the femoral canal are correct EXCEPT adductor hiatus. (A) It contains the femoral nerve. (E) It passes through the obturator foramen. (B) It is the medial compartment of the 30. Which of the following arteries supplies most femoral sheath. of the blood to the head and neck of the femur? (C) It allows the femoral vein to expand when venous return from the lower limb (A) medial circumflex femoral is increased. (B) obturator (D) It extends distally to the level of the (C) lateral circumflex femoral proximal edge of the saphenous (D) external pudendal opening. (E) deep circumflex iliac (E) It contains loose connective tissue, fat, a few lymphatic vessels, and sometimes a 31. All of the following statements concerning the deep inguinal (Cloquet’s adductor canal are correct EXCEPT node). (A) It is approximately 15 cm long. 27. All of the following statements concerning the (B) It extends from the apex of the femoral femoral ring are correct EXCEPT triangle to the adductor hiatus. (C) It contains the saphenous nerve. (A) The lacunar ligament forms its medial boundary. (D) Is is bounded posteriorly by the sartorius muscle. (B) The anterior boundary is formed by the inguinal ligament. (E) It contains the femoral artery and vein. (C) The femoral artery forms its lateral 32. Which of the following statements concerning boundary. the lesser sciatic foramen is correct? (D) Its proximal end is closed by extraperi- toneal fatty tissue that forms the femoral (A) It is the passageway for structures enter- septum. ing or leaving the pelvis. (E) It lies anterior to the pectineus muscle. (B) It is the passageway for structures enter- ing or leaving the perineum. 28. Which of the following statements concerning (C) All lower limb arteries and nerves leave the femoral artery is correct? the pelvis through this foramen. (A) It enters the femoral canal. (D) The femoral nerve passes through the lesser sciatic foramen. (B) It enters the adductor canal. (E) The obturator nerve enters the adduc- (C) It passes dorsal to the inguinal ligament. tor compartment via the lesser sciatic (D) It gives rise to the inferior epigastric foramen. artery. (E) It passes through the obturator canal. 33. All of the following structures pass through the greater sciatic foramen EXCEPT 29. Which of the following statements concerning the deep artery of the thigh is correct? (A) piriformis muscle (B) sciatic nerve (A) It is the largest branch of the femoral (C) superior gluteal vessels artery. (D) inferior gluteal vessels (B) It passes through the adductor canal. (E) pudendal nerve (C) It gives rise to the deep circumflex iliac branch. 34. All of the following statements concerning the are correct EXCEPT 0523-06 C hap 6 07/15/02 15:35 Page 91

Questions: 26–42 91

(A) It is used very little during casual 39. All of the following muscles are lateral rota- walking. tors of the thigh EXCEPT (B) It assists in making the knee stable. (A) quadratus femoris (C) It is used very little in climbing upstairs. (B) obturator internus (D) It is used in running. (C) obturator externus (E) It is used when rising from the sitting (D) gluteus medius position. (E) inferior gemellus 35. The ischial bursa separates the inferior part of 40. All of the following statements concerning the which of the following muscles from the ischial inferior clunial nerves are correct EXCEPT tuberosity? (A) They are gluteal branches of the poste- (A) gluteus maximus rior cutaneous nerve of the thigh. (B) gluteus minimus (B) These nerves curl around the inferior (C) gluteus medius border of the gluteus maximus. (D) piriformis (C) They are derivatives of the sacral plexus (E) obturator internus S1–S3. (D) They supply the inferior half of the 36. All of the following statements concerning buttock. the gluteus medius and minimus are correct (E) They are dorsal primary rami. EXCEPT (A) They all have the same nerve supply. 41. All of the following nerves are ventral primary (B) They have the same actions. rami EXCEPT (C) They are supplied by the same blood (A) posterior cutaneous nerve of the thigh vessels. (B) inferior gluteal (D) They abduct the thigh and rotate it (C) pudendal laterally. (D) sciatic (E) They are largely responsible for prevent- (E) superior cluneal ing sagging of the unsupported side of the pelvis during walking. 42. All of the following statements concerning the sciatic nerve are correct EXCEPT 37. The positive Trendelenburg sign is associated with injuries to which of the following? (A) It is the largest nerve in the body. (A) quadriceps femoris (B) It is really two nerves. (B) adductor muscles (C) It supplies all leg and foot muscles. (C) abductors of the thigh (D) It receives its blood supply from the superior gluteal nerve. (D) hamstring muscles (E) It passes inferolaterally under cover of (E) piriformis muscle the gluteus maximus, midway between the greater trochanter and the ischial 38. Which of the following muscles is part of the tuberosity. triceps coxae? (A) obturator externus (B) gemelli muscles (C) piriformis (D) gluteus medius (E) quadratus femoris 0523-06 C hap 6 07/15/02 15:35 Page 92

92 6: The Lower Limb

43. The piriformis syndrome involves which of 48. A line drawn from the anterior superior iliac the following nerves? spine to the ischial tuberosity (Nelaton’s line), passing over the lateral aspect of the hip, (A) sciatic normally passes over which of the following (B) obturator structures? (C) femoral (A) sciatic nerve (D) ilioinguinal (B) ischial spine (E) inferior gluteal (C) obturator canal 44. All of the following statements concerning the (D) pudendal nerve are correct EXCEPT (E) greater trochanter (A) It does not supply any structures in the 49. All of the following statements concerning the gluteal region. popliteal fossa are correct EXCEPT (B) It passes to the perineum with the pudendal nerve. (A) The biceps femoris forms the superolat- (C) It supplies the external genitalia. eral border. (D) It crosses the ischial tuberosity. (B) The semimembranosus forms the super- omedial border. (E) It re-enters the pelvis through the lesser sciatic foramen. (C) It contains both the tibial and common fibular nerves. 45. Which of the following muscles is located in (D) The lateral and medial heads of the gas- the posterior aspect of the thigh? trocnemius form the inferolateral and inferomedial borders. (A) semitendinosus (E) It contains the great saphenous vein. (B) gluteus maximus (C) piriformis 50. The floor of the popliteal fossa includes which (D) internal obturator of the following structures? (E) superior gemellus (A) oblique popliteal ligament (B) patella 46. All of the following statements concerning the hamstring muscles are correct EXCEPT (C) lateral meniscus (D) anterior cruciate ligament (A) They are extensors of the thigh. (E) posterior cruciate ligament (B) They are flexors of leg. (C) A person with paralyzed hamstrings 51. The lateral sural nerve is a branch of which of tends to fall backwards. the following nerves? (D) Most take origin from the ischial (A) femoral tuberosity. (B) common fibular (E) Most are innervated by the tibial divi- sion of the sciatic nerve. (C) tibial (D) obturator 47. The innervation for the short head of the biceps (E) posterior cutaneous nerve of the thigh is provided by which of the following nerves? 52. Which of the following muscles dorsiflexes the (A) obturator ankle? (B) femoral (C) tibial division of the sciatic (A) extensor digitorum longus (D) fibular division of the sciatic (B) fibularis longus (E) sartorius (C) soleus 0523-06 C hap 6 07/15/02 15:35 Page 93

Questions: 43–62 93

(D) plantaris (D) extensor digitorum longus (E) gastrocnemius (E) extensor hallucis longus

53. Muscles that evert the foot include which of the 58. Which of the following muscles is the power- following muscles? ful “push-off” muscle? (A) gastrocnemius (A) gastrocnemius (B) soleus (B) soleus (C) tibialis posterior (C) tibialis anterior (D) fibularis brevis (D) flexor hallucis longus (E) flexor digitorum longus (E) plantaris

54. All of the following statements correctly apply 59. Which of the following muscles contract to to the inferior extensor retinaculum EXCEPT assist the posterior cruciate ligament in pre- venting anterior displacement of the femur on (A) It passes from the fibula to the tibia prox- the tibia? imal to the malleoli. (B) It is a Y-shaped band of deep fascia. (A) plantaris (C) It attaches laterally to the anterosuperior (B) popliteus surface of the calcaneus. (C) soleus (D) It forms a strong loop around the ten- (D) gastrocnemius dons of the fibularis tertius. (E) tibialis posterior (E) It forms a strong loop around the ten- dons of the extensor digitorum longus. 60. Which of following muscles is included in the triceps surae? 55. Which of the following muscles is the strongest (A) gastrocnemius dorsiflexor and invertor of the foot? (B) tibialis anterior (A) extensor digitorum longus (C) tibialis posterior (B) tibialis anterior (D) popliteus (C) fibularis tertius (E) fibularis brevis (D) extensor hallucis longus (E) flexor hallucis longus 61. The lateral compartment of the foot contains which of the following muscles? 56. All of the following muscles are located in the (A) abductor digiti minimi brevis deep muscle group of the posterior compart- ment EXCEPT (B) flexor hallucis brevis (C) quadratus plantae (A) flexor digitorum longus (D) flexor hallucis longus (B) popliteus (E) abductor hallucis (C) plantaris (D) flexor hallucis longus 62. The central compartment of the foot contains (E) tibialis posterior all of the following muscles EXCEPT (A) flexor digitorum brevis 57. Which of the following muscles is a flexor of the knee joint? (B) flexor digitorum longus (C) quadratus plantae (A) popliteus (D) abductor hallucis (B) tibialis anterior (E) lumbricals (C) fibularis longus 0523-06 C hap 6 07/15/02 15:35 Page 94

94 6: The Lower Limb

63. The medial plantar nerve is a terminal branch (C) It contains a small nerve. of which of the following nerves? (D) Usually it contains a small artery. (A) femoral (E) Its wide end attaches to the margins of (B) tibial the acetabular notch and the transverse ligament. (C) fibular (D) obturator 68. Which of the following arteries provides the (E) sural main blood supply for the hip joint?

64. All of the following statements concerning the (A) obturator saphenous nerve are correct EXCEPT (B) medial circumflex (C) lateral circumflex (A) It is the largest cutaneous branch of the femoral nerve. (D) common iliac (B) It passes anterior to the medial malleolus (E) internal iliac to the dorsum of the foot. 69. Which of the following muscles is the most (C) It supplies skin along the medial side of important muscle in stabilizing the knee? the foot. (D) It supplies skin on the anterior and (A) biceps femoris medial side of the leg. (B) adductor magnus (E) It innervates the muscles of the foot in (C) obturator internus the medial compartment of the foot. (D) quadriceps femoris (E) piriformis 65. Which of the following ligaments prevents hyperextension of the hip joint during stand- 70. Which of the following muscles passes out of ing? the knee joint to reach the tibia? (A) iliofemoral (A) plantaris (B) ischiofemoral (B) popliteus (C) pubofemoral (C) sartorius (D) ligament of head of femur (D) gracilis (E) transverse acetabular (E) tibialis anterior

66. A synovial protrusion beyond the free margin 71. All of the following statements concerning the of the fibrous capsule onto the posterior aspect fibular collateral ligament are correct EXCEPT of the femoral neck forms a bursa for which of these muscle tendons? (A) It splits the tendon of the biceps femoris. (B) It is superficial to the tendon of the (A) piriformis popliteus. (B) obturator internus (C) It is connected to the lateral meniscus. (C) obturator externus (D) It is rounded and cordlike. (D) superior gemellus (E) It extends from the lateral epicondyle of (E) quadratus femoris the femur to the head of the fibula.

67. All of the following statements concerning the 72. Which of the following statements concerning ligament of the head of the femur are correct the tibial collateral ligament is correct? EXCEPT (A) It is attached to the lateral meniscus. (A) It is a weak ligament. (B) It is stronger than the fibular collateral (B) It is of little importance in strengthening ligament. the hip. 0523-06 C hap 6 07/15/02 15:35 Page 95

Questions: 63–79 95

(C) It is less frequently damaged than the (C) They are wedge-shaped in transverse fibular collateral ligament. section. (D) The tibial collateral ligament and medial (D) Their external margins attach to the meniscus are commonly torn during fibrous capsule of the knee joint. contact sports such as football. (E) The transverse ligament of the knee joins (E) It is an expansion of the tendon of the the posterior edges of the menisci. adductor magnus. 77. Which of the following statements correctly 73. The oblique popliteal ligament is an expan- applies to the lateral meniscus? sion of the tendon of which of the following (A) It is larger and less movable than the muscles? medial meniscus. (A) popliteus (B) It is in contact with the fibular collateral (B) plantaris ligament. (C) adductor magnus (C) It is attached to the posterior cruciate lig- (D) semimembranosus ament by the posterior meniscofemoral (E) gastrocnemius ligament. (D) It adheres to the deep surface of the tib- 74. All of the following ligaments of the knee are ial collateral ligament. intra-articular EXCEPT (E) It acts like a shock absorber. (A) anterior cruciate 78. Which of the following genicular branches sup- (B) lateral meniscus plies the cruciate ligaments? (C) patellar (A) femoral (D) posterior cruciate (B) popliteal (E) medial meniscus (C) anterior recurrent branches of the ante- 75. Which of the following statements concerning rior tibial recurrent the anterior cruciate ligament is correct? (D) circumflex fibular (E) posterior recurrent branches of the ante- (A) It is the stronger of the two cruciate liga- rior tibial recurrent ments. (B) It has a relatively poor blood supply. 79. Which of the following three ligaments are (C) It is the main stabilizing factor for the damaged in the “unhappy triad” of knee in- femur when one is walking downhill. juries? (D) It tightens during flexion of the knee (A) ACL, tibial collateral, and medial joint, preventing anterior displacement meniscus of the tibia on the femur. (B) PCL, fibular collateral, and lateral (E) It attaches to the anterior part of the lat- meniscus eral surface of the medial condyle of the femur. (C) ACL, fibular collateral, and medial meniscus 76. All of the following statements concerning the (D) PCL, tibial collateral, and lateral menisci of the knee joint are correct EXCEPT meniscus (E) ACL, tibial collateral, and lateral (A) They are thicker at their external meniscus margins. (B) They taper to thin, unattached edges in the interior of the joint. 0523-06 C hap 6 07/15/02 15:35 Page 96

96 6: The Lower Limb

80. Pain on lateral rotation of the tibia on the femur (C) The calcaneus is the keystone of this arch. indicates damage to which of the following (D) The tibialis anterior helps strengthen this structures? arch. (A) anterior cruciate ligament (E) The fibularis longus tendon also supports (B) posterior cruciate ligament this arch. (C) lateral meniscus 85. Which of the following conditions is associ- (D) medial meniscus ated with lateral deviation of the great toe? (E) posterior meniscofemoral ligament (A) hallux valgus 81. All of the following ligaments reinforce the (B) hammer toe fibrous capsule on the medial side of the ankle (C) claw toe EXCEPT (D) pes planus (A) tibiocalcaneal (E) club foot (B) posterior tibiotalar 86. All of the following statements concerning the (C) anterior tibiotalar transverse tarsal joint are correct EXCEPT (D) tibionavicular (E) calcaneofibular (A) It occurs where the talus rests on and articulates with the calcaneus. 82. Which of the following groups of muscles pro- (B) It is formed by the combined talonavicu- duce dorsiflexion of the ankle? lar part of the talocalcaneonavicular and calcaneocuboid joints. (A) anterior compartment of the leg (C) Transection across this joint is a standard (B) posterior compartment of the leg method for surgical amputation of the (C) medial compartment of the foot foot. (D) lateral compartment of the leg (D) It is two separate joints aligned trans- (E) lateral compartment of the foot versely. (E) Dorsiflexion and plantarflexion of the 83. All of the following statements concerning the foot are the main movements involving plantar calcaneonavicular ligament are correct this joint. EXCEPT (A) It extends from the sustentaculum tali to 87. Which of the of the following areas is involved the navicular. in tibial nerve entrapment? (B) It is also known as the spring ligament. (A) medial malleolus to the calcaneus (C) It lies deep to the plantar aponeurosis. (B) lateral malleolus to the navicular bone (D) It plays an important role in maintaining (C) medial malleolus to the calcaneal tendon the longitudinal arch of the foot. (D) lateral malleolus to the fibularis brevis (E) It forms a tunnel for the tendon of the (E) long plantar ligament to the tendon of fibularis longus. the fibularis longus

84. All of the following statements concerning the 88. Which of the following major joints is the most medial longitudinal arch of the foot are correct frequently injured? EXCEPT (A) shoulder (A) It is higher and more important than the (B) elbow lateral longitudinal arch. (C) hip (B) It is composed of the calcaneus, talus, (D) knee navicular, cuneiforms, and three metatarsals. (E) ankle 0523-06 C hap 6 07/15/02 15:35 Page 97

Questions: 80–98 97

89. Which of the following statements correctly 94. Which of the following muscles is the strongest applies in Pott’s fracture-dislocation? flexor of the hip joint? (A) The foot is forcibly dorsiflexed. (A) semitendinosus (B) The foot is forcibly plantar flexed. (B) iliopsoas (C) The foot is forcibly inverted. (C) gluteus medius (D) The foot is forcibly everted. (D) gracilis (E) The calcaneus is usually fractured. (E) pectineus

90. The grip of the malleoli on the trochlea is 95. All of the following muscles are lateral rota- strongest during which of the following move- tors of the hip joint EXCEPT ments of the ankle? (A) obturator externus (A) plantarflexion (B) superior gemellus (B) dorsiflexion (C) piriformis (C) eversion (D) gluteus minimus (D) inversion (E) quadratus femoris (E) lateral rotation 96. Which of the following arteries is usually eval- 91. Which of the following statements correctly uated during a physical examination of the applies to genu valgum? peripheral vascular system? (A) The tibia is diverted medially. (A) lateral plantar (B) The tibia is diverted laterally. (B) dorsalis pedis (C) The medial side of the knee takes all the (C) popliteal pressure. (D) fibular (D) This deformity causes wear and tear of (E) posterior tibial the medial meniscus. (E) This deformity does not influence 97. The tendon of the biceps femoris and the neck weight distribution. of the fibula may be used as a guide for locat- ing which of the following nerves? 92. Which of the following knee support struc- (A) saphenous tures is considered to be the most important in the stabilization of the joint? (B) sural (C) common fibular (A) lateral and medial menisci (D) medial plantar (B) anterior and posterior cruciate ligaments (E) tibial (C) medial and lateral collateral ligaments (D) fibrous capsule 98. Which of the following muscle tendons is com- (E) quadriceps femoris monly removed for grafting without causing disability? 93. Which of the following fractures is the most (A) extensor hallucis brevis troublesome and problematic? (B) fibularis tertius (A) medial malleolus of tibia (C) plantaris (B) lateral malleolus of fibula (D) flexor digitorum brevis (C) sustentaculum of calcaneus (E) tendons of the flexor digitorum longus (D) femoral neck (E) adductor tubercle 0523-06 C hap 6 07/15/02 15:35 Page 98

98 6: The Lower Limb

99. Shin splints usually involve muscles in which 100. The strongest dorsiflexor of the foot is which of the following compartments? of the following muscles? (A) anterior (A) fibularis tertius (B) lateral (B) extensor digitorum longus (C) posterior (C) tibialis anterior (D) medial plantar (D) gastrocnemius (E) lateral plantar (E) extensor hallucis brevis

DIRECTIONS (Questions 101 through 105): Identify DIRECTIONS (Questions 106 through 110): Identify the anatomical features indicated on the art below. the anatomical features indicated on the art below. 0523-06 C hap 6 07/15/02 15:35 Page 99

Questions: 99–115 99

DIRECTIONS (Questions 111 through 115): Identify the anatomical features indicated on the art below. 0523-06 C hap 6 07/15/02 15:35 Page 100

Answers and Explanations

1. (E) The femur is the longest and heaviest bone 6. (B) Elderly people fracture the neck of the in the body. In the average adult, its length femur and refer to the injury as a “broken hip” is approximately one-quarter of the person’s (Moore, p 511). height (108 cm or 18 in.). The distal end of the femur undergoes ossification just before birth 7. (C) The lateral surface of the medial malleolus (Moore, p 509). articulates with the talus, and the lateral malle- olus helps hold the talus in its socket (Moore, 2. (A) The medial surface of the medial condyle p 513). has a large and prominent medial epicondyle, superior to which is another elevation, the 8. (B) The body of the tibia is the most common adductor tubercle (Moore, p 509). site for a compound fracture—one in which the skin is perforated and blood vessels are torn 3. (D) The margin of the acetabulum is deficient (Moore, p 513). inferiorly at the acetabular notch, which makes the fossa resemble a cup with a piece of its lip 9. (B) The calcaneus is the largest and strongest missing (Moore, p 508). bone in the foot. The calcaneus transmits most of the body weight from the talus to the ground 4. (C) The acetabular notch and fossa comprise (Moore, p 515). a deficiency in the smooth articular lunate sur- face of the acetabulum, which actually articu- 10. (E) The shelf-like sustentaculum tali projects lates with the head of the femur (Moore, p 508). from the superior border of the medial surface of the calcaneus and supports the talar head 5. (D) A fracture of the femoral neck is among the (Moore, p 515). most troublesome and problematic of all frac- tures because of the instability of the fracture 11. (A) The talus articulates with the fibula, cal- site. Furthermore, the periosteum covering the caneus, navicular, and tibia (Moore, p 515). femoral neck is exceedingly thin and has ex- tremely limited powers of osteogenesis (bone 12. (C) The 1st metatarsal is shorter and stouter formation). Because the retinacular arteries than the others. The 2nd metatarsal is the arise from the medial circumflex femoral arter- longest. The bases of the metatarsals articulate ies and run parallel to the femoral neck on their with the cuneiform and cuboid bones. The base way to supply the femoral head, they are vul- of the 5th metatarsal has a large tuberosity nerable to injury when the neck of the femur (Moore, p 515). fractures. Rupture of these vessels cause de- generation (necrosis) of the femoral head and 13. (D) The deep fascia of the thigh is called fas- bleeding into the hip joint (Moore, p 511). cia lata and the deep fascia of the leg is called

100 0523-06 C hap 6 07/15/02 15:35 Page 101

Answers and Explanations: 1–25 101

crural fascia. Scarpa’s fascia is the membra- most powerful muscles in the body. The qua- nous fascia of the lower abdominal wall and driceps is the great extensor of the leg, and all Colles’ fascia is located in the perineum (Moore, four of its parts combine to from a tendinous p 522). attachment to the tibia. The patella also pro- vides additional leverage for the quadriceps 14. (C) The saphenous opening in the fascia lata (Moore, pp 532–534). is a deficiency in the deep fascia lata inferior to the medial part of the inguinal ligament, 20. (A) The rectus femoris is considered to be the approximately 4 cm inferolateral to the pubic “kicking muscle.” The vastus lateralis is the tubercle. The medial margin of the saphenous largest component of the quadriceps. The vas- opening is smooth, but its superior, lateral, tus intermedius lies deep to the rectus femoris, and inferior margins form a sharp crescentic and the articular muscle is considered to be a edge, the falciform margin. The sickle-shaped derivative of the vastus intermedius (Moore, margin of the saphenous opening is joined at p 534). its medial margin by fibrofatty tissue—the cribriform fascia (Moore, p 524). 21. (B) The long, strap-like muscle lies along the medial side of the thigh and knee. It is the only 15. (E) The great saphenous vein passes through muscle of the adductor group that crosses the the saphenous opening and cribriform fascia knee. It is the most superficial of the adduc- to enter the femoral vein. Some efferent lym- tor group and is the weakest member (Moore, phatic vessels from the superficial inguinal p 538). lymph nodes also pass through the saphenous opening and cribriform fascia to enter the deep 22. (B) The adductor magnus is the largest mus- inguinal lymph nodes (Moore, p 524). cle in the adductor group. It is located in the medial compartment of the thigh. This large 16. (D) The small saphenous vein ascends poste- adductor is a composite, triangular muscle that rior to the lateral malleolus and passes along has adductor and hamstring parts. The two the lateral border of the calcaneal tendon. It parts differ in their attachments, nerve supply, then ascends between the heads of the gastroc- and main actions (Moore, p 538). nemius muscle and empties into the popliteal vein in the popliteal fossa (Moore, p 526). 23. (D) The adductor hiatus is an opening in the aponeurotic distal attachment of the adductor 17. (B) The iliopsoas is the chief flexor of the thigh, magnus. It transmits the femoral artery and and when the thigh is fixed, it flexes the trunk vein from the adductor canal in the thigh to the on the hip. Its broad lateral part, the iliacus, and popliteal fossa posterior to the knee. The open- its long medial part, the psoas major, arise from ing is just superior to the adductor tubercle of the iliac fossa and lumbar vertebrae, respec- the femur (Moore, p 541). tively. It is an anterior thigh muscle (Moore, p 531). 24. (E) The femoral triangle is bounded superi- orly by the inguinal ligament, medially by the 18. (D) The sartorius is known as the “tailor’s adductor longus, and laterally by the sarto- muscle.” It is the longest muscle in the body rius. The femoral triangle is bisected by the and acts across two joints. It flexes the hip femoral artery and vein, which leave and enter joint and participates in flexion of the knee. It the adductor canal at its apex. The saphenous is located in the anterior compartment of the nerve descends through the femoral triangle thigh (Moore, p 531). (Moore, p 541).

19. (B) The quadriceps femoris forms the main 25. (C) The femoral sheath extends 3 to 4 cm in- bulk of the anterior thigh muscles and collec- ferior to the inguinal ligament and encloses tively constitutes the largest and one of the proximal parts of the femoral vessel and the 0523-06 C hap 6 07/15/02 15:35 Page 102

102 6: The Lower Limb

femoral canal. The sheath is formed by an infe- deep or posterior to the middle third of the sar- rior prolongation of transversalis and iliopsoas torius, the adductor canal provides an inter- fascia. The femoral sheath does not enclose the muscular passage through which the femoral femoral nerve. The sheath ends by becoming vessels pass to reach the popliteal fossa. The continuous with the adventitia of the femoral contents of the adductor canal include the vessels (Moore, p 542). femoral vessels, saphenous nerve, and nerve to the vastus medialis (Moore, p 549). 26. (A) The medial compartment of the femoral sheath is the femoral canal. It extends distally to 32. (B) The greater sciatic foramen is the passage- the level of the proximal edge of the saphenous way for structures entering or leaving the pel- opening. It allows the femoral vein to expand vis, whereas the lesser sciatic foramen is the when venous return from the lower limb is in- passageway for structures entering or leaving creased. It contains loose connective tissue, fat, the perineum. The greater sciatic foramen is the a few lymphatic vessels, and sometimes a deep opening for all lower limb arteries and nerves inguinal lymph node (Cloquet’s node) (Moore, leaving the pelvis and entering the gluteal pp 542–543). region (Moore, p 550).

27. (C) The femoral ring is closed at the proxi- 33. (E) The greater sciatic foramen is the passage- mal end by extraperitoneal fatty tissue, which way for the sciatic nerve, piriformis muscle, forms the femoral septum. The boundaries of and gluteal vessels. The pudendal nerve the femoral ring include the partition between enters the perineum through the lesser sciatic the femoral canal and the femoral vein laterally. foramen (Moore, p 553). Posteriorly, the superior ramus of the pubis is covered by the pectineus muscle. Its medial 34. (C) The gluteus maximus is used when rising boundary is the lacunar ligament; anteriorly, from the sitting position or straightening from the boundary is the medial part of the inguinal the bending position. It is used in climbing ligament (Moore, pp 543–545). steps and running. It also assists in making the knee stable. It is used very little during casual 28. (B) The femoral artery is the chief artery of the walking and when one is standing motionless lower limb and is the continuation of the exter- (Moore, p 552). nal iliac artery. It bisects the femoral triangle at its apex and enters the adductor canal deep to 35. (A) The ischial bursa separates the inferior the sartorius muscle (Moore, p 545). part of the gluteus maximus from the ischial tuberosity, which is often absent (Moore, p 552). 29. (A) The deep artery of the thigh is the largest branch of the femoral artery and the chief artery 36. (D) The gluteus medius and minimus have the of the thigh. It arises in the femoral triangle from same actions and nerve supply and are sup- the lateral side of the femoral artery (Moore, plied by the superior gluteal arteries. Both mus- p 545). cles abduct the thigh and rotate it medially. They play an essential role during locomotion 30. (A) The medial circumflex femoral artery is and are largely responsible for preventing sag- especially important because it supplies most ging of the unsupported side of the pelvis dur- of the blood to the head and neck of the femur ing walking (Moore, p 554). (Moore, p 545). 37. (C) When the weight is on both feet, the pelvis 31. (D) The adductor canal (Hunter’s canal) is is evenly supported and does not sag. When the approximately 15 cm long and is a narrow fas- weight is borne by one foot, the muscles on the cial tunnel in the thigh running from the apex same side hold the pelvis so the pelvis will not of the femoral triangle to the adductor hiatus in sag on the side of the raised foot. When the glu- the tendon of the adductor magnus. Located teus medius and minimus (abductors of the 0523-06 C hap 6 07/15/02 15:35 Page 103

Answers and Explanations: 26–51 103

thigh) are inactive owing to injury of the supe- the lesser sciatic foramen. The artery passes to rior gluteal nerve, the supporting and steady- the perineum with the pudendal nerve and ing action of these muscles is lost and the pel- supplies the external genitalia and muscles in vis falls on the side of the raised limb. This is the pelvic region. It does not supply any struc- referred to as a positive Trendelenburg sign tures in the gluteal region (Moore, p 562). (Moore, p 555). 45. (A) The three muscles in the posterior aspect 38. (B) The obturator internus and the superior of the thigh are the hamstrings, which include and inferior gemelli form a tricipital (three- the semitendinosus, semimembranosus, and headed) muscle that is sometimes called the biceps femoris (Moore, p 563). triceps coxae (Moore, p 550). 46. (C) The hamstring muscles arise from the is- 39. (D) The gluteus maximus, piriformis, obtu- chial tuberosity and are innervated by the tib- rator internus and externus, and superior and ial division of the sciatic nerve. The short inferior gemelli are all lateral rotators of the head of the biceps does not meet these crite- thigh. Both the gluteus medius and the glu- ria. The hamstrings are extensors of the thigh teus minimus are medial rotators of the thigh and flexors of the leg. A person with paralyzed (Moore, p 551). hamstrings tends to fall forward because the gluteus maximus muscles cannot maintain the 40. (E) The inferior clunial nerves are gluteal necessary muscle tone to stand straight (Moore, branches of the posterior cutaneous nerve of p 563). the thigh, a derivative of the sacral plexus (ven- tral rami S1 through S3). These nerves curl 47. (D) The long head of the biceps femoris is around the inferior border of the gluteus max- innervated by the tibial division of the sciatic imus and supply the inferior half of the buttock nerve and the short head of the biceps is inner- (Moore, p 556). vated by the fibular division of the sciatic nerve (Moore, p 563). 41. (E) The superior cluneal nerves are dorsal rami of L1–L3 and the middle cluneal nerves 48. (E) A line drawn from the anterior superior are dorsal rami of S1–S3. The sciatic, puden- iliac spine to the ischial tuberosity (Nélaton’s dal, posterior cutaneous nerve of the thigh, line), passing over the lateral aspect of the hip and the gluteal nerves are all ventral primary region, normally passes over or near the top of rami (Moore, p 557). the greater trochanter (Moore, p 567).

42. (D) The sciatic nerve is the largest nerve in the 49. (E) The biceps femoris forms the superolat- body. It is so large that it receives its own blood eral border and the semimembranosus muscle supply from the inferior gluteal artery. It runs forms the superomedial border. The medial and inferolaterally under cover of the gluteus max- lateral heads of the gastrocnemius form the imus, midway between the greater trochanter inferolateral and inferomedial borders. The and the ischial tuberosity. The sciatic nerve is contents of the fossa include the small saphe- really two nerves, the tibial and common fibu- nous vein, popliteal arteries and veins, and tib- lar (Moore, p 558). ial and common fibular nerves (Moore, p 571).

43. (A) A pain in the buttock may result from com- 50. (A) The floor of the popliteal fossa is formed by pression of the sciatic nerve by the piriformis the popliteal surface of the femur, the oblique muscle (piriformis syndrome) (Moore, p 559). popliteal ligament, and the popliteal fascia over the popliteus (Moore, p 571). 44. (D) The internal pudendal artery leaves the gluteal region immediately by crossing the is- 51. (B) The medial sural nerve is derived from the chial spine and re-entering the pelvis through tibial nerve and the lateral sural nerve is 0523-06 C hap 6 07/15/02 15:35 Page 104

104 6: The Lower Limb

derived from the common fibular. The medial 62. (D) The central compartment of the foot con- and lateral sural nerves unite to form the sural tains the flexor digitorum brevis, flexor digi- nerve (Moore, p 590). torum longus, quadratus plantae, lumbricals, proximal part of the tendon flexor hallucis 52. (A) Dorsiflexors of the ankle include the tibialis longus, and the lateral plantar nerve and ves- anterior, extensors digitorum longus, hallucis sel (Moore, p 596). longus, and fibularis tertius (Moore, p 577). 63. (B) The medial plantar nerve, the larger of 53. (D) Both the fibularis longus and brevis evert the two terminal branches of the tibial nerve, the foot. The fibularis tertius also aids in ever- passes deep to the abductor hallucis and runs sion of the foot (Moore, p 577). anteriorly between the muscles and the flexor digitorum brevis on the lateral side of the 54. (A) The superior extensor retinaculum is a medial plantar artery (Moore, p 601). strong, broad band of deep fascia passing from the fibula to the tibia, proximal to the malleoli. 64. (E) The saphenous nerve is the largest cuta- The inferior extensor retinaculum is a Y-shaped neous branch of the femoral nerve. In addition band of deep fascia that attaches laterally to to supplying the skin and fascia on the ante- the anterosuperior surface of the calcaneus. It rior and medial sides of the leg, the saphenous forms a strong loop around the tendons of the nerve passes anterior to the medial malleolus to fibularis tertius and the extensor digitorum the dorsum of the foot, where it supplies skin longus muscle (Moore, p 579). along the medial side of the foot as far as the head of the 1st metatarsal (Moore, p 601). 55. (B) The tibialis anterior is the strongest dorsi- flexor and inverter of the foot (Moore, p 579). 65. (A) The fibrous capsule is reinforced anteriorly by the strong, Y-shaped iliofemoral ligament 56. (C) The plantaris is located in the superficial (of Bigelow) which attaches to the anterior infe- group of muscles in the posterior compartment rior iliac spine and the acetabular rim proxi- (Moore, p 586). mally and the intertrochanteric line distally. The iliofemoral ligament prevents hyperexten- 57. (A) The popliteus is a flexor of the knee joint sion of the hip during standing by screwing the (Moore, p 588). femoral head into the acetabulum (Moore, p 611).

58. (D) The flexor hallucis longus is the powerful 66. (C) A synovial protrusion beyond the free “push-off” muscle during walking, running, margin of the fibrous capsule onto the posterior and jumping (Moore, p 589). aspect of the femoral neck forms a bursa for the obturator externus tendon (Moore, p 611). 59. (B) When a person is standing with the knee partly flexed, the popliteus contracts to assist 67. (C) The ligament of the head of the femur is the posterior cruciate ligament in preventing weak and of little importance in strengthening anterior displacement of the femur on the tibia the hip joint. Its wide end attaches to the mar- (Moore, p 588). gins of the acetabular notch and the transverse acetabular ligament. Its narrow end attaches 60. (A) Together, the two-headed gastrocnemius to the pit in the head of the femur. Usually the and soleus form the three-headed triceps surae ligament contains a small artery to the head of (Moore, p 586). the femur (Moore, p 611).

61. (A) The lateral compartment of the foot con- 68. (B) The main blood supply of the hip joint is tains the abductor and flexor digiti minimi bre- from branches of the circumflex femoral arter- vis (Moore, p 596). ies (especially the medial circumflex femoral artery) that travel in the retinacula (reflections 0523-06 C hap 6 07/15/02 15:35 Page 105

Answers and Explanations: 52–79 105

of the capsule along the neck of the femur to- joint. The popliteal tendon is also intra-articular ward the head). These retinacular vessels may during part of its course (Moore, p 620). be damaged in femoral neck fractures and re- sult in avascular necrosis of the femoral head 75. (B) The anterior cruciate ligament (ACL), the (Moore, p 613). weaker of the two cruciate ligaments, arises from the anterior intercondylar area of the tibia, 69. (D) The most important muscle in stabilizing just posterior to the attachment of the medial the knee joint is the large quadriceps femoris, meniscus. It extends superiorly, posteriorly, and particularly inferior fibers of the vastus medi- laterally to attach to the posterior part of the alis and lateralis. The knee joint will function medial side of the lateral condyle of the femur. surprisingly well following a ligament strain The ACL has a relatively poor blood supply. It if the quadriceps is well conditioned (Moore, is slack when the knee is flexed and taut when pp 617–618). it is fully extended, preventing posterior dis- placement. (Moore, p 620). 70. (B) The fibrous capsule of the knee is deficient on the lateral condyle to allow the tendon of the 76. (E) The menisci are thicker at their external popliteus to pass out of the joint to attach to the margins and taper to thin, unattached edges in tibia (Moore, p 618). the interior of the joint. Wedge-shaped in trans- verse section, the menisci are firmly attached at 71. (C) The fibular collateral ligament (lateral their ends to the intercondylar area of the tibia. collateral ligament), rounded and cordlike, is Their external margins attach to the fibrous cap- strong. It extends inferiorly from the lateral epi- sule of the knee joint. The transverse ligament condyle of the femur to the lateral surface of of the knee, a slender, fibrous band, joins the the head of fibula. The tendon of the popliteus anterior edges of the menisci, allowing them to passes deep to the fibular collateral ligament, move together during knee movements (Moore, separating it from the lateral meniscus. The ten- p 621). don of the biceps femoris is also split into two parts by this ligament (Moore, p 619). 77. (D) The lateral meniscus is nearly circular and is smaller and more movable than the 72. (D) The tibial collateral ligament (medial col- medial meniscus. The tendon of the popliteus lateral ligament) is a strong, flat band that ex- separates the lateral meniscus from the fibular tends from the medial epicondyle of the femur collateral ligament. A strong tendinous slip, to the medial surface of the tibia. At its mid- the posterior meniscofemoral ligament, joins point, the deep fibers of the tibial collateral lig- the lateral meniscus to the posterior cruciate ament are firmly attached to the medial me- ligament and the medial femoral condyle. niscus. The tibial collateral ligament, weaker The lateral meniscus acts like a shock than the fibular collateral ligament, is more absorber (Moore, p 621). often damaged. As a result, the tibial collateral ligament and medial meniscus are commonly 78. (B) The middle genicular branches of the pop- torn during contact sports such as football liteal artery penetrate the fibrous capsule of the (Moore, p 619). knee joint and supply the cruciate ligaments, synovial membrane, and peripheral margins of 73. (D) The oblique popliteal ligament is an expan- the menisci (Moore, p 626). sion of the tendon of the semimembranosus, which strengthens the fibrous capsule posteri- 79. (A) The ACL may tear when the tibial collateral orly (Moore, p 619). ligament ruptures. First, the tibial collateral lig- ament ruptures, opening the joint on the medial 74. (C) The patellar ligament is extracapsular. The side and possibly tearing the medial meniscus cruciate ligaments and menisci are classified as and ACL. This “unhappy triad of injuries” can intra-articular and are found within the knee result from clipping in football (Moore, p 626). 0523-06 C hap 6 07/15/02 15:35 Page 106

106 6: The Lower Limb

80. (C) Pain on lateral rotation of the tibia on the tion across the transverse tarsal joint is a stan- femur indicates injury of the lateral meniscus, dard method for surgical amputation of the whereas pain on medial rotation of the tibia on foot. Inversion and eversion of the foot are the the femur indicates injury of the medial menis- main movements involving these joints (Moore, cus (Moore, p 628). p 637).

81. (E) The lateral ligament consists of the anterior 87. (A) The tibial nerve leaves the posterior com- and posterior talofibular ligaments and the cal- partment of the leg by passing deep to the caneofibular ligament. The three discrete liga- flexor retinaculum in the interval between ments are collectively referred to as the lateral the medial malleolus and calcaneus. The area ligament. The fibrous capsule is reinforced involved is from the medial malleolus to the medially by the large, strong medial ligament calcaneus, and the heel pain results from com- (deltoid ligament), which attaches proximally pression of the tibial nerve by the flexor reti- to the medial malleolus. This ligament consists naculum (Moore, p 636). of the tibionavicular, tibiocalcaneal, and ante- rior and posterior tibiotalar ligaments (Moore, 88. (E) The ankle is the most frequently injured pp 633–635). major joint in the body. Ankle sprains are most common. A sprained ankle is nearly always an 82. (A) Dorsiflexion of the ankle is produced by inversion injury (Moore, p 636). the muscles in the anterior compartment of the leg (Moore, p 635). 89. (D) A Pott’s fracture-dislocation of the ankle occurs when the foot is forcibly everted. This 83. (E) The plantar calcaneonavicular ligamentum action pulls on the extremely strong medial lig- (spring ligament) extends from the sustentacu- ament, often tearing off the medial malleolus. lum tali to the posteroinferior surface of the The talus then moves laterally, shearing off the navicular. It plays an important role in main- lateral malleolus or, more commonly, breaking taining the longitudinal arch of the foot (Moore, the fibula superior to the inferior tibiofibular p 637). joint (Moore, p 636).

84. (C) The talar head is the keystone of the medial 90. (B) The grip of the malleoli on the trochlea is longitudinal arch (Moore, p 640). strongest during dorsiflexion of the foot, be- cause this movement forces the wider, anterior 85. (A) Hallux valgus is a foot deformity char- part of the trochlea posteriorly, spreading the acterized by lateral deviation of the great toe. tibia and fibula slightly apart (Moore, p 632). Hammertoe is a deformity in which the prox- imal phalanx is permanently flexed at the 91. (B) Genu varum and genu valgum result in metatarsophalangeal joint and the middle pha- deviation of the tibia from the midline. In genu lanx is plantarflexed at the interphalangeal joint. varum, the tibia is diverted medially, and in The distal phalanx is also flexed. Claw toes are genu valgum, the tibia is diverted laterally; characterized by hyperextension of the metatar- these deformities cause unequal weight distri- sophalangeal joints and flexion of the distal bution. In the varum deformity, the medial side interphalangeal joints. Pes planus is flat feet, of the knee takes all the pressure, leading to and clubfoot refers to a foot that is twisted. The wear and tear of the medial meniscus (Moore, foot is inverted, the ankle is plantarflexed, and p 630). the forefoot is adducted (Moore, pp 641–642). 92. (E) The stability of the knee joint depends on 86. (E) The transverse tarsal joint is formed by the the strength and actions of the surrounding combined talonavicular part of the talocalca- muscles and their tendons. The ligaments that neonavicular and calcaneocuboid joints, two connect the femur and tibia are also important. separate joints aligned transversely. Transec- Of these supports, the muscles are most impor- 0523-06 C hap 6 07/15/02 15:35 Page 107

Answers and Explanations: 80–115 107

tant; therefore, many sport injuries are pre- 98. (C) Because of its minor role, the plantaris ventable through appropriate conditioning tendon can be removed for grafting without and training. The most important muscle in causing any disability (Moore, p 586). stabilizing the knee joint is the large quadri- ceps femoris, particularly inferior fibers of the 99. (A) Shin splints commonly result from trau- vastus medialis and lateralis. The knee joint matic injury or athletic overexertion of muscles will function surprisingly well following a lig- in the anterior compartment—especially the ament strain if the quadriceps is well condi- tibialis anterior (Moore, p 580). tioned (Moore, pp 617–618). 100. (C) The tibialis anterior is the strongest dorsi- 93. (D) Fractures of the femoral neck are intracap- flexor and invertor of the foot (Moore, p 579). sular, and realignment of the neck fragments requires internal skeletal fixation. Femoral neck 101. deep peroneal nerve fractures are among the most troublesome and problematic of all fractures (Moore, p 614). 102. tibialis anterior

94. (B) The iliopsoas is the strongest flexor of the 103. extensor digitorum longus hip joint (Moore, p 613). 104. extensor digitorum brevis 95. (D) Medial rotators of the hip joint include anterior fibers of the gluteus medius, gluteus 105. dorsalis pedis artery minimus, and tensor fascia lata; lateral rotators include the obturator externus, obturator inter- 106. abductor digiti minimi nus, superior gemellus, piriformis, quadratus femoris, and gluteus maximus (Moore, p 613). 107. flexor digitorum brevis

96. (B) The pulse of the dorsalis pedis artery, or 108. lateral plantar nerve dorsal artery of the foot, is evaluated during a physical examination of the peripheral vascu- 109. medial plantar nerve lar system. Dorsalis pedis pulses may be pal- pated with the feet slightly dorsiflexed (Moore, 110. lumbrical pp 603–604). 111. gluteus minimus 97. (C) The tendon of the biceps femoris may be traced by palpating its distal attachment to the 112. sciatic nerve lateral side of the head of the fibula. This tendon and the neck of the guide the examin- 113. semitendinosus ing finger to the common fibular nerve (Moore, p 592). 114. long head of biceps femoris

115. soleus This page intentionally left blank 0523-07 C hap 7 07/15/02 15:35 Page 109

CHAPTER 7 The Head and Neck Questions

DIRECTIONS (Questions 1 through 146): Each of the (C) bregma numbered items or incomplete statements in this (D) pterion section is followed by answers or by completions (E) vertebra prominens of the statement. Select the ONE lettered answer or completion that is BEST in each case. 5. Which of the following best describes the land- mark known as lambda? 1. Which of the following is NOT a bone of the neurocranium? (A) point on calvaria at junction of sagittal and lambdoid sutures (A) palatine bones (B) point on calvaria at junction of sagittal (B) parietal bones and coronal sutures (C) sphenoid bones (C) junction of the greater wing of the sphe- (D) temporal bones noid, squamous temporal, frontal, and (E) ethmoid bone parietal bones (D) star-shaped landmark at junction of pari- 2. Which of the following is NOT a bone of the etomastoid, occipitomastoid, and lamb- facial skeleton? doid sutures (A) maxilla (E) smooth prominence on frontal bone superior to root of nose (B) zygomatic bone (C) frontal bone 6. The superior point of the neurocranium in the (D) inferior nasal concha midline is known as which of the following? (E) nasal bone (A) pterion 3. The metopic suture is a persistence of which of (B) bregma the following? (C) vertex (D) asterion (A) frontal suture (E) nasion (B) coronal suture (C) sagittal suture 7. Which of the following foramina is NOT in the (D) lambdoid suture middle cranial fossa? (E) hypophyseal suture (A) foramen rotundum 4. The external occipital protuberance is also (B) foramen spinosum known as which of the following? (C) foramen lacerum (D) groove of greater petrosal nerve (A) nasion (E) foramen magnum (B) inion

109 0523-07 C hap 7 07/15/02 15:35 Page 110

110 7: The Head and Neck

8. Which of the following foramina is located in (C) the anterior cranial fossa? (D) ophthalmic arteries (A) foramen cecum (E) nerve branches to the meninges (B) optic canals 14. The maxillary division of the trigeminal nerve (C) superior orbital fissures is transmitted through which of the following? (D) foramen ovale (E) condylar canal (A) foramen rotundum (B) foramen ovale 9. Which of the following foramina does NOT (C) superior orbital fissure transmit emissary veins? (D) foramen spinosum (A) foramen cecum (E) foramen lacerum (B) condylar canal 15. Which of the following transmits a nerve branch (C) mastoid foramen to the meninges? (D) parietal foramen (E) anterior ethmoidal foramina (A) foramen spinosum (B) mastoid foramen 10. The superior orbital fissure transmits all of the (C) jugular foramen following EXCEPT (D) foramen magnum (A) ophthalmic division of the trigeminal (E) foramen oval nerve (B) maxillary division of the trigeminal nerve 16. The foramen magnum transmits all of the fol- lowing EXCEPT (C) oculomotor nerve (D) trochlear nerve (A) medulla and meninges (E) abducens nerve (B) vertebral arteries (C) spinal roots of the accessory nerve 11. All of the following transmit an arterial branch (D) dural veins to the meninges EXCEPT (E) internal carotid artery (A) foramen ovale (B) foramen rotundum 17. The jugular foramen transmits all of the fol- lowing EXCEPT (C) groove of greater petrosal nerve (D) jugular foramen (A) glossopharyngeal nerve (E) mastoid foramen (B) vagus nerve (C) accessory nerve 12. The foramina in the cribriform plate transmit (D) inferior petrosal and sigmoid sinuses which of the following? (E) sympathetic plexus (A) axons of olfactory cells (B) posterior ethmoidal arteries 18. Which of the following is NOT true regarding the buccinator? (C) ophthalmic arteries (D) dural veins (A) It is a muscle of mastication. (E) sympathetic plexus (B) It is innervated by the facial nerve. (C) It presses the cheek against the molar 13. In addition to the optic nerves, the optic canals teeth to assist in chewing. transmit which of the following? (D) It expels air from the oral cavity. (A) ophthalmic veins (E) It draws the mouth to one side when act- (B) oculomotor nerve ing unilaterally. 0523-07 C hap 7 07/15/02 15:35 Page 111

Questions: 8–28 111

19. Which of the following is NOT a muscle of fa- (D) zygomaticus major cial expression? (E) depressor septi (A) platysma 25. In respect to the orbicularis oculi, which of the (B) nasalis following is true? (C) frontal belly of occiptofrontalis (D) orbicularis oculi (A) Its orbital part delicately closes the eye- lids in blinking. (E) temporalis (B) Its palpebral part draws the eyelids 20. The mentalis does which of the following? medially so that tears may be drained. (C) Its lacrimal part tightly closes the eyelids (A) elevates eyebrows and skin of forehead in squinting. (B) functions as a sphincter of oral opening (D) It is innervated by a zygomatic branch of (C) elevates lip upper limb and dilates the facial nerve. nostril (E) It takes origin from the skin of the margin (D) elevates and protrudes lower lip of the orbit and the tarsal plate, and it (E) draws ala of nose toward inserts on the lacrimal bone, medial palpe- bral ligament, and medial orbital margin. 21. The orbicularis oculi is innervated by 26. Which of the following cutaneous nerves is (A) optic nerve derived from the ophthalmic nerve? (B) oculomotor nerve (C) trochlear nerve (A) external nasal nerve (D) trigeminal nerve (B) infraorbital nerve (E) facial nerve (C) zygomaticotemporal nerve (D) zygomaticofacial nerve 22. The facial nerve innervates all of the following (E) auriculotemporal nerve EXCEPT 27. Which of the following nerves arises by two (A) procerus roots that surround the middle meningeal (B) corrugator supercilii artery? (C) masseter (A) auriculotemporal nerve (D) depressor anguli oris (B) buccal nerve (E) zygomaticus major (C) mental nerve 23. All muscles of facial expression develop from (D) zygomaticotemporal nerve which pharyngeal arch? (E) zygomaticofacial nerve (A) first arch 28. Which of the following nerves is NOT cor- (B) second arch rectly matched with its distribution? (C) third arch (A) frontal nerve . . . skin of forehead, scalp, (D) fourth arch eyelid, and nose (E) sixth arch (B) supraorbital nerve . . . skin of forehead as far as vertex 24. Which of the following does NOT insert on the angle of the mouth? (C) supratrochlear . . . skin in middle of fore- head (A) platysma (D) infratrochlear . . . skin and conjunctiva of (B) buccinator upper eyelid (C) risorius (E) lacrimal . . . skin on dorsum of nose 0523-07 C hap 7 07/15/02 15:35 Page 112

112 7: The Head and Neck

29. Which of the following sequences of nerve (D) pterygopalatine ganglion branches is NOT correct? (E) geniculate ganglion (A) ophthalmic nerve . . . frontal nerve . . . supraorbital nerve 33. Which of the following is NOT a branch of the facial nerve? (B) ophthalmic nerve . . . frontal nerve . . . infratrochlear nerve (A) temporal (C) maxillary nerve . . . infraorbital nerve (B) zygomatic (D) mandibular nerve . . . auriculotemporal (C) buccal nerve (D) mental (E) mandibular nerve . . . inferior alveolar (E) cervical nerve . . . mental nerve 34. The temporal branch of CN VII does NOT 30. Which of the following nerves is correctly innervate which of the following? described with respect to its course? (A) auricularis superior (A) The auriculotemporal travels from the (B) auricularis anterior anterior division of the mandibular (C) occipitofrontalis (frontal belly) nerve in the infratemporal fossa to reach the cheek. (D) orbicularis oculi (superior part) (B) The buccal nerve travels from the poste- (E) orbicularis oculi (inferior part) rior division of the mandibular nerve between the neck of the and 35. Which of the following is true regarding the the external acoustic meatus. stylomastoid foramen? (C) The lacrimal nerve passes through the (A) It is located between the styloid and palpebral fascia of the upper eyelid near mastoid processes of the sphenoid bone. the lateral canthus of the eye. (B) CN V is transmitted through it. (D) The infratrochlear nerve passes superi- (C) Sensory nerves of the face travel orly on the medial surface of the supra- through it. orbital nerve. (D) The stylomastoid branch of the posterior (E) The supratrochlear nerve emerges auricular artery travels though it. through the supraorbital notch and (E) It is a common site of lesion for the glos- divides into branches. sopharyngeal nerve.

31. The lacrimal nerve innervates the lacrimal 36. Which of the following is NOT a branch of the gland with fibers ? (A) from the ophthalmic nerve (A) (B) borrowed via a communicating branch (B) from the maxillary nerve (C) lateral nasal artery (C) borrowed from the mandibular nerve (D) (D) from the optic nerve (E) retromandibular artery (E) from the sympathetic plexus 37. Which of the following is a branch of internal 32. The maxillary nerve gives off branches to carotid artery? which of the following ganglia? (A) supratrochlear artery (A) otic ganglion (B) facial artery (B) ciliary ganglion (C) posterior auricular artery (C) submandibular ganglion 0523-07 C hap 7 07/15/02 15:35 Page 113

Questions: 29–45 113

(D) superficial temporal artery (A) skin, connective tissue, auricular layer, (E) mental artery loose connective tissue, pericranium (B) skin, aponeurosis, connective tissue, 38. Which of the following is NOT true? loose connective tissue, pericranium (A) The angular artery is distributed to the (C) skin, connective tissue, aponeurosis, superior part of the cheek and lower loose connective tissue, paradural layer eyelid. (D) skin, connective tissue, aponeurosis, (B) The is distributed to the loose connective tissue, pericranium scalp in the back of the head. (E) skin, cartilaginous layer, aponeurosis, (C) The superficial temporal artery is distrib- loose connective tissue, pericranium uted to the parotid gland and duct. 43. Which of the following descriptions is correct? (D) The mental artery is distributed to facial muscles and skin of the chin. (A) The superior sagittal sinus runs superior (E) The supratrochlear artery is distributed to the brain in the inferior free border of to the muscles and skin of the scalp. the cerebral falx and ends in the straight sinus. 39. Which of the following is NOT contained (B) The inferior sagittal sinus is formed by within the parotid gland? the union of the superior sagittal sinus and the great cerebral vein and ultimately (A) facial nerve joins the confluence of the sinuses. (B) facial artery (C) The transverse sinuses follow S-shaped (C) retromandibular vein courses in the posterior cranial fossa and (D) parotid lymph nodes ultimately become the internal jugular (E) branches of the facial nerve veins. (D) The occipital sinus lies in the convex 40. Parasympathetic fibers from CN IX travel to attached border of the cerebral falx, run- the parotid gland via which of the following ning from the crista galli to the internal nerves? occiptal protuberance. (A) auriculotemporal nerve (E) The cavernous sinus is situated on each (B) great auricular nerve side of the sella turcica. (C) directly from the glossopharyngeal 44. The cavernous sinus receives blood from all of nerve the following EXCEPT (D) external carotid nerve plexus (E) retromandibular nerve (A) superior and inferior petrosal sinuses (B) superior ophthalmic veins 41. Parasympathetic fibers from CN IX synapse in (C) inferior ophthalmic veins which ganglion before traveling to the parotid (D) superficial middle cerebral vein gland? (E) sphenoparietal sinus (A) otic ganglion (B) ciliary ganglion 45. Which of the following nerves is NOT con- tained within the cavernous sinus? (C) submandibular ganglion (D) pterygopalatine ganglion (A) optic nerve (E) trigeminal ganglion (B) oculomotor nerve (C) trochlear nerve 42. Which of the following lists the layers of the (D) trigeminal nerve (specifically the scalp in the correct order? ophthalmic division) (E) abducent nerve 0523-07 C hap 7 07/15/02 15:35 Page 114

114 7: The Head and Neck

46. Which of the following does NOT contribute aqueduct . . . 4th ventricle . . . interven- to innervation of the dura mater? tricular foramen . . . subarachnoid space (A) ophthalmic division of the trigeminal (E) lateral ventricles . . . straight sinus . . . 3rd nerve ventricle . . . confluence of the sinuses . . . 4th ventricle . . . median and lateral aper- (B) maxillary division of the trigeminal tures . . . subarachnoid space nerve (C) mandibular division of the trigeminal 49. Which of the following is true regarding the nerve carotid canal? (D) facial nerve (A) The carotid canal is located in the infe- (E) C1, C2, and C3 rior surface of the sphenoid bone in the middle cranial fossa. 47. Which of the following is correct in respect to the brain? (B) Structures actually pass across rather than through the area of the carotid (A) The midbrain is composed of the epithal- canal, which is an artifact in dry amus, dorsal thalamus, and hypothala- and is actually closed by cartilage in life. mus and surrounds the third ventricle. (C) The carotid canal contains the internal (B) The pons is the rostral part of the brain- carotid artery with associated para- stem and lies at the junction of the mid- sympathetic nerves. dle and posterior cranial fossae. (D) The carotid canal contains the internal (C) The diencephalon lies dorsal to the pons carotid venous plexus connecting the cav- and medulla and ventral to the posterior ernous sinus and the internal jugular vein. part of the cerebrum, beneath the tento- (E) The greater petrosal nerve enters behind rium. and above the carotid canal and leaves (D) The cavity of the medulla oblongata anteriorly as the nerve of the pterygoid forms the inferior part of the fourth canal. ventricle. (E) The cerebrum occupies the middle and 50. Which of the following foramina is unpaired? posterior cranial fossae and houses the (A) foramen lacerum third and fourth ventricles. (B) greater palatine foramen 48. Which of the following correctly describes the (C) foramen cecum flow of cerebrospinal fluid? (D) lesser palatine foramen (A) lateral ventricles . . . cerebral aqueduct . . . (E) pterygoid canal 3rd ventricle . . . interventricular fora- men . . . 4th ventricle . . . median and 51. The oculomotor nerve emerges between which lateral apertures . . . subarachnoid space two arteries of the cerebral arterial circle? (B) lateral ventricles . . . interventricular fora- (A) posterior inferior cerebellar artery and men . . . 3rd ventricle . . . cerebral aque- anterior inferior cerebellar artery duct . . . 4th ventricle . . . median and (B) anterior inferior cerebellar artery and lateral apertures . . . subarachnoid space (C) lateral ventricles . . . interventricular (C) labyrinthine artery and superior cerebel- foramen . . . 3rd ventricle . . . median and lar artery lateral apertures . . . 4th ventricle . . . (D) superior cerebellar artery and posterior cerebral aqueduct . . . subarachnoid cerebral artery space (E) posterior cerebral artery and middle (D) lateral ventricles . . . median and lateral cerebral artery apertures . . . 3rd ventricle . . . cerebral 0523-07 C hap 7 07/15/02 15:35 Page 115

Questions: 46–58 115

52. What nerve emerges between the labyrinthine (E) lacrimal punctum . . . lacrimal lake . . . artery and the anterior inferior cerebellar lacrimal papilla . . . lacrimal sac . . . naso- artery? lacrimal duct (A) optic 56. Which of the following best describes the path- (B) trochlear way of parasympathetic fibers to the lacrimal (C) trigeminal gland? (D) abducent (A) CN VII . . . greater petrosal nerve . . . (E) facial nerve of the pterygoid canal . . . ptery- gopalatine ganglion . . . zygomatic 53. Which artery is NOT a branch of the vertebral branch of V2 . . . lacrimal branch of V1 artery system? (B) CN IX . . . lesser petrosal nerve . . . nerve (A) basilar of the pterygoid canal . . . pterygopala- (B) posterior cerebral tine ganglion . . . infraorbital branch of (C) posterior communicating V2 . . . lacrimal branch of V1 (D) anterior spinal (C) CN VII . . . greater petrosal nerve . . . otic ganglion . . . infraorbital branch of V ... (E) ophthalmic 2 lacrimal branch of V1 54. Which of the following arteries is correctly (D) CN VII . . . deep petrosal nerve . . . nerve matched with its distribution? of the pterygoid canal . . . pterygopala- tine ganglion . . . infratrochlear branch of (A) anterior cerebral . . . temporal and occip- V2 . . . lacrimal branch of V1 ital lobes of brain (E) CN V . . . ophthalmic branch of V1 ... (B) middle cerebral . . . inferior aspect of lacrimal branch of V1 cerebral hemispheres and occipital lobe (C) middle meningeal . . . calvaria 57. Which of the following muscles does NOT take (D) posterior cerebral . . . brainstem and its origin from the common tendinous ring? cerebellum (A) superior rectus (E) basilar . . . optic tract, cerebral pedun- (B) inferior rectus cle, internal capsule, and (C) lateral rectus thalamus (D) medial rectus 55. Which of the following lists best describes the (E) superior oblique pathway of tears from the lacrimal glands to the ? 58. Which of the following muscles is NOT inner- vated by the oculomotor nerve? (A) lacrimal ducts . . . lacrimal lake . . . lacrimal canaliculi . . . lacrimal sac . . . (A) levator palpebrae superioris nasolacrimal duct (B) lateral rectus (B) lacrimal ducts . . . lacrimal sac . . . lacri- (C) medial rectus mal canaliculi . . . lacrimal lake . . . (D) inferior oblique nasolacrimal duct (E) superior rectus (C) lacrimal canaliculi . . . lacrimal lake . . . lacrimal ducts . . . lacrimal sac . . . nasolacrimal duct (D) lacrimal canaliculi . . . lacrimal sac . . . lacrimal ducts . . . lacrimal lake . . . nasolacrimal duct 0523-07 C hap 7 07/15/02 15:35 Page 116

116 7: The Head and Neck

59. Which of the following muscles is NOT prop- (A) central artery of retina . . . runs adjacent erly matched with it main action? to optic nerve, supplying rods and cones (A) lateral rectus . . . abducts eyeball (B) . . . runs along medial rec- tus to supply lacrimal gland and frontal (B) superior rectus . . . elevates, adducts, and sinus rotates eyeball medially (C) short posterior cilaries . . . pierce sclera (C) inferior rectus . . . depresses, adducts, to supply choroid, rods, and cones and rotates eyeball medially (D) long posterior cilaries . . . supplies lens (D) superior oblique . . . adducts, elevates, and cornea and rotates eyeball laterally (E) posterior ethmoidal . . . supplies dorsal (E) inferior oblique . . . abducts, elevates, aspect of nose and rotates eyeball laterally

60. Which of the following nerves is correctly 64. Which of the following is NOT contained in matched with its distribution? the infratemporal fossa? (A) long ciliary . . . postsynaptic sympathetic (A) parts of temporal, lateral pterygoid, and fibers to the dilator pupillae medial pterygoid muscles (B) short ciliary . . . parasympathetic and (B) sympathetic fibers to lens and cornea (C) pterygoid venous plexus (C) frontal . . . conjunctiva and lacrimal (D) mandibular, inferior alveolar, buccal, gland and lingual nerves (D) infratrochlear . . . mucous membrane of (E) pterygopalatine ganglion sphenoidal and ethmoid sinuses (E) ethmoidal . . . conjunctiva and eyelids 65. Which of the following muscles is NOT a mus- cle of mastication? 61. Which of the following is true in respect to the (A) buccinator ciliary ganglion? (B) temporalis (A) Sympathetic fibers synapse in the ciliary (C) medial pterygoid ganglion. (D) lateral pterygoid (B) Afferent fibers from the iris and cornea (E) masseter pass through the ganglion. (C) The ganglion is located between the 66. The muscles of mastication are associated with optic nerve and medial rectus. which branchial arch? (D) Parasympathetic fibers in the ganglion are derived from CN VII. (A) first arch (E) Parasympathetic fibers in the ganglion (B) second arch are distributed to the retina and lens. (C) third arch (D) fourth arch 62. Which of the following arteries is NOT a branch (E) fifth arch of the ? (A) supraorbital 67. Which of the following depresses the mandible? (B) supratrochlear (A) lateral pterygoid (C) lacrimal (B) medial pterygoid (D) anterior ethmoidal (C) temporalis (E) infraorbital (D) masseter 63. Which of the following arteries is correctly (E) mylohyoid paired with its course and distribution? 0523-07 C hap 7 07/15/02 15:35 Page 117

Questions: 59–74 117

68. Which of the following is NOT a branch of the (E) It is transmitted through the incisive first (mandibular) part of the first part of the foramen. maxillary artery? 72. Which of the following is true in respect to the (A) deep auricular otic ganglion? (B) anterior tympanic (C) middle meningeal and accessory (A) It is located in the infratemporal fossa meningeal just inferior to the foramen rotundum. (D) inferior alveolar (B) Presynaptic parasympathetic fibers in the ganglion are derived from the facial (E) descending palatine nerve. 69. Which of the following is NOT a branch of (C) Postsynaptic parasympathetic fibers in the second (pterygoid) part of the maxillary the ganglion are destined for the parotid artery? gland. (D) Sympathetic fibers synapse in the gan- (A) deep temporal glion before continuing on to sweat (B) labyrinthine glands, erector pili muscles, and blood (C) pterygoid vessels. (D) masseteric (E) The ganglion contains cell bodies for (E) buccal fibers of the trigeminal nerve.

70. Which of the following branches of the third 73. Which nerve is NOT correctly matched with (pterygopalatine) part of the maxillary artery its distribution? is correctly paired with its distribution? (A) lingual nerve . . . sensation from the (A) infraorbital . . . maxillary molar and pre- anterior two-thirds of the tongue molar teeth, lining of maxillary sinus, (B) lingual nerve . . . sensation from floor of gingival mouth and lingual gingivae (B) posterior superior alveolar . . . inferior (C) chorda tympani nerve . . . taste fibers eyelid, lacrimal sac, side of nose, supe- from anterior two-thirds of tongue rior lip (D) chorda tympani nerve . . . secretomotor (C) pharyngeal . . . maxillary gingiva, pala- fibers to submandibular and sublingual tine glands, mucous membrane of roof glands of mouth (E) chorda tympani nerve . . . motor fibers to (D) artery of pterygoid canal . . . superior tensor tympani part of pharynx, auditory tube, tympanic cavity 74. The temporomandibular joint is what type of (E) descending palatine . . . roof of pharynx, joint? sphenoidal sinus, inferior part of audi- (A) fibrous joint tory tube (B) 71. Which of the following is NOT true in respect (C) modified hinge-type synovial joint to the ? (D) pivot-type synovial joint (E) saddle-type synovial joint (A) It is the termination of the maxillary artery. (B) It supplies the lateral nasal wall. (C) It supplies the nasal septum. (D) It supplies the paranasal sinuses. 0523-07 C hap 7 07/15/02 15:35 Page 118

118 7: The Head and Neck

75. Which of the following is correct in respect to (D) filiform papillae the hard palate? (E) fungiform papillae (A) The hard palate is composed primarily of the maxillary bones. 79. Which of the following muscles is NOT inner- vated by the hypoglossal nerve? (B) The incisive canal and foramen transmit the incisive nerves and greater palatine (A) genioglossus vessels. (B) hyoglossus (C) The greater palatine foramen transmits (C) styloglossus the nasopalatine nerves and greater (D) palatoglossus palatine nerve. (E) intrinsic muscles of the tongue (D) The lesser palatine foramina transmit the lesser palatine nerves but not the lesser 80. Which of the following tongue muscles is cor- palatine vessels. rectly paired with its action? (E) The descending palatine artery is a branch of the internal carotid artery. (A) superior and inferior longitudinal . . . curls tip and sides of tongue and short- 76. Which of the following palate muscles is NOT ens tongue innervated by the cranial part of the accessory (B) transverse . . . flattens and broadens nerve through a pharyngeal branch of the tongue vagus nerve via the pharyngeal plexus? (C) vertical . . . narrows and elongates tongue (A) tensor veli palatini (D) genioglossus and hyoglossus . . . elevates posterior part of tongue (B) levator veli palatini (E) palatoglossus . . . depresses and retracts (C) palatoglossus tongue (D) palatopharyngeus (E) musculus uvulae 81. Which of the following is true in respect to in- nervation of the tongue? 77. Which of the following is correctly matched with its action? (A) The chorda tympani nerve innervates the mucosa of the anterior two-thirds (A) musculus uvulae . . . pulls uvula of the tongue in respect to general sensa- inferiorly tion (touch and temperature). (B) palatopharyngeus . . . pulls walls of phar- (B) The chorda tympani nerve innervates ynx inferiorly, posteriorly, and laterally the anterior two-thirds of the tongue in during swallowing respect to special sensation (taste). (C) palatoglossus . . . depresses posterior (C) The lingual nerve innervates the poste- part of tongue and draws soft palate rior one-third of the tongue in respect away from tongue to general sensation (touch and temper- (D) levator veli palatini . . . depresses soft ature). palate during swallowing and yawning (D) The lingual nerve innervates the poste- (E) tensor veli palatini . . . opens auditory rior one-third of the tongue in respect to tube during swallowing and yawning special sensation (taste). (E) The glossopharyngeal nerve innervates 78. Which of the following is NOT a type of lin- the area of the tongue just anterior to the gual papilla? epiglottis in respect to both general and (A) vallate papillae special sensation. (B) foliate papillae 82. How do parasympathetic and taste fibers from (C) bacilliform papillae the chorda tympani reach their destination? 0523-07 C hap 7 07/15/02 15:35 Page 119

Questions: 75–89 119

(A) via the lingual nerve, a branch of the 86. Which of the following is NOT an opening to mandibular division of the trigeminal the pterygopalatine fossa? nerve (A) pterygomaxillary fissure (B) via the lingual nerve, a branch of the (B) sphenopalatine foramen glossopharyngeal nerve (C) inferior orbital fissure (C) via the lingual nerve, a branch of the vagus nerve (D) foramen rotundum (D) via intermingled fibers in the otic ganglion (E) foramen ovale (E) via the submandibular nerve, a branch of 87. Which of the following is NOT contained in the hypoglossal nerve the pterygopalatine fossa? 83. Which of the following taste sensations is cor- (A) third part of maxillary artery rectly paired with its tongue region? (B) maxillary nerve (A) savoriness . . . posterior part (C) nerve of the pterygoid canal (B) sourness . . . apex (D) pterygopalatine ganglion (C) bitterness . . . apex (E) optic nerve (D) saltiness . . . lateral margins 88. Which of the following foramina are NOT prop- (E) sweetness . . . posterior part erly matched with the structures they transmit? 84. Which of the following vessels is NOT cor- (A) inferior orbital fissure . . . ophthalmic rectly paired with its respective area of supply nerve, infraorbital vessels, orbital or drainage? branches of pterygopalatine ganglion (A) dorsal lingual arteries . . . supply sub- (B) infraorbital foramen and canal . . . infra- mandibular gland orbital nerve and vessels (B) deep . . . supplies anterior (C) palatovaginal canal (pharyngeal) tongue canal . . . pharyngeal nerves from maxil- lary nerve and pterygopalatine ganglion (C) sublingual artery . . . supplies sublingual and pharyngeal branch of maxillary gland and floor of mouth artery (D) dorsal lingual veins . . . accompany the (D) zygomaticofacial foramen . . . zygoma- lingual artery ticofacial nerve and vessels (E) deep lingual veins . . . drain the apex of (E) zygomaticotemporal foramen . . . the tongue, joining the sublingual vein zygomaticotemporal nerve and vessels 85. Which of the following is NOT correct? 89. The nerve of the pterygoid canal is composed (A) The parotid gland is supplied by of which of the following? branches of the external carotid and (A) lesser petrosal nerve and deep petrosal superficial temporal arteries. nerve (B) The submandibular gland is supplied by (B) greater petrosal nerve and deep petrosal the . nerve (C) The submandibular gland is innervated (C) greater petrosal nerve and lesser petrosal by the parasympathetic fibers of the nerve glossopharyngeal nerve that synapsed in the submandibular ganglion. (D) maxillary nerve and deep petrosal nerve (D) The sublingual glands are supplied by (E) maxillary nerve and greater petrosal the sublingual and submental arteries. nerve (E) The sublingual glands are innervated by parasympathetic fibers of the facial nerve. 0523-07 C hap 7 07/15/02 15:35 Page 120

120 7: The Head and Neck

90. The nerve of the pterygoid canal does NOT (C) greater palatine nerve innervate which of the following? (D) anterior ethmoidal nerve (A) lacrimal gland (E) posterior ethmoidal nerve (B) palatine glands 96. Which of the following is NOT correct regard- (C) mucosal glands of nasal cavity ing innervation of the ? (D) mucosal glands of upper pharynx (E) submandibular gland (A) The auricle is innervated by the great auricular nerve and auriculotemporal 91. Which of the following paranasal sinuses com- nerve. municates with the nasal cavity in the superior (B) The external surface of the tympanic meatus? membrane is innervated by the auricu- lotemporal nerve and even a small (A) posterior ethmoidal sinuses branch of the vagus. (B) frontal sinus (C) The pharyngotympanic tube is inner- (C) middle ethmoidal sinuses vated by the tympanic plexus (fibers from (D) sphenoidal sinus the facial and glossopharyngeal nerves). (E) maxillary sinus (D) The internal surface of the tympanic membrane is innervated by the 92. Where does the nasolacrimal duct communi- glossopharyngeal nerve. cate with the nasal cavity? (E) Sensory cell bodies of the vestibulo- (A) superior meatus cochlear nerve are located in the genicu- late ganglion. (B) middle meatus (C) inferior meatus 97. Which of the following correctly describes a (D) nasopharynx wall of the tympanic cavity and its underlying (E) sphenoidal sinus structure? (A) tegmental roof . . . cochlea, contained in 93. Which of the following is a separate bone? the promontory (A) superior nasal concha (B) floor . . . superior bulb of internal jugular (B) middle nasal concha vein (C) inferior nasal concha (C) medial wall . . . dura mater of the middle (D) crista galli cranial fossa (E) (D) anterior wall . . . mastoid cells and facial nerve 94. Which of the following does NOT supply the (E) posterior wall . . . carotid canal medial and lateral walls of the nasal cavity? 98. Which of the following is NOT contained in (A) sphenopalatine artery the tympanic cavity? (B) anterior and posterior ethmoidal arteries (C) (A) auditory (D) (B) tympanic plexus (E) superior labial artery (C) chorda tympani nerve (D) lesser petrosal nerve 95. Which of the following does NOT innervate (E) stapedius and tensor tympani muscles the nasal mucosa? 99. Which of the following is NOT true in respect (A) sphenopalatine nerve to the pharyngotympanic tube? (B) nasopalatine nerve 0523-07 C hap 7 07/15/02 15:35 Page 121

Questions: 90–105 121

(A) The salpingopharyngeus closes the tube. (D) mastoid canaliculus . . . transmits auricu- (B) It opens posterior to the inferior meatus lar branch of the vagus of the nasal cavity. (E) tympanic canaliculus . . . tympanic (C) It equalizes pressure in the middle ear branch of the glossopharyngeal nerve with atmospheric pressure. (D) The tensor veli palatini and levator veli 103. Which of the following is true? palatini work together to open the tube. (A) The cochlear labyrinth is composed of (E) It is supplied by the ascending pharyn- the utricle, the saccule, and three semi- geal artery, , circular canals. and artery of the pterygoid canal. (B) The contains per- ilymph. 100. What innervates the tensor tympani? (C) The basilar membrane secures the coch- (A) maxillary nerve lear duct to the spiral canal of the cochlea. (B) mandibular nerve (D) The spiral membrane forms the floor of (C) chorda tympani the cochlear duct. (D) (E) The saccule is continuous with the coch- lear duct through the ductus reuniens. (E) vagus 104. Which of the following is NOT true in respect 101. Which of the following is true? to typical cervical vertebrae? (A) The malleus articulates with the stapes (A) They have short, bifid spinous processes. and is moved by the tensor tympani. (B) The inferior facets of articular processes (B) The round window is an opening in the are directed inferoposteriorly, and supe- medial wall of the tympanic cavity lead- rior facets are directed superoposteriorly. ing to the vestibule of the inner ear and is closed by the base of the stapes. (C) The transverse processes contain a fora- men transversarium, which transmits (C) The tensor tympani assists in preventing the vertebral vein and artery except for damage to the internal ear when one C7, where it transmits only the vertebral hears loud noises. artery. (D) The stapedius is innervated by the (D) The vertebral foramen is large and chorda tympani. triangular. (E) The stapedius pulls the stapes anteriorly (E) The vertebral body is small, with a con- and flattens its base, loosening the annu- cave superior surface and a convex infe- lar ligament and increasing oscillatory rior surface. range. 105. Which fascial layer is NOT correctly matched 102. Which of the following foramina is NOT cor- with the structures it encloses? rectly matched with its function? (A) superficial cervical fascia . . . platysma (A) cochlear aqueduct . . . allows bony labyrinth to communicate with sub- (B) investing layer of deep cervical fascia . . . arachnoid space; also contains trapezius and sternocleidomastoid labyrinthine vein (C) pretracheal layer of deep cervical (B) aqueduct of the vestibule . . . transmits fascia . . . suprahyoid muscles endolymphatic duct, an artery, and a (D) prevertebral layer of deep cervical vein fascia . . . longus colli, longus capitis, (C) internal acoustic meatus . . . transmits scalenes, deep cervical muscles vestibulocochlear nerve and vestibular (E) carotid sheath . . . carotid arteries, inter- artery nal jugular vein, vagus 0523-07 C hap 7 07/15/02 15:35 Page 122

122 7: The Head and Neck

106. Which of the following is NOT a superior 111. Which of the following is innervated by the attachment of the trapezius? trigeminal nerve? (A) lateral surface of mastoid process (A) mylohyoid (B) medial third of superior nuchal line (B) geniohyoid (C) external occipital protuberance (C) stylohyoid (D) ligamentum nuchae (D) posterior belly of the digastric (E) spinous processes of C7–T12 (E) thyrohyoid

107. Which of the following is NOT correct in re- 112. Which of the following is NOT an anterior ver- spect to the posterior triangle of the neck? tebral muscle? (A) Its anterior boundary is formed by the (A) longus colli posterior border of the SCM. (B) longus capitis (B) Its posterior boundary is formed by the (C) splenius capitis anterior border of the trapezius. (D) rectus capitis anterior (C) Its inferior boundary is formed by the (E) rectus capitis lateralis middle third of the clavicle. (D) Its roof is formed by the platysma. 113. Of the following, which is innervated by the (E) Its floor is formed by the muscles cov- dorsal rami? ered by the prevertebral layer of deep (A) splenius capitis cervical fascia. (B) levator scapulae 108. The anterior triangle of the neck does NOT con- (C) posterior scalene tain which of the following smaller triangles? (D) middle scalene (A) supraclavicular triangle (E) anterior scalene (B) 114. Which of the following NEVER exists? (C) (D) (A) middle thyroid artery (E) (B) middle thyroid vein (C) thyroid ima artery 109. Which of the following muscles is NOT con- (D) parathyroid veins tained in the posterior cervical triangle? (E) anterior jugular vein (A) splenius capitis 115. Which of the following contains PAIRED (B) levator scapulae laryngeal cartilages? (C) middle scalene (D) posterior scalene (A) thyroid, cricoid, epiglottic (E) stylohyoid (B) arytenoid, corniculate, cuneiform (C) arytenoid, cricoid, epiglottic 110. Which of the following is a suprahyoid muscle? (D) corniculate, cricoid, cuneiform (A) mylohyoid (E) cuneiform, corniculate, epiglottic (B) sternohyoid 116. Which of the following intrinsic laryngeal (C) omohyoid muscles is NOT innervated by the recurrent (D) sternothyroid laryngeal nerve? (E) thyrohyoid (A) transverse arytenoids (B) oblique arytenoids 0523-07 C hap 7 07/15/02 15:35 Page 123

Questions: 106–126 123

(C) cricothyroid 122. Which of the following has an insertion on the (D) posterior cricoarytenoid pharyngeal tubercle of the occipital bone? (E) lateral cricoarytenoid (A) superior constrictor (B) middle constrictor 117. Which of the following is the primary tensor of (C) palatopharyngeus the vocal fold? (D) salpingopharyngeus (A) cricothyroid (E) stylopharyngeus (B) thyroarytenoid (C) vocalis 123. Which of the following passes through the gap (D) aryepiglottic between the superior constrictor and the ? (E) posterior cricoarytenoid (A) tensor veli palatini (B) ascending palatine artery 118. Which of the following abducts the vocal fold? (C) glossopharyngeal nerve (A) vocalis (D) stylohyoid ligament (B) transverse arytenoids (E) stylopharyngeus (C) oblique arytenoids (D) thyroarytenoid 124. Which of the following passes through the gap (E) posterior cricoarytenoid between the superior and middle constrictors? (A) glossopharyngeal nerve 119. Which of the following is the sensory nerve of (B) levator veli palatini the ? (C) internal laryngeal nerve (A) external laryngeal nerve (D) superior laryngeal artery (B) recurrent laryngeal nerve (E) superior laryngeal vein (C) internal laryngeal nerve (D) paratracheal nerve 125. Which of the following passes through the (E) inferior thyroid nerve gap between the middle and inferior con- strictors? 120. Which of the following are commonly referred (A) superior laryngeal artery to as the adenoids? (B) stylopharyngeus (A) pharyngeal tonsils (C) vagus nerve (B) submandibular glands (D) recurrent laryngeal nerve (C) palatine tonsils (E) inferior laryngeal artery (D) lingual tonsils (E) sublingual glands 126. Which of the following passes through the gap inferior to the inferior constrictor? 121. Which of the following is NOT innervated by (A) vagus nerve the cranial root of the accessory nerve? (B) internal laryngeal nerve (A) middle constrictor (C) superior laryngeal nerve (B) inferior constrictor (D) inferior laryngeal artery (C) palatopharyngeus (E) superior laryngeal vein (D) salpingopharyngeus (E) stylopharyngeus 0523-07 C hap 7 07/15/02 15:35 Page 124

124 7: The Head and Neck

127. Which of the following is correct? (C) superior ophthalmic veins (A) Le Fort I fracture: horizontal fracture of (D) great cerebral vein the maxillae (E) meningeal veins (B) Le Fort I fracture: fracture through the maxillary sinuses, infraorbital foramina, 132. An epidural hematoma consists of blood from lacrimals, and ethmoids which vessel? (C) Le Fort II fracture: horizontal fracture (A) middle meningeal artery through superior orbital fissures, eth- (B) cerebral veins moid and nasal bones, and greater wings (C) internal carotid artery of the sphenoids (D) circle of Willis (D) Le Fort III fracture: horizontal fracture of (E) vertebral artery the maxillae (E) Le Fort III fracture: fracture through the 133. Cerebral compression is NOT attributed to maxillary sinuses, infraorbital foramina, which of the following? lacrimals, and ethmoids (A) intracranial collections of blood 128. Which of the following is NOT present at birth? (B) obstruction of CSF flow (A) styloid process (C) intracranial tumors or abscesses (B) mastoid process (D) edema of brain (C) external occipital protuberance (E) viral accumulation at blood-brain barrier (D) tympanic membrane 134. Ptosis results from a lesion of which nerve? (E) clavicles (A) optic nerve 129. The inferior alveolar nerve is best blocked at (B) oculomotor nerve which location for dental work? (C) trochlear nerve (A) mental foramen (D) trigeminal nerve (B) greater palatine foramen (E) abducens nerve (C) less palatine foramen 135. Horner syndrome is caused by a lesion of which (D) mandibular foramen of the following? (E) lingual foramen (A) oculomotor nerve 130. A lesion to the zygomatic branch of CN VII (B) trigeminal nerve would cause which of the following? (C) facial nerve (A) the inability to empty food from the (D) vagus nerve vestibule of the cheeks (E) cervical sympathetic trunk (B) a drooping corner of the mouth (C) a ringing in the ear 136. Which of the following is NOT a symptom of Horner syndrome? (D) paralysis of the muscles of mastication (E) a drooping lower eyelid (A) pupillary constriction (B) ptosis 131. The facial veins make clinically important con- (C) sinking in of one eye nections with the cavernous sinus through (D) absence of sweating on face and neck which veins? (E) lack of lacrimation (A) lingual veins (B) trigeminal veins 137. A lesion of the hypoglossal nerve would result in which of the following? 0523-07 C hap 7 07/15/02 15:35 Page 125

Questions: 127–146 125

(A) loss of taste on posterior one-third of (D) pressure from herniating uncus on the tongue nerve or fracture in the cavernous sinus (B) deviation of protruded tongue toward (E) acoustic neuroma unaffected side (C) deviation of protruded tongue toward 142. A laceration or contusion in the parotid region affected side or a fracture of the temporal bone might dam- (D) inability to retract tongue age which nerve? (E) loss of salivation (A) trigeminal nerve (B) abducens nerve 138. What type of injury or condition might cause (C) facial nerve a lesion to the olfactory tract? (D) glossopharyngeal nerve (A) fracture involving optic canal (E) vagus nerve (B) fracture of cribriform plate (C) intracerebral clot in occipital lobe of brain 143. Which nerve might be damaged by a fracture involving the cavernous sinus? (D) pituitary tumor (E) epidural hematoma (A) olfactory tract (B) optic nerve 139. Sagging of the soft palate, deviation of the (C) abducens nerve uvula to the normal side, and hoarseness (D) facial nerve might be caused by a lesion to which nerve? (E) vestibulocochlear nerve (A) facial nerve (B) glossopharyngeal nerve 144. The stylopharyngeus is associated with which branchial arch? (C) vagus nerve (D) accessory nerve (A) first (E) hypoglossal nerve (B) second (C) third 140. A superficial neck laceration might result in (D) fourth which abnormal finding? (E) sixth (A) paralysis of the scm and superior fibers of the trapezius, drooping of the 145. The arytenoid and cricoid cartilages and laryn- shoulder geal connective tissue are formed by what? (B) paralysis of the mylohyoid, anterior (A) lateral plate mesoderm belly of the digastric, tensor tympani, (B) paraxial mesoderm and tensor veli palatini (C) neural crest (C) paralysis of the posterior belly of the (D) ectodermal placodes digastric, stylohyoid, and stapedius (E) endoderm (D) anosmia (E) tinnitus 146. Which of the following prominences is NOT correctly paired with the structures formed 141. The eye is turned down and out. What type from it? and site of lesion do you expect? (A) frontonasal: forehead, bridge of nose (A) fracture of cribriform plate (B) maxillary: lateral portion of upper lip (B) stretching of a nerve as it courses around (C) medial nasal: of upper lip, crest the brainstem and tip of nose (C) laceration or contusion in the parotid (D) lateral nasal: alae of nose region (E) mandibular: cheeks 0523-07 C hap 7 07/15/02 15:35 Page 126

126 7: The Head and Neck

DIRECTIONS (Questions 147 through 151): Identify the anatomical features indicated on the art below. 0523-07 C hap 7 07/15/02 15:35 Page 127

Questions: 147–156 127

DIRECTIONS (Questions 152 through 156): Identify the anatomical features indicated on the art below. 0523-07 C hap 7 07/15/02 15:35 Page 128

128 7: The Head and Neck

DIRECTIONS (Questions 157 through 161): Identify the anatomical features indicated on the art below. 0523-07 C hap 7 07/15/02 15:35 Page 129

Questions: 157–171 129

DIRECTIONS (Questions 162 through 166): Identify the anatomical features indicated on the art below.

DIRECTIONS (Questions 167 through 171): Identify the anatomical features indicated on the art below. 0523-07 C hap 7 07/15/02 15:35 Page 130

130 7: The Head and Neck

DIRECTIONS (Questions 172 through 176): Identify the anatomical features indicated on the art below.

DIRECTIONS (Questions 177 through 181): Identify the anatomical features indicated on the art below. 0523-07 C hap 7 07/15/02 15:35 Page 131

Questions: 172–186 131

DIRECTIONS (Questions 182 through 186): Identify the anatomical features indicated on the art below. 0523-07 C hap 7 07/15/02 15:35 Page 132

Answers and Explanations

1. (A) The bones of the neurocranium include the trigeminal nerve, oculomotor nerve, trochlear frontal bone, paired parietal bones, paired tem- nerve, abducens nerve, and sympathetic fibers poral bones, the occipital bone, the sphenoid (Moore, p 846). bone, and the ethmoid bone (Moore, p 832). 11. (B) The foramen ovale transmits the accessory 2. (C) The bones of the facial skeleton (viscerocra- meningeal artery. The foramen spinosum trans- nium or splanchnocranium) include the vomer, mits the middle meningeal artery. The groove the mandible, inferior nasal conchae, the pala- of the greater petrosal nerve transmits the tine bones, the zygomatic bones, the maxillae, petrosal branch of the middle meningeal artery. the nasal bones, and the lacrimal bones (Moore, The jugular foramen transmits the meningeal p 832). branches of the ascending pharyngeal and oc- cipital arteries. The mastoid foramen transmits 3. (A) When the frontal suture persists, it is the meningeal branch of the occipital artery known as the metopic suture (Moore, p 834). (Moore, p 846).

4. (B) The external occipital protuberance is also 12. (A) The foramina in the cribriform plate trans- known as the inion (Moore, p 839). mit axons of olfactory cells in the olfactory epi- thelium (Moore, p 846). 5. (A) Lambda is the point on the calvaria at the junction of the sagittal and lambdoid sutures 13. (D) The optic canals transmit the optic nerves (Moore, p 842). and the ophthalmic arteries (Moore, p 846).

6. (C) The vertex is the superior point of the neu- 14. (A) The foramen rotundum transmits the max- rocranium in the midline (Moore, p 842). illary division of the trigeminal nerve (Moore, p 846). 7. (E) The foramen magnum is in the posterior cranial fossa (Moore, p 846). 15. (A) The foramen spinosum transmits the men- ingeal branch of the mandibular division of the 8. (A) The foramen cecum is located in the ante- trigeminal nerve (Moore, p 846). rior cranial fossa (Moore, p 846). 16. (E) The foramen magnum transmits the me- 9. (E) The anterior and posterior ethmoidal fora- dulla and meninges, vertebral arteries, spinal mina transmit anterior and posterior ethmoidal roots of the accessory nerve, dural veins, and the arteries and nerves, not emissary veins (Moore, anterior and posterior spinal arteries (Moore, p 846). p 846).

10. (B) The superior orbital fissure transmits the 17. (E) The jugular foramen transmits the glos- ophthalmic veins, ophthalmic division of the sopharyngeal nerve, vagus nerve, accessory

132 0523-07 C hap 7 07/15/02 15:35 Page 133

Answers and Explanations: 1–39 133

nerve, internal jugular vein, inferior petrosal 29. (B) The ophthalmic nerve gives rise to the naso- and sigmoid sinuses, and meningeal branches ciliary nerve, which in turn gives rise to the of ascending pharyngeal and occipital arteries infratrochlear nerve (Moore, p 860). (Moore, p 846). 30. (C) The lacrimal nerve passes through the pal- 18. (A) The buccinator, a muscle of facial expres- pebral fascia of the upper eyelid near the lateral sion, is innervated by the facial nerve. It presses canthus of the eye (Moore, p 860). the cheek against the molar teeth to assist in chewing, and it expels air from the oral cavity. 31. (B) The lacrimal nerve innervates the lacrimal It also draws the mouth to one side when act- gland with fibers borrowed via a communicat- ing unilaterally (Moore, p 851). ing branch from the maxillary nerve (Moore, p 859). 19. (E) The temporalis is a muscle of mastication, not of facial expression (Moore, p 851). 32. (D) The maxillary nerve gives off branches to the pterygopalatine ganglion (Moore, p 859). 20. (D) The mentalis elevates and protrudes the lower lip (Moore, p 851). 33. (D) The mental nerve is a branch of the tri- geminal nerve (Moore, p 863). 21. (E) The facial nerve innervates the orbicularis oculi (Moore, p 851). 34. (E) The zygomatic branch of CN VII inner- vates the inferior part of the orbicularis oculi 22. (C) The trigeminal nerve innervates the mas- (Moore, p 863). seter (Moore, p 921). 35. (D) The stylomastoid foramen is located be- 23. (B) The muscles of facial expression develop tween the styloid and mastoid processes of the from the second pharyngeal arch (Moore, p 852). temporal bone. CN VII and the stylomastoid branch of the posterior auricular artery travel 24. (E) The depressor septi inserts into part of the through it (Moore, p 862). nasal septum and widens the alar part of the during deep inspiration (Moore, 36. (E) The facial artery has the following branches: p 856). inferior labial, superior labial, lateral nasal, and angular (Moore, p 866). 25. (D) The orbicularis oculi is innervated by a zygomatic branch of the facial nerve (Moore, 37. (A) The supratrochlear and supraorbital ar- pp 851, 855–856). teries are branches of the internal carotid ar- tery, while most other arteries of the face are 26. (A) The external nasal nerve is a cutaneous branches of the (Moore, nerve derived from the ophthalmic nerve p 866). (Moore, p 857). 38. (C) The superficial temporal artery supplies 27. (A) The auriculotemporal nerve arises by two the facial muscles and skin of the frontal and roots that surround the middle meningeal temporal regions, while the transverse facial artery (Moore, p 861). artery supplies the parotid gland and duct as well as muscles and skin of the face (Moore, 28. (E) The lacrimal nerve is distributed to the p 866). lacrimal gland and a small area of skin as well as part of the conjunctiva. The external nasal 39. (B) Within the parotid gland are found the fa- nerve is distributed to skin on the dorsum of cial nerve and its branches, the retromandibu- the nose, including the tip of the nose (Moore, lar vein, the external carotid artery, and parotid p 860). lymph nodes (Moore, p 870). 0523-07 C hap 7 07/15/02 15:35 Page 134

134 7: The Head and Neck

40. (A) The parasympathetic component of the 50. (C) The foramen cecum is unpaired (Moore, glossopharyngeal nerve travels to the parotid p 846). gland via the auriculotemporal nerve (Moore, p 870). 51. (D) The oculomotor nerve emerges between the superior cerebellar artery and the posterior 41. (A) Parasympathetic fibers from CN IX syn- cerebral artery (Moore, p 894). apse in the otic ganglion before traveling to the parotid gland (Moore, p 870). 52. (D) The abducent nerve emerges between the labyrinthine artery and the anterior inferior 42. (D) The scalp is composed of skin, connective cerebellar artery (Moore, p 894). tissue, aponeurosis, loose connective tissue, and pericranium (Moore, p 872). 53. (E) The vertebral artery system has the fol- lowing branches: posterior communicating, 43. (E) The cavernous sinus is situated on each posterior cerebral, basilar, superior cerebellar, side of the sella turcica. It extends on each side anterior inferior cerebellar, posterior inferior from the superior orbital fissure to the petrous cerebellar, labyrinthine, and anterior spinal. part of the temporal bone (Moore, p 880). The ophthalmic artery is a branch of the inter- nal carotid artery system (Moore, p 895). 44. (A) The cavernous sinus receives blood from the superior ophthalmic veins, the inferior oph- 54. (C) The middle meningeal artery supplies thalmic veins, the superficial middle cerebral more blood to the calvaria than to the dura, vein, and the sphenoparietal sinus (Moore, which is supplied by the vertebral artery (Moore, p 880). pp 883, 895).

45. (A) The cavernous sinus contains the internal 55. (A) Tears flow from the lacrimal ducts across carotid artery with its small branches, the ca- the eye to the lacrimal lake. The tears then pass rotid plexus of sympathetic nerves, the oculo- through the lacrimal punctum on the lacrimal motor nerve, the trochlear nerve, the trigeminal papilla, into the lacrimal canaliculi, and into nerve (the ophthalmic division and occasion- the lacrimal sac, which is the widened end of ally the maxillary division), and the abducent the nasolacrimal duct (Moore, pp 901–902). nerve (Moore, p 882). 56. (A) Parasympathetic fibers from CN VII travel 46. (D) The dura mater is innervated by all three first via the greater petrosal nerve and then via divisions of the trigeminal nerve, C1–C3, and the nerve of the pterygoid canal to the ptery- possibly the vagus nerve (Moore, p 883). gopalatine ganglion, where they synapse with postsynaptic fibers. These fibers join the zygo-

47. (D) The cavity of the medulla oblongata forms matic branch of V2, which in turn conveys the

the inferior part of the fourth ventricle. fibers to the lacrimal branch of V1 (via a com- municating branch). This nerve delivers the 48. (B) Cerebrospinal fluid flows from each lateral parasympathetic fibers to the lacrimal gland ventricle through an interventricular foramen (Moore, p 903). into the 3rd ventricle, then through the cere- bral aqueduct into the 4th ventricle, and finally 57. (E) The superior oblique takes its origin from through median and lateral apertures into the the body of the sphenoid bone, while all four subarachnoid space (Moore, p 899). rectus muscles take origin from the common tendinous ring (Moore, p 910). 49. (D) The carotid canal is located in the inferior surface of the petrous temporal bone in the 58. (B) The lateral rectus is innervated by the ab- middle cranial fossa and contains the internal ducent nerve (Moore, p 910). carotid artery, sympathetic plexus, and internal carotid venous plexus (Moore, p 893). 0523-07 C hap 7 07/15/02 15:35 Page 135

Answers and Explanations: 40–80 135

59. (D) The superior oblique abducts, depresses, the nasopalatine and superior nasal nerves and rotates the eyeball medially (Moore, p 910). (Moore, pp 922, 950).

60. (A) The long ciliary nerves convey post- 72. (C) Postsynaptic parasympathetic fibers pass synaptic sympathetic fibers to the dilator pupil- from the ganglion to the parotid gland via the lae and afferent fibers from the iris and cornea auriculotemporal nerve (Moore, pp 922–923). (Moore, pp 911–912). 73. (E) The tensor tympani is innervated by the 61. (B) Afferent fibers from the iris and cornea pass mandibular division of the trigeminal nerve through the ciliary ganglion (Moore, p 912). (Moore, p 971).

62. (E) The infraorbital artery is a branch of the 74. (C) The temporomandibular joint is a modi- third part of the maxillary artery (Moore, p 913). fied hinge-type synovial joint (Moore, p 923).

63. (C) The short posterior pierce 75. (A) The hard palate is composed of the pala- the sclera near the optic nerve and supply the tine processes of the maxillary bones as well as choroid, which subsequently supplies the rods the horizontal plates of the palatine bones and cones of the retina (Moore, p 913). (Moore, p 935).

64. (E) The infratemporal fossa contains the otic— 76. (A) The tensor veli palatini is innervated by the not the pterygopalatine—ganglion (Moore, medial pterygoid nerve, which is a branch of p 920). the mandibular division of the trigeminal nerve (Moore, p 939). 65. (A) The buccinator is a muscle of facial expres- sion, not a muscle of mastication (Moore, p 921). 77. (E) The tensor veli palatini tenses the soft pal- ate and opens the mouth of the auditory tube 66. (A) The muscles of mastication are associated during swallowing and yawning (Moore, p 939). with the first branchial arch and are innervated by the trigeminal nerve (Moore, p 921). 78. (C) The four types of lingual papillae are the vallate papillae, foliate papillae, filiform papil- 67. (A) The lateral pterygoids, when acting lae, and fungiform papillae (Moore, pp 940–941). together, depress and protrude the mandible, with the assistance of the suprahyoid mus- 79. (D) The hypoglossal nerve innervates the four cles, , and gravity (Moore, intrinsic muscles of the tongue in addition to pp 921, 926). the following three extrinsic muscles of the tongue: genioglossus, hyoglossus, and stylo- 68. (E) The descending palatine artery is a branch glossus. The vagus innervates the palatoglos- of the third (pterygopalatine) part of the max- sus, which is both an extrinsic tongue muscle illary artery (Moore, pp 920, 922). and a palate muscle (Moore, p 942).

69. (B) The labyrinthine artery is a branch of the 80. (E) The genioglossus depresses the tongue and basilar artery, which is part of the vertebral assists in protrusion. The hyoglossus depresses artery system (Moore, p 895). and retracts the tongue. The styloglossus re- tracts the tongue and draws it up for swallow- 70. (D) The artery of the pterygoid canal supplies ing. The palatoglossus elevates the posterior the superior part of the pharynx, the auditory part of the tongue. The superior and inferior tube, and the tympanic cavity (Moore, p 922). longitudinal muscles curl the tip and sides of the tongue and shorten it. The transverse nar- 71. (E) The sphenopalatine artery is transmitted rows and elongates the tongue, assisting in pro- through the sphenopalatine foramen along with trusion. The vertical flattens and broadens the tongue, assisting in protrusion (Moore, p 942). 0523-07 C hap 7 07/15/02 15:35 Page 136

136 7: The Head and Neck

81. (B) In respect to general sensation (touch and by the sublingual and submental arteries and temperature), the mucosa of the anterior two- innervated by parasympathetic fibers of the thirds of the tongue is innervated by the lingual facial nerve (Moore, p 948). nerve, which is a branch of the mandibular division of the trigeminal nerve. The mucosa of 86. (E) Openings to the pterygopalatine fossa in- the posterior one-third of the tongue is inner- clude the pterygomaxillary fissure, sphenopal- vated by the lingual branch of the glossopha- atine foramen, inferior orbital fissure, foramen ryngeal nerve, and the mucosa of the area just rotundum, and pterygoid canal (Moore, p 950). anterior to the epiglottis is innervated by small branches of the internal laryngeal nerve, which 87. (E) The pterygopalatine fossa contains the third is a branch of the vagus. In respect to special part of maxillary artery, the maxillary nerve, the sensation (taste), the chorda tympani, which nerve of the pterygoid canal, and the pterygo- is a branch of the facial nerve, innervates the palatine ganglion (Moore, p 950). anterior two-thirds of the tongue except for the vallate papillae in the posterior one-third 88. (A) The inferior orbital fissure transmits the of the tongue, which are innervated by the lin- infraorbital and zygomatic branches of the max- gual branch of the glossopharyngeal nerve. illary nerve, the infraorbital vessels, the inferior Branches of the internal laryngeal nerve of the ophthalmic veins, and the orbital branches of vagus innervate the area anterior to the epi- the pterygopalatine ganglion (Moore, p 951). glottis in respect to taste (Moore, p 944). 89. (B) The greater petrosal nerve (parasympa- 82. (A) Parasympathetic and taste fibers from the thetic fibers of the facial nerve) joins the deep chorda tympani reach their destination by join- petrosal nerve (sympathetic nerves from the ing the lingual nerve, a branch of the mandi- internal carotid plexus) to form the nerve of the bular division of the trigeminal nerve (Moore, pterygoid canal (Moore, p 951). p 944). 90. (E) The nerve of the pterygoid canal inner- 83. (D) Sweetness is detected on the apex, or tip, vates the lacrimal gland, the palatine glands, of the tongue, saltiness on the lateral margins, the mucosal glands of the nasal cavity, and and sourness and bitterness on the posterior the mucosal glands of upper pharynx (Moore, part (Moore, p 944). p 951).

84. (A) The dorsal lingual arteries supply the 91. (A) The superior meatus is a narrow passage, posterior tongue and the palatine tonsil, while inferior to the superior nasal concha, in which the deep lingual artery supplies the anterior the posterior ethmoidal sinuses open. The mid- tongue. The sublingual artery supplies the sub- dle meatus contains openings to the frontal si- lingual gland and the floor of the mouth. The nus, maxillary sinus, and anterior and middle dorsal lingual veins accompany the lingual ar- ethmoidal sinuses (Moore, p 956). tery, and the deep lingual veins drain the apex of the tongue, ultimately joining the sublingual 92. (C) The nasolacrimal duct drains into the infe- vein (Moore, p 944). rior meatus (Moore, p 956).

85. (C) The parotid gland is supplied by branches 93. (C) The inferior nasal concha is a separate of the external carotid and superficial tempo- bone, whereas the superior and middle nasal ral arteries and innervated by the glossopha- conchae are simply extensions of the ethmoid ryngeal nerve. The submandibular gland is bone (Moore, p 832). supplied by the submental artery and inner- vated by the parasympathetic fibers of the fa- 94. (D) The medial and lateral walls of the nasal cial nerve that synapsed in the submandibular cavity are supplied by the sphenopalatine ar- ganglion. The sublingual glands are supplied tery, anterior and posterior ethmoidal arteries, 0523-07 C hap 7 07/15/02 15:35 Page 137

Answers and Explanations: 81–108 137

greater palatine artery, and superior labial ar- 100. (B) The mandibular division of the trigeminal tery (Moore, p 956). nerve innervates the tensor tympani (Moore, p 971). 95. (A) The nasal mucosa is innervated by the nasopalatine nerve, greater palatine nerve, ante- 101. (C) The tensor tympani assists in preventing rior ethmoidal nerve, and posterior ethmoidal damage to the internal ear when one hears loud nerve (Moore, p 956). noises by pulling the handle of the malleus me- dially, thereby tensing the tympanic membrane 96. (E) The auricle is innervated by the great auri- and reducing the amplitude of its oscillations cular nerve and auriculotemporal nerve. The (Moore, p 971). external surface of the tympanic membrane is innervated by the auriculotemporal nerve and 102. (C) The internal acoustic meatus transmits the even a small branch of the vagus. The inter- facial nerve, vestibulocochlear nerve, and laby- nal surface of the tympanic membrane is in- rinthine artery (Moore, p 976). nervated by the glossopharyngeal nerve. The pharyngotympanic tube is innervated by the 103. (E) The saccule is continuous with the cochlear tympanic plexus (fibers from the facial and glos- duct through the ductus reuniens (Moore, p 975). sopharyngeal nerves). The sensory cell bodies of the vestibulocochlear nerve are located in 104. (C) Typical cervical vertebrae (C3–C6) have the spiral and vestibular ganglia (Moore, pp 962, transverse processes with foramina transver- 967, 969, 975). saria, which transmit both the vertebral vein and artery. In C7, however, this foramen trans- 97. (B) The dura mater of the floor of the middle mits only the vertebral vein (Moore, p 996). cranial fossa is superior to the tegmental roof. The internal jugular vein is inferior to the floor. 105. (C) The pretracheal layer of deep cervical fas- The lateral membranous wall is formed by the cia encloses, or invests, the infrahyoid muscles epitympanic recess and contains the head of the (Moore, p 998). malleus. The medial labyrinthine wall sepa- rates the tympanic cavity from the internal ear, 106. (A) The superior attachments of the trapezius which contains the cochlea. The anterior ca- are the medial third of the superior nuchal rotid wall separates the tympanic cavity from line, external occipital protuberance, ligamen- the carotid canal. The posterior mastoid wall tum nuchae, spinous processes of C7–T12, and guards the mastoid cells and the facial nerve lumbar and sacral spinous processes (Moore, (Moore, p 967). p 1002).

98. (D) The tympanic cavity contains the audi- 107. (D) The posterior triangle of the neck contains tory ossicles, tympanic plexus, chorda tym- an anterior boundary formed by the poste- pani nerve, and stapedius and tensor tympani rior border of the SCM, a posterior boundary muscles (Moore, p 967). formed by the anterior border of the trapezius, an inferior boundary formed by the middle 99. (A) The pharyngotympanic tube opens poste- third of the clavicle, an apex where the SCM rior to the inferior meatus of the nasal cavity and trapezius meet on the superior nuchal line, and serves to equalize the pressure in the mid- a roof formed by the investing layer of deep cer- dle ear with atmospheric pressure. The tensor vical fascia, and a floor formed by the muscles veli palatini and levator veli palatini work covered by the prevertebral layer of deep cer- together to open the tube, which is supplied vical fascia (Moore, p 1003). by the ascending pharyngeal artery, middle meningeal artery, and artery of the pterygoid 108. (A) The anterior triangle contains the sub- canal (Moore, p 969). mandibular (digastric), submental, carotid, and muscular (omotracheal) triangles. The poste- 0523-07 C hap 7 07/15/02 15:35 Page 138

138 7: The Head and Neck

rior triangle contains the occipital and supra- strictors, palatopharyngeus, and salpingopha- clavicular (omoclavicular or subclavian) trian- ryngeus are innervated by the cranial root of gles (Moore, p 1004). the accessory nerve (Moore, p 1056).

109. (E) The posterior cervical triangle contains the 122. (A) The superior constrictor inserts on the splenius capitis, levator scapulae, middle sca- median raphe of the pharynx and the pharyn- lene, and posterior scalene (Moore, p 1004). geal tubercle on the basilar part of the occipital bone (Moore, p 1056). 110. (A) The suprahyoid muscles are the mylo- hyoid, geniohyoid, stylohyoid, and digastric 123. (B) The ascending palatine artery, levator veli muscles (Moore, p 1016). palatini, and pharyngotympanic tube pass through the gap between the superior con- 111. (A) The mylohyoid and anterior belly of the strictor and the skull (Moore, pp 1055, 1058). digastric are innervated by the trigeminal nerve (Moore, p 1016). 124. (A) The glossopharyngeal nerve, stylopharyn- geus, and stylohyoid ligament pass through 112. (C) The anterior vertebral muscles are the lon- the gap between the superior and middle con- gus colli, longus capitis, rectus capitis anterior, strictors (Moore, p 1058). and rectus capitis lateralis (Moore, p 1026). 125. (A) The superior laryngeal artery and vein and 113. (A) The only lateral vertebral muscle that is the internal laryngeal nerve pass through the innervated by dorsal rami is the splenius capi- gap between the middle and inferior constric- tis (Moore, p 1026). tors (Moore, p 1058).

114. (A) There is NOT a middle thyroid artery, but 126. (D) The inferior laryngeal artery and recurrent there is a middle thyroid vein (Moore, pp 1030– laryngeal nerve pass through the gap inferior 1033). to the inferior constrictor (Moore, p 1058).

115. (B) Arytenoids, corniculate, and cuneiform 127. (A) A Le Fort I fracture is a horizontal fracture cartilages are paired, whereas the thyroid, cri- of the maxillae (Moore, p 836). coid, and epiglottic are single (Moore, p 1038). 128. (B) The mastoid processes are absent at birth. 116. (C) All intrinsic laryngeal muscles are inner- Therefore, the facial nerves are close to the sur- vated by the recurrent laryngeal nerve except face when they emerge from the stylomastoid the cricothyroid, which is innervated by the foramina and may be injured during delivery external laryngeal nerve (Moore, p 1045). (Moore, p 847).

117. (A) The cricothyroid stretches and tenses the 129. (D) The site of anesthetic injection to block the vocal fold (Moore, p 1045). inferior alveolar nerve is the mandibular fora- men (Moore, p 861). 118. (E) The posterior cricoarytenoid abducts the vocal fold (Moore, p 1045). 130. (E) A lesion of the zygomatic branch of CN VII would cause paralysis of the orbicularis oculi 119. (C) The internal laryngeal nerve is the sensory and therefore a drooping of the lower eyelid. nerve of the larynx (Moore, p 1048). Subsequently, tears would fail to spread over the cornea, resulting in a corneal scar and there- 120. (A) The pharyngeal tonsils are commonly fore impaired vision (Moore, p 864). referred to as the adenoids (Moore, p 1051). 131. (C) The facial veins make clinically important 121. (E) The salpingopharyngeus is innervated by connections with the cavernous sinus through the glossopharyngeal nerve. The three con- 0523-07 C hap 7 07/15/02 15:35 Page 139

Answers and Explanations: 109–156 139

the superior ophthalmic veins. Infections in the CN III, resulting in a dilated pupil, ptosis, the orbit, nasal sinuses, and superior part of the face eye being turned down and out, and a loss of may lead to cavernous sinus thrombosis (Moore, pupillary reflex on the side of the lesion (Moore, p 883). p 1098).

132. (A) An epidural hematoma consists of blood 142. (C) A laceration or contusion in the parotid re- from the middle meningeal artery (Moore, gion, a fracture of the temporal bone, or a stroke p 886). might damage the facial nerve, resulting in paralysis of facial muscles, an eye that remains 133. (E) Cerebral compression is typically attributed open, a drooping mouth, a smooth-appearing to intracranial collections of blood, obstruction forehead (no wrinkles), dry cornea, and loss of of CSF flow, intracranial tumors or abscesses, taste in the anterior two-thirds of the tongue and edema of the brain (Moore, p 888). (Moore, p 1098).

134. (B) Ptosis (drooping upper eyelid) is caused by 143. (C) A fracture involving the cavernous sinus a lesion to the oculomotor nerve, which inner- might damage the oculomotor nerve or the vates the levator palpebrae superioris (Moore, abducens nerve (Moore, p 1098). p 903). 144. (C) The third arch is associated with the glos- 135. (E) Interruption of the cervical sympathe- sopharyngeal nerve, stylopharyngeus, and the tic trunk results in Horner syndrome (Moore, greater horn and lower portion of the body of p 912). the (Sadler, p 348).

136. (E) Symptoms of Horner syndrome include 145. (A) The arytenoid and cricoid cartilages and pupillary constriction, ptosis, sinking in of one laryngeal connective tissue are formed by lat- eye, and absence of sweating on the face and eral plate mesoderm (Sadler, p 345). neck (Moore, p 1030). 146. (E) The lower lip is formed from the mandibu- 137. (C) A lesion to the hypoglossal nerve (due to a lar prominence (Sadler, p 370). neck laceration or basal skull fracture) would result in the protruded tongue deviating to- 147. temporalis ward the affected side in addition to altered articulation (Moore, p 1098). 148. masseter

138. (B) Fracture of the cribriform plate might cause 149. buccinator a lesion to the olfactory tract, resulting in anos- mia or CSF rhinorrhea (Moore, p 1098). 150. depressor anguli oris

139. (C) Sagging of the soft palate, deviation of the 151. depressor labii inferioris uvula to the normal side, and hoarseness might be caused by a lesion to the vagus nerve at the 152. auriculotemporal nerve brainstem or in the neck (Moore, p 1098). 153. buccal nerve 140. (A) A neck laceration might damage the spi- nal root of the accessory nerve, resulting in 154. lingual nerve paralysis of the SCM and superior fibers of the trapezius and drooping of the shoulder (Moore, 155. inferior alveolar nerve p 1098). 156. mylohyoid nerve 141. (D) Pressure from the uncus, a fracture in the cavernous sinus, or aneurysms could damage 0523-07 C hap 7 07/15/02 15:35 Page 140

140 7: The Head and Neck

157. palatine glands 172. trigeminal ganglion

158. greater palatine nerve 173. maxillary nerve

159. greater palatine artery 174. anterior superior alveolar nerve

160. palatopharyngeal arch 175. pterygopalatine ganglion

161. uvula 176. posterior superior alveolar nerve

162. medial rectus 177. petrosquamous suture

163. inferior rectus 178. external acoustic meatus

164. superior rectus 179. mandibular fossa

165. superior oblique 180. mastoid process

166. lateral rectus 181. styloid process

167. inner hair cell 182. head of malleus

168. tectorial membrane 183. vestibular nerve

169. vestibular membrane 184. tympanic membrane

170. outer hair cell 185. stapes

171. basal membrane 186. tensor tympani 0523-08 BM (refs) 07/15/02 15:36 Page 141

References

Moore KL, Dalley AF: Clinically Oriented Anatomy, Sadler TW: Langman’s Medical Embryology, 8th ed. 4th ed. New York, Lippincott Williams and Wilkins, New York, Lippincott Williams and Wilkins, 2000. 1999.

141 This page intentionally left blank 0523-08 Index 07/15/02 15:36 Page 143

Index

A Anal columns, 71, 82 B Abdominal muscles Anal sphincter, 77, 86 Bacilliform papillae, 118, 135 internal, 49, 61 Anatomical snuff box, 16, 26 Basal membrane, 129, 140 transverse, 47, 60 Anconeus muscle, 14, 25 Biceps, short head of, 92, 103 Abdominal oblique aponeurosis Anesthesia, epidural, caudal, 5, 9 Biceps brachii muscle, 21, 25, 29 external, 59, 66 Ankle joint, 96, 106 Biceps femoris muscle, long head of, internal, 59, 66 dorsiflexion of, 92–93, 96, 104, 106 99, 107 Abdominal oblique muscle, external, grip of malleoli on trochlea and, Biceps femoris tendon, 97, 107 47, 48, 60 97, 106 Biceps muscle, 20, 28 “Abdominal policeman,” 62 injury of, 96, 106 Bile duct, 51, 52–53, 62, 63 Abdominal wall, anterolateral, 47, 60 Anococcygeal ligament, 74, 83 Bladder Abducens nerve, 125, 139 Antebrachial cutaneous nerve, apex of, 77, 86 Abducent nerve, 115, 134 medial, 21, 29 fundus of, 78, 86 Abductor digiti minimi brevis Anterior compartment, shin splints neck of, in females, 73, 83 muscle, 93, 104 and, 98, 107 separation from pubic bones in Abductor digiti minimi muscle, 98, Anus, 74, 84 females, 75, 84 107 Aorta, 39, 45, 49, 54, 61, 65 trigone of, 69, 80 Abductor hallucis muscle, 93, 104 abdominal, 74, 83 uvula of, 69, 80 Abductor pollicis longus muscle, 16, bifurcation of, 55, 64–65 Brachial artery, 14, 21, 25, 29 25, 26 Aortic arch, 32, 35, 36, 38, 42, 43, Brachialis muscle, 14, 20, 22, 24, 28, 29 Accessory hemiazygous vein, 36, 43 44–45 Brachial plexus, 13–14, 24 Aortic hiatus of diaphragm, 38, 44, Accessory nerve, cranial root of, 123, injury of, 19, 28 58, 66 138 Brachiocephalic vein, right, 39, 45 Appendicular artery, 52, 63 Acetabular notch, 87, 100 Brachioradialis muscle, 15, 25 Appendix, 52, 63 Acetabulum, 87, 100 Brain, ventricles of, fourth, 114, 134 Appendix testis, 72, 83 Acetylcholine, 8 Branchial arches, 125, 139 Arch of foot, longitudinal, medial, Acoustic meatus 96, 106 first, 116 external, 130, 140 Arcuate ligaments, lateral, 54, 65 Breast internal, 121, 137 Arcuate line, 48, 60–61 blood supply of, 34, 43 Acromioclavicular joint, 18, 27 Articular processes, cervical, 1, 7 cancer of, 31, 41 Acromion, fractures of, 18, 27 Arytenoid cartilage, 125, 139 Bronchi Adductor canal, 90, 102 Atria, 33, 42 left main, 32, 41 Adductor hiatus, 89, 101 Auricle, left, 39, 45 right main, 32, 41, 55, 65–66 Adductor magnus muscle, 89, 101 Auricular artery, posterior, Bronchopulmonary segments, 32, Adductor pollicis muscle, 17, 26 stylomastoid branch of, 112, 34–35, 41–42, 43 Adductor tubercle, 87, 100 133 Buccal nerve, 127, 139 Adenoids, 123, 138 Auriculotemporal nerve, 111, 113, Buccinator muscle, 110, 116, 126, 132, Alveolar nerve 127, 132, 134, 139 139 inferior, 124, 127, 138, 139 Auscultation, triangle of, 19, 27–28 Buck’s fascia, 85 superior, 130, 140 Axilla, 13–14, 24 Bulbocavernosus muscle, 77, 86 Alveoli, 32, 42 injury of, 20, 28 Bulbourethral glands, 69, 80 Ampullae Axillary artery, 14, 25 of rectum, support of, 74, 83 Axillary muscle, 19, 28 of uterine tubes, 70, 81 Axillary nerve, 13, 24, 25 C Anal canal, 71, 82 Azygous vein, 33, 35, 36, 42, 43 Calcaneofibular joint, 96, 106

143 0523-08 Index 07/15/02 15:36 Page 144

144 Index

Calcaneonavicular ligament, plantar, Cluneal nerves of arm, posterior, 12, 23 96, 106 inferior, 78, 86, 91, 103 brachial Calcaneus, 88, 96, 100, 106 superior, 91, 103 lateral, 25 Calculi, urinary, 75, 84 Coccyx, 67, 77, 79, 85 medial, 25, 34, 43 Calvaria, 109, 132 Cochlear duct, 121, 137 femoral, lateral, 49, 59, 61, 66 Caput medusae, 53, 63 Collateral ligaments Cystic artery, 53, 63 Cardiac plexus, 36, 44 fibular, 94, 104–105 Cystic duct, 52–53, 63 Cardiac vein, anterior, 33, 42 tibial, 94–95, 105 Cardiovascular system. See also Colon Heart ascending, 52, 55, 63, 66 D derivation of, 38, 44 sigmoid, 52, 63 Deep arteries Carotid artery, internal, 110, Connective tissue, loose, in of arm, 14, 24 112–113, 132, 133 extradural space, 3, 9 of thigh, 90, 102 Constrictor muscle, superior, 123, Carotid canal, 114, 134 Deferential artery, 76, 85 138 Carpals, 12, 23 Deltoid muscle, 12, 18, 23, 27 Conus arteriosus, 32, 42 Carpal tunnel, 17 Depressor anguli oris muscle, 126, Copula, 40, 45 Carpometacarpal joint, of thumb, 18, 139 Coracobrachialis muscle, 20, 22, 28, 27 Depressor labii inferioris muscle, 29 126, 139 Carpus, 12, 23 Cornea, afferent fibers from, 116 Caudal epidural anesthesia, 5, 9 Depressor septi muscle, 111, 132 Coronary arteries, 38, 44–45, 46–47 Diaphragm, 35, 43, 44, 47, 54, 65 Caval opening, 58, 66 right, 32, 35, 42, 43 Cavernous sinus, 113, 134 aortic hiatus of, 38, 44, 58, 66 Coronary sinus, 33, 42 central tendon of, 54, 65 fractures involving, 125, 139 Corpora cavernosa, 72, 74, 75, 78, 82, domes of, 44, 47 Celia, 54, 65 84, 85, 86 esophageal hiatus of, 38, 44, 55, 58, Celiac ganglion, 44, 47 Corpus spongiosum, 74, 75, 84, 85 64, 66 Celiac plexus, parasympathetic root Costal cartilage, 40, 45 inferior vena cava and, 44, 47 of, 55, 64 Costoclavicular artery, 34, 43 inspiration and, 54, 65 Celiac trunk, 57, 66 Cranial fossa, 109, 132 pelvic, musculofascial, 67, 79 Cephalic vein, 21, 29 anterior, 110, 132 Cerebellar arteries Cranial nerves Diaphragmatic crus, 55, 64 left, 54, 65 inferior, anterior, 115, 134 abducens (sixth), 125, 139 right, 58, 66 posterior, 114, 134 accessory (eleventh), 123, 138 Diaphragmatic hernias, 55, 64 superior, 114, 134 facial (eleventh), 111, 132 Digestion, 55, 66 Cerebral compression, 124, 139 facial (seventh), 114, 124, 125, 134, Dorsalis pedis artery, 97, 98, 107 Cerebrospinal fluid, flow of, 114, 134 138, 139 Dorsal nerve root, 33, 42 Cervical fascia, deep, 121, 137 glossopharyngeal (ninth), 123, 138 Dorsal rami, 3, 8, 34, 43, 122, 138 Cervical spine, 1, 5, 7, 8 hypoglossal (twelfth), 118, Ductus deferens, 49, 61 lesions in, 15, 25 124–125, 135, 139 Duodenojejunal junction, 51, 62 Cervical triangle, posterior, 122, 138 oculomotor (third), 114, 115, 124, Duodenum, 51, 53, 62, 64 Cervix, uterine, 70, 81 134–135, 139 optic (second), 113, 119, 134, 136 C-shaped curve of, 52, 63 Chest pain, 34, 42, 44, 47 digestion in, 55, 66 Chordae tendineae, 33, 42 trigeminal (fifth), 110, 122, 132, 138 vagus (tenth), 39, 44, 45, 47, 50, 55, suspensory muscle of, 51, 62 Chorda tympani nerve, 117, 118–119, 61–62, 64, 70, 81, 125, 139 3rd part of, 56, 65 135, 136 Cremaster fascia, 49, 61 Dura mater, innervation of, 114, 134 Chromaffin cells, 55, 65 Cremaster muscle, 49, 61 Chyle cistern, 55, 65 Cribriform plate Ciliary arteries, posterior, short, 116 foramina in, 110, 132 E Ciliary ganglion, 116 fracture of, 125, 139 Ear, innervation of, 120, 137 Ciliary nerves, long, 116 Cricoarytenoid muscle, posterior, Elbow joint, 18, 27 Circular folds, of jejunum, 51, 62–63 123, 138 innervation of, 16, 26 Circumflex artery, 35, 43 Cricoid cartilage, 125, 139 Emissary veins, 110, 132 femoral, medial, 90, 102 Cricothyroid muscle, 123 Epididymis, 49, 61 medial, 94, 104–105 Cruciate ligaments, 95, 105 Epidural anesthesia, caudal, 5, 9 Clavicle, 11, 18, 23, 27, 33, 42 Crural fascia, 88, 101 Epidural hematomas, 124, 139 “Clawhand,” 19, 28 Cubital fossa, 15, 25 Epidural space, 3, 8 Cleft hand, 20, 28 Cutaneous nerve Epigastric artery Clitoris antebrachial inferior, 48, 59, 60, 66 development of, 72–73, 83 lateral, 15, 25 superior, 59, 66 prepuce of, 78, 86 medial, 21, 29 Episiotomy, 74, 84 0523-08 Index 07/15/02 15:36 Page 145

Index 145

Epo-ophoron, 72, 82–83 Flexor hallucis longus muscle, 93, Gluteus minimus muscle, 91, 97, 99, Erector spinae muscles, 2, 7 104 102, 107 Esophageal hiatus of diaphragm, 38, Flexor pollicis brevis muscle, 17, 26 Gracilis muscle, 89, 101 44, 55, 58, 64, 66 Flexor pollicis longus muscle, 15, 16, Great toe, lateral deviation of, 96, 106 Esophagus, 55, 62, 65–66 25, 26 Gubernaculum, 49, 61 compression of, 36, 43 Foot Gynecoid pelvis, 73, 83 Ethmoidal foramina, anterior, 110, dorsiflexion of, 93, 98, 104, 107 132 eversion of, 93, 104 Ethmoidal sinuses, posterior, 120 inversion of, 93, 104 H Extensor carpi radialis longus Foramen cecum, 114, 134 Hair cells tendon, 16, 25 Foramen magnum, 109, 110, 132 inner, 129, 140 Extensor digitorum brevis muscle, Foramen ovale, 119, 136 outer, 129, 140 98, 107 Foramen rotundum, 110, 132 Hallux valgus, 96, 106 Extensor digitorum longus muscle, Foramen spinosum, 110, 132 Hamstring muscles, 91, 92, 101, 103 92–93, 98, 104, 107 Foramen transversarium, 121, 137 Hand, innervation of, 16–17, 26 Extensor pollicis brevis muscle, 16, 25 Foregut, ventral wall of, 38, 45 Heart Extensor retinaculum muscle, Fornix, vaginal, posterior, 70, 80–81 atria of, 32, 33, 42 inferior, 93, 104 Fourchette, 74, 84 interventricular septum of, 32, 42 Extradural space, 3, 8 Fractures location of, 32, 42 Extraperitoneal cavity, 49, 61 of acromion, 18, 27 pain and, 44, 47 Eyelids, lower, drooping, 124, 138 of femoral neck, 87, 97, 100, 107 sympathetic stimulation in, 44, Le Fort I, 124, 138 46–47 of scapula, 18, 27 valves of, 33, 42 F temporal, 125, 139 ventricles of, 33, 42 Facet joints, thoracic, 34, 42 tibial, 88, 100 Helicine arteries, 74, 84 Facial artery, 112, 113, 133 Frenulum, of labia minora, 74, 84 Hemiazygous vein, 35, 36, 43 Facial expression, muscles of, 111, Frontal bone, 109, 132 Hepatic artery 132 Frontal nerve, 112, 133 common, 53, 57, 63, 66 Facial nerve, 111, 114, 125, 132, 134, Frontal suture, 109, 132 proper, 57, 66 139 Fundus Hepatic veins, 53, 63 Facial skeleton, 109, 132 of bladder, 78, 86 Hepatoduodenal ligament, 53, 63 Facial veins, 124, 138–139 of uterus, 77, 86 Hernias Falciform ligament, 56, 57, 65, 66 diaphragmatic, 55, 64 Fallopian tubes. See Uterine tubes inguinal, direct, 48, 61 Fascia lata, 88, 100–101 G Hinge joints, 18, 27, 117, 135 Fat, in extradural space, 3, 9 Gallbladder, neck of, 53, 63 Hip joint Femoral artery, 90, 102 Gastric artery, left, 57, 66 blood supply to, 94, 104–105 Femoral canal, 90, 102 Gastrocnemius muscle, 93, 104 flexion of, 97, 107 Femoral nerve, 59, 66, 89, 90, 101–102 Gastrocolic ligament, 57, 66 fractures of, 87, 100 Femoral sheath, 89, 101–102 Gastroduodenal artery, 51, 62 hyperextension of, 94, 104 Femoral triangle, 89, 101 Gastrohepatic ligament, 62 rotation of, 97, 107 Femur, 87, 100 Gastrolienal ligament, 57, 66 Hoarseness, 125, 139 head of Gastro-omental artery, left, 50–51, Horner syndrome, 124, 139 blood supply to, 90, 102 56, 62, 65 Humerus, 18–19, 27 ligament of, 94, 104 Geniculate ganglion, 120, 137 condyle of, 11, 23 length of, 87, 100 Genital swellings, 72, 83 Humpback (hunchback), 5, 8 neck of Genital tubercle, 72–73, 83 Hypogastric nerve, pelvic, 68, 79 blood supply to, 90, 102 in male, 72, 83 Hypogastric plexus, inferior, 68, 79 fractures of, 87, 97, 100, 107 Genitofemoral nerve, 59, 66 Hypoglossal nerve, 118, 124–125, Fibula, 88, 93, 100, 104 genital branch of, 49, 61 135, 139 neck of, 97, 107 Genu valgum, 97, 106 Hypomere, 20, 29 Fibularis brevis muscle, 93, 104 Glans penis, 71, 74, 75, 82, 84–85 Hypothenar eminence, 17, 26 Fibularis longus tendon, 96, 106 Glenohumeral joint, 17, 18, 26, 27 Fibular nerve, common, 92, 97, Glenoid cavity, 11, 23 103–104, 107 Glossopharyngeal nerve, 123, 138 I Flexor carpi radialis muscle, 15, 25 Gluteal artery, superior, 75, 84 Ileocecal fold, inferior, 56, 65 Flexor digitorum brevis muscle, 98, Gluteal nerve, superior, 68, 79, 91, Ileocolic artery, 52, 63 107 103 Ileum, innervation of, 51, 62 Flexor digitorum profundus muscle, Gluteus maximus muscle, 90–91, 102 Iliac artery, internal, 68–69, 76, 80, 15, 25 Gluteus medius muscle, 91, 102 85 0523-08 Index 07/15/02 15:36 Page 146

146 Index

Iliac spines, superior, anterior, 67, 79 muscle stabilizing, 94, 104–105 Lumbricals, 17, 98, 107 Iliofemoral ligament, 94, 104 stabilization of, 97, 106–107 Lunate surface, of acetabulum, 87, Iliohypogastric nerve, 59, 66 “unhappy triad” of injuries to, 95, 100 Ilioinguinal nerve, 48, 61 105 Lungs Iliopsoas muscle, 88, 97, 101, 107 Kyphosis, 5, 8 development of, 38, 45 Iliopubic tract, 49, 61 gas exchange in, 32, 42 Ilium, abdominal aorta and, 55, left, bronchopulmonary segments 64–75 L of, 34–35, 43 Incisive foramen, 117, 135 Labia minora, 72, 82 right, 32, 41 Inferior vena cava, 39, 44, 45, 47, 53, development of, 73, 83 surfactant in, 44, 47 55, 63, 65 frenulum of, 74, 84 sympathetic stimulation of, 44, 47 Infraorbital artery, 116, 120, 136–137 Labyrinthine artery, 115, 117, 134, , breast cancer Infraspinatus muscle, 12, 23 135 and, 31, 41 Infratemporal fossa, 116 Lacrimal gland, parasympathetic Infratrochlear nerve, 112, 133 fibers to, 115, 134 Inguinal canal, 48, 61 Lacrimal nerve, 111, 112, 132, 133 M Inguinal fossae, medial, 48, 61 Lacrimation, 124 Malleoli Inguinal hernias, direct, 48, 61 Lacunar ligament, 49, 61 grip on trochlea, 97, 106 Inguinal ligament, 47–49, 60, 61 Lambda, 109, 132 lateral, 87, 100 Inguinal rings, 48, 61 Lambdoid suture, 109, 132 medial, 87, 100 deep, 48, 60 Laryngeal artery Malleus, head of, 131, 140 Inion, 109, 132 inferior, 123, 138 Mammary glands, 34, 43 Inspiration, diaphragm and, 54, 65 superior, 123, 138 , depression of, 116–117, Intercarpal joints, 18, 27 Laryngeal cartilages, 122, 138 135 Intercostal arteries Laryngeal muscles, 122–123, 138 Mandibular foramen, 124, 138 anterior, 34, 43 Laryngeal nerve, internal, 123, 138 Mandibular fossa, 130, 140 posterior, 34, 43 Larynx Mandibular nerve, 121, 137 Intercostal muscles, 34, 43 connective tissue of, 125, 139 Mandibular prominence, 125, 139 Intercostal veins, posterior, 36, 43 sensory muscle of, 123, 138 Manubrium, of sternum, 40, 45 Intercostobrachial nerve, 12, 23, 34, Lateral cord, 21, 29 Masseter muscle, 111, 126, 132, 139 43 Lateral plate mesoderm, 125, 139 Mastication, muscles of, 110, 116, 132 Interosseous muscles, dorsal, 17, 26 , 19, 28 Mastoid process, 124, 130, 138, 140 Interosseous nerve, anterior, 16, 26 Le Fort I fractures, 124, 138 lateral surface of, 122, 137 Interventricular septum, 32, 42 Leptomeningeal space, 4, 8 Maxillary artery, 117, 135 Intervertebral discs, cervical, 1, 7 Levator ani muscle, 74, 76, 77, 83, 85, Maxillary bones, 118, 135 Iris, afferent fibers from, 116 86 Maxillary nerve, 112, 130, 133, 140 Ischial bursa, 91, 102 Levatores costarum, 34, 43 Medial epicondyle, 19, 27 Ischial tuberosities, 68, 70–71, 77, 79, Levator scapulae muscles, 13, 24 Median nerve, 15–17, 20, 21, 25, 26, 81, 85, 91, 92, 102, 103 Ligament of Treitz, 51, 62 28, 29 , 71, 72, 77, Ligamentum arteriosum, 39, 45 recurrent branch of, 17, 26 81, 82, 86 Limbs, innervation of, 3, 8 Mediastinum, 35, 43 Linea alba, 47, 60 Medulla oblongata, 114, 134 Lingual arteries, dorsal, 119, 136 Meningeal artery, middle, 111, 115, J Lingual nerve, 118–119, 127, 136, 139 124, 132, 134, 139 Jejunum Lingual papillae, 118, 135 Meninges circular folds of, 51, 62–63 Linguals, 32, 39, 41, 45 arterial branches to, 110, 132 innervation of, 51, 62 of left lung, 34–35, 43 nerve branches to, 110, 132 Jugular foramen, 110, 132 , elevation and protrusion of, Meningomyelocele, 5, 9 Jugular vein, internal, left, 39, 45 111, 132 Menisci, 95, 105 Lithotripsy, 75, 84 lateral, 94, 95, 105 Liver pain related to, 96, 106 K caudate lobe of, 62 medial, 94–95, 105 Kidneys. See also Renal entries lobes of, 56, 65 Mentalis muscle, 111, 132 diaphragmatic hernias and, 55, 64 round ligament of, 53, 63 Mental nerve, 112, 133 innervation of, 54, 65 Lobster claw deformity, 20, 28 Mesenteric artery posterior surfaces of, 53, 64 Longissimus muscle, 2, 7 inferior, 74, 84 Knee joint, 89, 101 Longus capitis muscle, 3, 8 superior, 56, 65 flexion of, 93, 104 Longus colli muscle, 2, 7 Mesenteric vein, superior, 56, 65 ligaments of, 95, 105 Lordosis, during pregnancy, 8 Mesentery, root of, 51, 62 menisci of. See Menisci Lumbar spine, 1, 5, 7, 9 Mesoderm, 38, 44 0523-08 Index 07/15/02 15:36 Page 147

Index 147

Mesometrium, 76, 85 Omental appendices, 52, 63 pelvic splanchnic nerves and, 55, Mesonephric system, 72, 82–83 Omental foramen, 62 65 Mesosalpinx, 73, 83 Omentum postsynaptic, 4, 8, 117, 135 , second, 88, 100 greater, 62 in otic ganglion, 117, 135 Metopic suture, 109, 132 lesser, 57, 62, 66 in pulmonary plexus, 44, 47 Midclavicular planes, 47, 60 Ophthalmic arteries, 110, 115, 116, testicular, 50, 61–62 Midgut, embryonic development of, 132, 134 Parasympathetic stimulation, 56, 65 Ophthalmic nerve, 111, 112, 132, 133 engorgement of erectile tissue Musculocutaneous nerve, 15, 21, 25, Ophthalmic veins, superior, 124, in bulbs of vestibule by, 72, 82 29 138–139 Paraumbilical vein, 53, 63 Musculophrenic arteries, 34, 43 Optic nerve, 113, 119, 134, 136 Paraurethral glands, 69, 75, 80, 84 Mylohyoid muscle, 122, 138 Orbicularis oculi muscle, 111, 112, Paraxial mesoderm, rib formation Mylohyoid nerve, 127, 139 132, 133 from, 38, 44 Myoblasts, 20, 29 Orbital fissure Parotid duct, 113, 133 inferior, 119, 136 Parotid gland, 113, 133 superior, 110, 132 Parotid injuries, 125, 139 N Os, external, 70, 81 Patellar ligament, 95, 105 Nasal bones, 120 Osteoporosis, 5, 8 Pecten pubis, 67, 79 Nasal cavity Otic ganglion, 113, 117, 134, 135 Pectinate line, 71, 82 blood supply to, 120, 136–137 Ovarian arteries, 74, 83 Pectinate muscle, 33, 42 nasolacrimal duct communication Ovaries, suspensory ligament of, 70, Pectoral arteries, 35, 43 with, 120 81 Pectoralis major muscle, 18, 20, 27, Nasal conchi, inferior, 120 29 Nasal mucosa, innervation of, 120, Pectoralis minor muscle, 12, 23 137 P Pectoral nerve, lateral, 14, 21, 24, 29 Nasal nerve, external, 111, 132 Palate Pelvic cavity, 67, 79 Nasolacrimal duct, 120 hard, 118, 135 formation of, 67, 79 Navicular, 88, 100 muscles of, 118, 135 Pelvic floor, 68, 79 Navicular fossa, 71, 75, 82, 84–85 soft, sagging of, 125, 139 Pelvic inlet, 67, 79 Neck. See also Cervical spine Palatine artery Pelvic wall, 68, 79 posterior triangle of, 122, 137 ascending, 123, 138 Pelvis superficial lacerations of, 125, 139 descending, 117, 135 gynecoid, 73, 83 Nelaton’s line, 92, 103 greater, 128, 140 weak areas of, 68, 79 Nerve supply Palatine bones, 109, 132 Penis, 72, 82. See also Glans penis of back, 3, 8 Palatine glands, 128, 140 Pericardiacophrenic ligament, 33, 42 of upper limb, 12, 13–14, 24 Palatine nerve, greater, 128, 140 Pericardiacophrenic vessels, 39, 45 Neurocranium, 109, 132 Palatoglossus muscle, 118, 135 Pericardium, 33, 35, 42, 43 Palatopharyngeal arch, 128, 140 fibrous, 38, 44 Palmaris brevis muscle, 17, 26 Perineal body, 71, 74, 81, 84 O Palmaris longus muscle, 16, 26 Perineal membrane, 75, 84 Oblique aponeurosis, external, 48, 60 Palmaris longus tendon, 15, 25 Perineal muscle, transverse, 77, 86 Oblique fissure, 40, 45 Pampiniform plexus, 50, 61 deep, 75, 84 Oblique muscle Pancreas, 52, 54, 63, 65 Perineal pouch abdominal, internal, 59, 66 head of, 52, 63 deep superior, 115, 116, 129, 134, 140 Pancreatic duct, 51, 52–53, 62, 63 in females, 71, 81 Obturator artery, 69, 80 Pancreaticoduodenal arteries, 51, 62 muscles of, 71, 81–82 Obturator externus tendon, 94, 104 Papillary muscle, posterior, 33, 42 superficial Obturator fascia, 71, 81 Paramesonephric ducts, 72, 82–83 in males, 71, 81 Obturator internus muscle, 68, 76, Paranasal sinuses, ethmoidal, 120 muscles of, 71, 81–82 79, 85 Parasympathetic fibers, 4, 8, 36, 44 Perineum, 74, 84, 90, 102 Obturator nerve, 71, 75, 82, 85 to bladder, 69, 80 boundaries of, 70, 81 Occipital bone, pharyngeal tubercle celiac plexus and, 55, 64 division into triangles, 70–71, 81 of, 123, 138 from chorda tympani, 118–119, Peritoneal cavity, 62 Occipital nerve, lesser, 3, 8 136 Peritoneal fluid, 62 Occipital protuberance, external, colonic, 52, 63 Peroneal nerve, deep, 98, 107 109, 132 inhibition of, 44, 46–47 Petrosal nerves, 119, 136 Oculomotor nerve, 114, 115, 124, to lacrimal gland, 115, 134 lesser, 120, 137 134–135, 139 in otic ganglion, 117, 135 Petrosal sinuses, 113, 134 Olfactory cells, axons of, 110, 132 to parotid gland, 113, 134 Petrosquamous suture, 130, 140 Olfactory tract, lesions to, 125, 139 of pelvic splanchnic nerves, 76, 85 Phallus. See Glans penis; Penis 0523-08 Index 07/15/02 15:36 Page 148

148 Index

Pharyngeal arches, muscles of facial Pubic bones, separation from Renal hilum, 54, 65 expression development bladder in females, 75, 84 Renal papillae, 53, 64 from, 111, 132 Pubic ramus Renal pyramids, 54, 65 Pharyngeal tonsils, 123, 138 inferior, 67, 77, 79, 85 Renal vein, left, 52, 63 Pharyngotympanic tube, 120–121, superior, 67, 79 Respiratory system, 38, 45 137 Pubic symphysis, 77, 85, 86 Retromammary space, 31, 41 Phrenic arteries, inferior, 35, 43, 54, anterior aspect of, 67, 79 Retromandibular artery, 112, 133 65 Pubococcygeus muscle, 73, 83 Retropubic space, 75, 84 Phrenic nerve, 39, 45 Pubovesical ligaments, 73, 83 , 12–13, 19, 24, 28 Phrenic veins, superior, 36, 43 Pudendal artery, internal, 92, 103 Ribs Piriformis muscle, 68, 76, 79, 85 Pudendal canal, 71, 81 elevation of, 34, 43 Piriformis syndrome, 92, 103 Pudendal nerve, 68, 71, 76, 79, 81–82, formation of, 38, 44 Plantaris muscle, 93, 104 85, 90, 102 transverse process articulation Plantaris tendon, grafting of, 97, 107 perineal branches of, 78, 86 with, 31, 41 Plantar nerve Pudendum, 74, 84 Rotator cuff injuries, 19, 28 lateral, 98, 107 Pulmonary arteries, 39, 45 Rotatores muscles, 2–3, 7 medial, 94, 104 right, 39, 45 Round ligaments Platysma, 122, 137 Pulmonary plexus, 44, 47 of liver, 53, 63 Pleura, parietal, 32, 41 Pulmonary veins, 32, 42 of uterus, 73, 83 Pleural cavity, 31, 41 inferior, left, 39, 45 Rugae, 56, 62, 65 Pleural fluid, 31, 41 left, 39, 45 Pleuropericardial membranes, 38, 44 Pylorus, 57, 66 Plicae circulares, of jejunum, 51, S 62–63 Saccule, 121, 137 Poland anomaly, 20, 29 Q Sacral spine, 70, 81 Popliteal artery, 95, 105 Quadrangular space, 14, 24 Sacrospinous ligament, 70, 81 Quadratus lumborum fasciae, 54, 65 Popliteal fossa, 92, 103 Sacrotuberous ligament, 77, 85 Quadratus lumborum muscle, 58, 66 floor of, 92, 103 Sagittal suture, 109, 132 Quadriceps femoris muscle, 89, 94, Popliteal ligament, oblique, 92, 95, Salivary glands, submandibular, 119, 97, 101, 104–105, 106–107 103, 105 120, 136 Popliteal vein, 88, 101 Saphenous nerve, 94, 104 Popliteus muscle, 93, 94, 104–105 Saphenous vein R Porta hepatis, 53, 63 great, 88, 92, 101, 103 Radial artery, 16, 26 Portal triad, 62 small, 88, 101 Radial nerve, 12, 17, 21, 23, 25, 26, 29 Portal vein, 51, 62 Sartorius muscle, 88–89, 90, 101, 102 Radius, 11, 23 Posterior longitudinal ligament, 2, 7 Rami communicantes, 4, 8 Scalp, layers of, 113, 134 Pott’s fracture-dislocation, 97, 106 Rectal artery, superior, 74, 84 Scaphoid, 12, 23 Pregnancy, lordosis during, 8 Rectal nerve, inferior, 78, 86 Scapula, 11, 23 Prepuce, of clitoris, 78, 86 Rectal venous plexus, internal, 71, 82 fractures of, 18, 27 Presynaptic fibers, 36, 44 Rectal vessels, superior, 71, 82 innervation of skin over, 14, 24 Prevertebral ganglia, 36, 44 Rectosigmoid junction, 70, 81 muscles attached to coracoid Prevertebral muscles, 35, 43 Rectouterine pouch, 77, 86 process of, 12, 23 Primary rami Rectovesical pouch, 78, 86 “winging” of, 19, 27 dorsal, 91, 103 Rectovesical septum, 73, 83 Scapular nerve, dorsal, 12–13, 24 ventral, 91, 103 Rectum, 52, 63 Sciatic foramen Pronator quadratus muscle, 15, 25 ampulla of, support of, 74, 83 greater, 68, 79 Prostate, 73, 75, 78, 83, 84, 86 male, anatomical relationships of, lesser, 75, 84, 90, 102 lateral lobes of, 75, 84 74, 83 Sciatic nerve, 68, 79, 92, 99, 103, 107 Prostatic ductules, 69, 80 Rectus abdominis muscle, 48, 59, 60, fibular division of, 92, 103 Prostatic sinus, 69, 80 66 Sciatic notch, lesser, 67, 79 Psoas, 55, 65 Rectus capitis posterior major Scoliosis, 5, 8–9 Pterygoid canal muscle, 3, 8 Scrotal ligament, 49, 61 artery of, 117, 135 Rectus femoris muscle, 89, 101 Scrotum nerve of, 119, 120, 136 Rectus muscle development of, 72, 83 Pterygoid muscle, lateral, 116–117, inferior, 129, 140 innervation of, 49, 61, 71, 75, 82, 85 135 lateral, 115, 129, 134–135, 140 Sella turcica, 113, 134 Pterygopalatine fossa, 119, 136 medial, 129, 140 Semimembranosus muscle, 95, 105 Pterygopalatine ganglion, 112, 116, superior, 129, 140 Seminal vesicles, 69, 73, 78, 80, 83, 86 130, 133, 140 Rectus sheath, 48, 60–61 Semitendinosus muscle, 92, 99, 103, Ptosis, 124, 139 Renal calyces, minor, 53, 64 107 0523-08 Index 07/15/02 15:36 Page 149

Index 149

Serratus anterior muscle, 47, 60 Suprahyoid muscles, 121, 122, 137, Thymus, 35, 43 Serratus posterior inferior muscle, 2, 138 Thyroid artery, 122, 138 7, 34, 43 Suprarenal arteries, superior, 54, 65 Thyroid disorders, 34, 42 Shin splints, 98, 107 Suprarenal cortex, 54, 65 Thyroid vein, inferior, 39, 45 Shoulder dislocations, 18, 27 Suprarenal glands, 54, 65 Tibia, 88, 93, 94, 100, 104–105 Sigmoid colon, 52, 63 innervation of, 54, 65 in genu valgum, 97, 106 Sinoatrial node, 33, 42 left, 54, 65 pain on lateral rotation on femur, Snuff box, 16, 26 Suprarenal medulla, 55, 65 96, 106 Soleus muscle, 99, 107 Supraspinatus muscle, 13, 24 Tibialis anterior muscle, 93, 98, 104, Sphenopalatine artery, 117, 135 Supraspinatus tendon, 19, 28 107 Sphenopalatine nerve, 120, 137 Supratrochlear artery, 112–113, 133 Tibial nerve, 94, 104 Spina bifida, 5, 9 Sural nerve, lateral, 92, 103–104 entrapment of, 96, 106 Spinal arteries, 2, 4, 7, 8 Surfactant, 44, 47 Tongue Spinal cord Suspensory ligament, of ovaries, 70, hypoglossal nerve lesions and, enlargement for innervation of 81 124–125, 139 limbs, 3, 8 Sustentaculum tali, 88, 100 innervation of, 118, 136 veins of, 4, 8 Sweat glands, 34, 36, 43, 44 muscles of, 118, 135 Spinalis muscle, 2, 7 Tonsils, pharyngeal, 123, 138 Spinal nerves, 4, 8, 33, 42 Sympathetic nerves, 35, 43 Trabeculae carneae, 32, 42 Spiral valve, 53, 63 Sympathetic nervous system, 36, 44 Transumbilical plane, 47, 60 Splanchnic nerves stimulation in heart and, 44, 46–47 Transversalis fascia, 47, 48, 49, 59, 60, greater, 44, 47, 55, 64 stimulation in lungs and, 44, 47 61, 66 least, 44, 47 Sympathetic plexus, 110, 132 lesser, 44, 47, 55, 64 Sympathetic trunks Transverse ligament, of knee, 95, 105 origin of, 44, 47 cervical, 124, 139 Transverse processes, 121, 137 pelvic, 52, 55, 63, 65, 68, 69, 79, 80 sacral, 68, 79 Transversospinalis muscles, 2, 7 parasympathetic fibers in, 76, 85 Syndactyly, 20, 28 Transversus abdominis aponeurosis, thoracic Synovial joints, 18, 27, 117, 135 59, 66 lesser, 38, 44 Transversus abdominis muscle, 59, 66 lower, 44, 47 Trapezius muscle, 11, 13, 23, 24, 137 Spleen, 52, 54, 56, 63, 65 T Trendelenburg sign, 91, 102–103 Splenic arteries, 50–51, 52, 56, 57, 62, Talus, 87, 88, 100 Triceps brachii muscle, 14, 24 63, 65, 66 Tarsal joint, transverse, 96, 106 lateral head of, 22, 29 , 2, 7, 122, 138 Taste fibers, from chorda tympani, long head of, 22, 29 Splenius cervicis muscle, 2, 7 118–119, 136 Triceps coxae, 91, 103 Spondylolysis, 73, 83 Taste sense, 118–119, 136 Triceps surae, 93, 104 Stapes, 131, 140 Tears, pathway followed by, 115, 134 Tricuspid valve, 33, 42 Sternal angle, 31, 41 Tectorial membrane, 129, 140 Trigeminal ganglion, 130, 140 Sternoclavicular joint, 17, 34, 43 Temporal artery, superficial, 113, 133 Trigeminal nerve, 122, 138 Sternum, manubrium of, 40, 45 Temporal bone, fracture of, 125, 139 maxillary division of, 110, 132 Stomach, 54, 65 Temporalis muscle, 111, 126, 132, 139 Trigone, of bladder, 69, 80 digestion in, 55, 66 Temporomandibular joint, 117, 135 Trochanter, greater, 92, 103 innervation of, 51, 62 Tendinous intersections, of rectus Trochlea, grip of malleoli on, 97, 106 rugae of, 56, 62, 65 abdominis muscle, 48, 60 Tunica albuginea, 49, 61 Styloid process, 130, 140 Tendinous ring, common, 115, 134 Tunica dartos, 75, 85 Stylomastoid branch foramen, 112, Teniae coli, 52, 63 Tunica vaginalis, 49, 61 133 Tensor tympani, 121, 131, 137, 140 Tympanic cavity, 120, 137 Stylopharyngeus muscle, 123, 125, Tympanic membrane, 131, 140 139 Tensor veli palatini, 118, 135 Subarachnoid space, 4, 8 Testes, 49, 61 Subclavian artery, groove for, 31, 41 development of, 49, 61 Subcostal nerve, 59, 66 thermoregulation and, 50, 61 U Submandibular gland, 119, 120, 136 Testicular artery, 68, 79 Ulna, 11, 22, 23, 29 Suboccipital triangle, 3, 8 Thermoregulation, of testes, 50, 61 Ulnar nerve, 15, 16–17, 19–21, 25–29 Subscapular artery, 13, 24 Thoracic artery, internal, 31, 35, 41, Umbilical arteries, 48, 61 Superior oblique muscle, 3, 8 43 Umbilical folds, 48, 61 Superior vena cava, 39, 45 Thoracic duct, 38, 44, 55, 64, 65 Umbilical ligament, median, 73, 83 Supinator muscle, 16, 25 Thoracic fascia, anterior, 38, 44 Umbilical vein, 53, 63 Supraclavicular nerves, 25 Thoracic spine, 1, 5, 7, 9, 34, 36, 42, Uncus, herniating, 125, 139 Supraclavicular triangle, 122, 43 “Unhappy triad” of knee injuries, 95, 137–138 Thoracoabdominal artery, 4, 8 105 0523-08 Index 07/15/02 15:36 Page 150

150 Index

Urachal remnants, 73, 83 V Vertebral foramina, thoracic, 1, 7 Ureters, 69, 74, 76, 80, 83, 85 Vagina, 74, 76, 84, 85 Vertex, of neurocranium, 109, 132 Urethra innervation of, 76, 85 Vesicle, inferior, 49, 61, 76, 85 external orifice of, 69, 80 posterior fornix of, 70, 80–81 Vestibulae male, 75, 84 Vaginal fornices, 72, 83 bulb of, 78, 86 prostatic part of, 69, 80 Vagus nerve, 50, 61–62, 70, 81, 125, female, 72, 74, 82, 84 Urethral folds, 73, 83 139 Vestibular glands, lesser, 74, 84 Urethral sphincter, external, 71, 75, left, 39, 45 Vestibular membrane, 129, 140 81, 84 posterior trunk of, 55, 64 Vestibular nerve, 131, 140 Urinary bladder. See Bladder Vasculature, of vertebral column, 2, Vestibulocochlear nerve, 120, 137 Urinary calculi, 75, 84 4, 7, 8 Visceral afferent fibers, 35, 43 Urogenital hiatus, 76, 85 Vasoconstriction, 36, 44 Vocal folds, tensor of, 123, 138 Uterine artery, ligation of, 75, 84 Veins, of spinal cord, 4, 8 Uterine tubes, 73, 77, 83, 86 Vulva, 74, 84 ampullae of, 70, 81 Ventricles Uterus, 76, 85 of brain, fourth, 114, 134 cervix of, 70, 81 left, 33, 42 W fundus of, 77, 86 Vertebrae. See also specific regions of Water, reabsorption of, 55, 66 mesentery of, 76, 85 spine Wrist joint, abduction of hand at, 16, position of, 70, 81 transverse processes of, 31, 41 26 round ligament of, 73, 83 Vertebral arch, in spondylolysis, 73, Uvula, 128, 140 83 of bladder, 69, 80 Vertebral arteries, 115, 134 X deviation of, 125, 139 Vertebral column, 1, 7 Xiphoid process, 40, 45